You are on page 1of 184

NGUYỄN HỮU ĐIỂN

PHƯƠNG PHÁP SỐ PHỨC


VÀ HÌNH HỌC PHẲNG

NHÀ XUẤT BẢN ĐẠI HỌC QUỐC GIA HÀ NỘI


©ebook 2.0 của cuốn sách nguyên gốc từ bản in, các bạn tham
khảo, cho ý kiến sai sót và lời khuyên tái bản. Mọi liên hệ
Tác giả: Nguyễn Hữu Điển
Điện thoại: 0989061951
Email: huudien@vnu.edu.vn
Email: nguyenhuudien@hus.edu.vn
Web: https://nhdien.wordpress.com
Web: https://vietex.blog.fc2.com/

Chịu trách nhiệm xuất bản:


Giám đốc: Nguyễn Văn Thỏa
Tổng biên tập: Nghiêm Đình Vỳ

Biên tập và sửa bản in:


Nguyễn Lan Hương
Trình bày bìa:
Ngọc Anh
Chế bản:
Nguễn Hữu Điển

PHƯƠNG PHÁP SỐ PHỨC VÀ HÌNH HỌC PHẲNG


Mã số: 01.120.DDH99-475.99
In 1000 cuốn, tại số 2 Phạm Ngũ Lão, XN in 15
Số xuất bản: 98/457/CXB. Số trích ngang 52KH/XB
In xong và nộp lưu chiểu tháng 5/2000
LỜI NÓI ĐẦU

Do nhu cầu phát triển của toán học, số phức đã ra đời từ thế
kỷ trước. Sau đó số phức lại thúc đẩy phát triển không những Toán
học mà còn các ngành khoa học khác. Ngày nay, số phức không thể
thiếu được trong các ngành khoa học lý thuyết cũng như kỹ thuật.
Thế nhưng số phức được học trong các trường phổ thông ở những
năm cuối cùng, chỉ mang tính chất giới thiệu. Chúng tôi biên soạn
cuốn sách này không phải để phổ biến số phức, mà chỉ dùng số
phức như là công cụ giải những bài toán hình học điển hình ở phổ
thông. Do vậy, chúng tôi trình bày sơ lược về số phức mà ta sẽ dùng
chứ không đi sâu nghiên cứu số phức, phần quan trọng là dùng số
phức để giải bài toán hình học, chúng tôi cố gắng phân loại những
bài toán hình học theo một dạng nào đấy để thấy mặt mạnh của
phương pháp số phức. Ngoài ra những bài tập trong cuốn sách này
là chọn lọc những bài toán hay trong hình học.
Để đọc tài liệu này, không cần yêu cầu bạn đọc biết trước về số
phức, chúng tôi sẽ giới thiệu ngắn gọn và các tính chất của số phức
để dùng sau này. Nếu bạn đọc còn bỡ ngỡ và tìm hiểu theo một
hướng khác, thì nên xem:
A.I. Markusevits,Số phức và ánh xạ bảo giác, NXB KHKT, 1987
N.C. Toàn, Tập cho học sinh giỏi toán làm quen dần với nghiên
cứu toán học, NXB GD, 1992.
Ngày nay số phức cũng là khởi đầu một ngành nghiên cứu mới
trong toán học, đó là hình học Fractal của thời đại vi tính. Hy vọng

3
4 Lời nói đầu

chúng tôi sẽ giới thiệu loại hình học mới này trong một cuốn sách
khác tiếp theo.
Với khuôn khổ một cuốn sách nhỏ không thể vẽ tất cả các hình
theo chỉ dẫn của bài tập, vì vậy bạn đọc với cây bút và tờ giấy trắng
hãy tự vẽ lấy hình theo chỉ dẫn. Nội dung cuốn sách bao gồm từ
Chương 1 đến Chương 3 là những khái niệm chính về số phức để
ta dùng sau này và cách tiếp cận số phức như một phương pháp để
giải các bài toán hình học phẳng. Những chương tiếp theo là dùng
số phức để khảo sát bài tập hình học phẳng theo các chủ đề. Chương
12 trả lời các bài tập hoặc gợi ý giải. Những chương còn lại chúng
ta bàn luận riêng về một khía cạnh mở rộng.
Chúng tôi cũng mong muốn đây là tài liệu tham khảo cho các
học sinh khá giỏi yêu thích môn toán, hoặc làm tài liệu cho các buổi
ngoại khoá về môn Toán đối với các thầy cô giáo. Trong biên soạn
chúng tôi cũng cố gắng tạo ra những chủ đề trong hình học để các
bạn say mê toán học làm việc tiếp tục. Lần đầu tiên biên soạn không
tránh khỏi sai sót và nhầm lẫn, mong các bạn đọc góp ý bổ sung và
sửa đổi.Tác giả bày tỏ lòng biết ơn sâu sắc GS Phan Huy Khải đã hết
sức giúp đỡ, chỉ dẫn và khuyến khích để cuốn sách ra mắt bạn đọc.
Sách được soạn bằng chương trình Pctex for Window 2.1, phông
chữ tiếng Việt và hình minh họa do chính tác giả soạn thảo và cài
đặt trong TEX. Mọi thư từ liên hệ với : Nguyễn Hữu điển, Viện Toán
học, Hộp thư 631, Bờ Hồ, Hà Nội, Việt Nam.
Chúc các bạn thành công.
Hà Nội, 2000
Chương 1
KHÁI NIỆM VỀ SỐ PHỨC

1.1. Định nghĩa số phức . . . . . . . . . . . . . . . . . . . . . . . . . . . . . . . . . . . . 5


1.2. Biểu diễn đại số của số phức . . . . . . . . . . . . . . . . . . . . . . . . . 7
1.3. Dạng lượng giác của số phức . . . . . . . . . . . . . . . . . . . . . . . . 9
1.4. Công thức Moa vrơ . . . . . . . . . . . . . . . . . . . . . . . . . . . . . . . . . . 12

1.1. ĐỊNH NGHĨA SỐ PHỨC


Từ thế kỷ trước do nhu cầu phát triển của toán học về giải những
phương trình đại số mà số phức đã xuất hiện. đã có nhiều nhà toán
học nghiên cứu về số phức và tìm cách biểu diễn hình học cho số
phức, điển hình là Gaus, Hamilton,... Còn ứng dụng của số phức thì
với khoa học hiện đại không thể thiếu được. Mục đích của chương
này trình bầy khái niệm đơn giản nhất về số phức mà ta sẽ sử dụng
về sau.
Có nhiều cách tiếp cận số phức, ở đây ta chọn cách định nghĩa
số phức theo tiên đề, đồng thời cũng giải thích các tiên đề đó bằng
hình học cho dễ hiểu. Như ta đã biết số thực được biểu diễn bởi một
đường thẳng có hướng, thường được gọi là trục số. Bây giờ, trong
mặt phẳng ta chọn một hệ tọa độ vuông góc, thì mỗi điểm Z của
mặt phẳng được xác định theo tọa độ (a, b) đối với hệ tọa độ đã cho.
Thường người ta ký hiệu cặp số thực (a, b) ứng với một điểm Z trên
6 Chương 1. Khái niệm về số phức

mặt phẳng. Như vậy với một hệ tọa độ cho trước thì tập hợp những
điểm trên mặt phẳng và tập hợp các cặp số (a, b) là một quan hệ
một-một. Mỗi điểm trên mặt phẳng tương ứng với một cặp số thực
và dựa vào đó ta sẽ xây dựng một tập hợp những số phức với điểm
trên mặt phẳng. Với mục đích ta đưa vào định nghĩa các phép toán
trên các cặp số thực sao cho các định luật của đại số vẫn còn đúng
như trong trường hợp số thực. Chúng ta chọn ba tiên đề sau:

1. Hai cặp số z1 = (a1 , b1 ) và z2 = (a2 , b2 ) bằng nhau nếu a1 = a2


và b1 = b2 .
2. Nếu cho hai cặp số z1 = (a1 , b1 ) và z2 = (a2 , b2 ), thì tổng của
chúng z = z1 + z2 là một cặp số z = (a, b), mà a = a1 + a2 và
b = b1 + b2 .
3. Nếu cho hai cặp số z1 = (a1 , b1 ) và z2 = (a2 , b2 ), thì tích của
chúng z = z1 z2 gọi là một cặp số z = (a, b), mà a = a1 a2 − b1 b2
và b = a1 b2 + a2 b1 .

Từ những định nghĩa trên ta có thể kiểm tra tất cả định luật của
đại số vẫn còn đúng như: tính bắc cầu của đẳng thức, tính đối xứng
và tính phân phối của các phép cộng và phép nhân ở trên. Và cũng
đưa vào phép trừ hoặc chia các cặp số (tất nhiên không chia cho số
0 = (0, 0), các bạn tự kiểm tra các tính chất như một bài tập).
Tập hợp tất cả những cặp số thực với các phép tính quan hệ
bằng nhau, phép cộng và phép nhân như ở trên gọi là tập hợp các
số phức.
Như vậy, cho một hệ tọa độ vuông góc trong mặt phẳng thì tập
hợp các số phức có thể đồng nhất với những điểm trên mặt phẳng
này.
Bây giờ, ta xét trường hợp đặc biệt là những điểm nằm trên
1.2. Biểu diễn đại số của số phức 7

trục hoành của hệ tọa độ, hay là những điểm có dạng (a, 0)
với a là số thực bất kỳ. Do (a1 , 0) + (a2 , 0) = (a1 + a2 , 0) và
(a1 , 0)(a2 , 0) = (a1 a2 , 0) như là phép cộng và phép nhân những tọa
độ ở trục hoành đối với các điểm này. Vì thế ta có thể đồng nhất các
điểm trên trục hoành với số thực, đáng lẽ phải viết (a, 0) ta chỉ viết
a.(ví dụ: (0, 0) = 0, (1, 0) = 1, ...).
Ta xét một số phức đặc biệt dạng (0, 1).
Tính (0, 1)(0, 1) = (−1, 0) = −1. Như vậy tồn tại một số phức bình
phương bằng một số thực. Theo truyền thống ta ký hiệu i = (0, 1).

1.2. BIỂU DIỄN ĐẠI SỐ CỦA SỐ PHỨC


Ta đã thấy rằng sự đồng nhất của số thực với tập hợp con của
số phức dạng (a, 0) = a với a là một số thực. Một số phức đặc biệt
i = (0, 1) và theo truyền thống người ta gọi là đơn vị ảo.
Ta xét tích của một số thực b = (b, 0) với đơn vị ảo
bi = (b, 0)(0, 1) = (0, b). đây là một điểm nằm trên trục tung với
tung độ b. Thế còn một điểm bất kỳ thì sao ? Do định nghĩa phép
cộng nên có thể biểu diễn z = (a, 0) + (0, b). Suy ra z = a + ib.
Một số phức viết dưới dạng z = a + ib gọi là dạng đại số của số
phức. Số thực a gọi là phần thực của z và được ký hiệu là Re(z),
còn số b gọi là phần ảo của z và được ký hiệu là Im(z). Mặt phẳng
chứa toàn bộ số phức gọi là mặt phẳng phức.
Trục hoành của hệ tọa độ vuông góc trong mặt phẳng phức gọi
là trục thực (chứa toàn bộ số thực). Trục tung gọi là trục ảo (chứa
toàn bộ số ảo).
Các phép tính cộng, trừ, nhân, chia những số phức viết dưới
8 Chương 1. Khái niệm về số phức

dạng biểu diễn đại số như sau:


(a + ib) + (c + id) = (a + c) + i(b + d),
(a + ib) − (c + id) = (a − c) + i(b − d),
(a + ib)(c + id) = (ac − bd) + i(ad + bc),
(a + ib) ac + bd bc − ad
= 2 2
+i 2 .
(c + id) c +d c + d2
Ba công thức đầu ta dễ dàng chứng minh được từ sự biểu diễn
đại số của số phức. Công thức cuối cùng hơi khó một chút bằng cách
tiến hành dựa vào ba công thức trên:
(a + ib) (a + ib)(c − id)
=
(c + id) (c + id)(c − id)
(ac + bd) + i(bc − ad) ac + bd bc − ad
= = 2 +i 2 .
c2 + d2 c + d2 c + d2
Trong cách chứng minh trên ta có dùng số phức (c − id) trong quá
trình biến đổi và số này có mối liên hệ chặt chẽ với số phức c + id.
Trong thực tế, để thuận tiện thực hiện các phép tính và biến đổi số
phức người ta đưa vào ký hiệu z̄ = a − ib và gọi là số liên hợp của
z = a + ib.
Những tính chất sau đây thường được dùng đối với số phức
liên hợp:
1) z + z̄ = 2a = 2Rez.
2) z z̄ = (a + ib)(a − ib) = a2 + b2 .
z1 z̄1
3) z1 .z2 = z̄1 z̄2 ; z1 + z2 = z̄1 + z̄2 ; = ;...
z2 z̄2
Tóm lại, f (x1 , ..., xn ) là một hàm hữu tỷ với hệ số thực, z1 , ..., zn
là những số phức bất kỳ sao cho f (z1 , ..., zn ) có nghĩa, khi đó
f (z1 , ..., zn ) = f (z̄1 , z̄2 , ..., z̄n ).
4) Một số phức z là một số thực khi và chỉ khi z̄ = z.
5) Nếu z̄ = −z, thì Rez = 0. Khi đó số z là hoàn toàn ảo.
1.3. Dạng lượng giác của số phức 9

1.3. DẠNG LƯỢNG GIÁC CỦA SỐ PHỨC


Trên mặt phẳng cho một hệ tọa độ vuông góc, sự biểu diễn số
phức theo những điểm trên mặt phẳng cho ta dễ dàng nghiên cứu
các phép toán trên số phức: Cho hai số phức dạng đại số z1 = a1 +
ib1 , z2 = a2 + ib2 . đó là hai điểm Z1 , Z2 trong hệ tọa độ vuông góc
ứng với số trên. điểm O là tọa độ gốc.
Ta nối điểm Z1 , Z2 với gốc O
−−→ −−→ y Z
và xác định vectơ OZ1 , OZ2 . Sau
Z1
đó dựng hình bình hành OZ1 ZZ2 .
Như vậy đỉnh thứ tư z = (a1 +
a2 , b1 + b2 ) biểu diễn tọa độ của Z2
số phức z1 + z2 như tổng của hai 0 x
số phức đã cho.
Z0
Do đó tổng hai số phức có thể
biểu diễn hình học như cộng hai
vectơ trong mặt phẳng. Hình 1.1.

Bởi vì mỗi điểm trên mặt phẳng tương ứng với một bán kính
−−→ −−→ −→
vectơ OZ và ta thấy ngay OZ1 + OZ2 = OZ, ta có nhận xét là khi
xem số phức như là những điểm trên mặt phẳng với hệ tọa độ gốc
O thì có thể xem số phức như là những vectơ trong mặt phẳng này,
chính điều nhận xét này mà ta áp dụng được số phức vào giải những
bài toán trong hình học phẳng.
Một số phức xác định như là một điểm trong mặt phẳng với hệ
tọa độ cho trước. Ngoài ra, một điểm trong mặt phẳng cũng hoàn
toàn xác định bởi hệ tọa độ cực: thật vậy, cho z = a + ib 6= 0 thì
−→
số phức này ứng với một vectơ OZ, ta ký hiệu r là độ dài bán kính
10 Chương 1. Khái niệm về số phức

vectơ này, còn ϕ là độ lớn của góc định hướng giữa trục hoành và
vectơ xác định số phức (góc có hướng dương là góc có chiều quay
trục hoành đến vectơ theo chiều ngược kim đồng hồ, góc có hướng
âm thì ngược lại).
Rõ ràng r là một số thực không âm. Nếu điểm z nằm trên trục
hoành thì số r chính là môđun của số thực tương ứng, vì vậy cho số
phức z ta cũng định nghĩa r là môđ un của z và ký hiệu là |z|.

Do đó r = a2 + b2 hoặc r2 = a2 + b2 = z z̄. Góc ϕ được gọi là
argumen của số phức và ký hiệu
y Z = a + ib
là arg z. Giá trị của ϕ có thể là
âm hoặc dương phụ thuộc vào r
hướng quay của trục hoành đến
nó. Có thể xác định ϕ bằng φ
a a
cos ϕ = = √ 0 x
r a2 + b2

b b
sin ϕ = = √ Hình 1.2.
r a2 + b2
argumen của số phức z 6= 0 nhận vô cùng nhiều giá trị. Nếu một giá
trị ϕ đã xác định thì argumen được xác định theo công thức

arg z = ϕ + 2kπ

k là một số nguyên. Thường thường ta chỉ dùng giá trị của argumen
trong khoảng [0, 2π).
Những số r và ϕ biểu diễn một tọa độ cực của z. Nếu cho một
điểm z = a + ib 6= 0, thì mối liên hệ giữa tọa độ cực và tọa độ vuông
góc như sau

a = r cos ϕ, b = r sin ϕ.
1.3. Dạng lượng giác của số phức 11

Khi đó số phức z có thể viết


z = r cos ϕ + ir sin ϕ = r(cos ϕ + i sin ϕ).
Một số phức viết theo dạng trên người ta gọi là dạng lượng giác
của số phức.
Cho hai số phức dưới dạng lượng giác z1 = r1 (cos ϕ1 + i sin ϕ1 )
và z2 = r2 (cos ϕ2 + i sin ϕ2 ). Ta có tính chất sau:
1) Nếu z1 trùng với z2 , thì môđun của chúng bằng nhau và ar-
gumen của chúng ϕ1 , ϕ2 khác nhau một số nguyên lần 2π.
2) Tích của hai số phức
z = z1 z2 = r1 (cos ϕ1 + i sin ϕ1 )r2 (cos ϕ2 + i sin ϕ2 )
= r1 r2 [(cos ϕ1 cos ϕ2 − sin ϕ1 sin ϕ2 )+
+ i(cos ϕ1 sin ϕ2 + i sin ϕ1 cos ϕ2 )]
= r1 r2 [cos(ϕ1 + ϕ2 ) + i sin(ϕ1 + ϕ2 )].

Như vậy, tích z của hai số phức viết dưới dạng lượng giác
z = r(cos ϕ + i sin ϕ), ở đó r là tích của r1 r2 hai môđ un của hai
thừa số. Hoặc là |z1 z2 | = |z1 ||z2 |, còn argumen ϕ là tổng ϕ1 + ϕ2 của
hai argumen thừa số, hay nói cách khác
arg z1 z2 = arg z1 + arg z2 .

Bằng phương pháp qui nạp dễ dàng chứng minh được


[r1 (cos ϕ1 + i sin ϕ1 )][r2 (cos ϕ2 + i sin ϕ2 )]...[rn (cos ϕn + i sin ϕn )]
= r1 r2 ...rn [cos(ϕ1 + ϕ2 + ... + ϕn ) + i sin(ϕ1 + ϕ2 + ... + ϕn )].
Hoàn toàn tương tự ta có thể làm phép chia các số phức
z1 r1 (cos ϕ1 + i sin ϕ1 )
=
z2 r2 (cos ϕ2 + i sin ϕ2 )
r1 (cos ϕ1 + i sin ϕ1 )(cos ϕ2 − i sin ϕ2 )
=
r2 (cos ϕ2 + i sin ϕ2 )(cos ϕ2 − i sin ϕ2 )
12 Chương 1. Khái niệm về số phức
r1
= [cos ϕ1 cos ϕ2 + sin ϕ1 sin ϕ2 + i(sin ϕ1 cos ϕ2 − cos ϕ1 sin ϕ2 )]
r2
r1
= [cos(ϕ1 − ϕ2 ) + i sin(ϕ1 − ϕ2 )]
r2

z1 |z1 |
Do đó, | |= và Z(z1 z2 )
z2 |z2 | y
z1
arg = arg z1 − arg z2 .
z2 Z2 (z2 )
Bây giờ, dễ dàng biểu diễn hình Z1 (z1 )
học tích của hai số phức:
Số phức z = z1 z2 với 0 E(1) x

z1 = r1 (cos ϕ1 + i sin ϕ1 ),
Hình 1.3.
z2 = r2 (cos ϕ2 +i sin ϕ2 ) là một
điểm với bán kính vectơ r1 r2 và
argumen ϕ1 + ϕ2 (Hình 1.3).

1.4. CÔNG THỨC MOA VRƠ


Cho một số phức bất kỳ dưới dạng lượng giác
z = r(cos ϕ + i sin ϕ), theo công thức nhân ở trên ta có

z n = (r(cos ϕ + i sin ϕ))n = rn (cos nϕ + i sin nϕ)

với n là một số nguyên bất kỳ. Công thức trên mang tên Moa-vrơ.
Công thức Moa-vrơ còn đúng với các số mũ nguyên âm. Thật vậy,
1
z −1 = = r−1 (cos ϕ − i sin ϕ)
r(cos ϕ + i sin ϕ)
= r−1 (cos(−ϕ) + i sin(−ϕ))
1.4. Công thức Moa vrơ 13

z −n = (z −1 )n = [r−1 (cos(−ϕ) + i sin(−ϕ))]n =


= r−n [cos(−nϕ) + i sin(−nϕ)].

Dựa vào công thức Moa-vrơ ta định nghĩa căn bậc n của số phức:
Cho z = r(cos ϕ + i sin ϕ). Căn bậc n của số phức z là một số phức
biểu diễn dưới dạng lượng giác z1 = ρ(cos θ+i sin θ), sao cho z1n = z,
hay là [ρ(cos θ + i sin θ)]n = r(cos ϕ + i sin ϕ).

Theo công thức Moa-vrơ, ta có ρn = r, suy ra ρ = n r (đây là
căn số học bậc n của số không âm). Còn argumen nθ và ϕ khác
nhau bởi số nguyên lần 2π, hay là nθ = ϕ + 2kπ, k là số nguyên.
ϕ + 2kπ
Vậy θ = .
n
Ngược lại, khi ta nâng bậc mũ n số


 
n
ϕ + 2kπ ϕ + 2kπ
z1 = r cos + i sin ,
n n

ở đây k là số nguyên bất kỳ, thì chính là z. Như vậy,


 
p
n n
ϕ + 2kπ ϕ + 2kπ
r(cos ϕ + i sin ϕ) = r cos + i sin
n n

với k = 0, 1, 2, ...., n − 1 sẽ nhận được n giá trị khác nhau cho n z.
√ 2π
Mỗi giá trị của n z tạo thành cấp số cộng với số dư và số đầu
n
ϕ
tiên là .
n
Do tính chu kỳ của hàm số sin x và cos x với k > n + 1 thì những

giá trị của n z lại lặp lại một trong n giá trị ban đầu.
14 Chương 1. Khái niệm về số phức

Do đó, căn bậc n của một số phức có Zn


Zn−1
đúng n giá trị khác nhau. Những số này biểu
diễn như đỉnh của n đa giác đều nằm trên
đường tròn với tâm là gốc tọa độ và bán

kính là n r. O

Ví dụ: Ta tìm nghiệm của một phương


trình đặc biệt dạng xn = 1 trong mặt phẳng
phức. Thật vậy, ký hiệu wk , k = 1, 2, ...n là Hình 1.4.
nghiệm của phương trình trên.

Vì 1 = cos 0 + i sin 0 và n 1 = 1 chúng ta sẽ có
 
2kπ 2kπ
wk = cos + i sin , k = 1, 2, ..., n.
n n
Rõ ràng wn = 1, còn những nghiệm khác nhận được bằng cách quay
2π 2π 2π
vectơ đơn vị đi , 2 , ..., (n − 1) .
 n n  n
2π 2π
Số wk = cos + i sin có tính chất đặc biệt là các nghiệm
n n
khác bằng chính w1 nâng lên lũy thừa số thứ tự nghiệm, vì
2π k
   
2kπ 2kπ 2π
wk = cos + i sin = cos + i sin .
n n n n

Nghiệm của xn = 1 là đỉnh của đa giác đều n đỉnh nằm trên


đường tròn đơn vị.
Chương 2
ĐỘ ĐO GÓC CỦA HAI TIA

2.1. Góc định hướng . . . . . . . . . . . . . . . . . . . . . . . . . . . . . . . . . . . . . . 15


2.2. Ví dụ . . . . . . . . . . . . . . . . . . . . . . . . . . . . . . . . . . . . . . . . . . . . . . . . . . . 18
2.3. Bài tập. . . . . . . . . . . . . . . . . . . . . . . . . . . . . . . . . . . . . . . . . . . . . . . . . 22

2.1. GÓC ĐỊNH HƯỚNG


Như ta đã thấy mỗi điểm trong hệ tọa độ vuông góc tương ứng
với một số phức. Quan hệ tập hợp các số phức và tập hợp các điểm
trong mặt phẳng là tương ứng một-một. Điểm Z với tọa độ (a, b)
ứng với số phức z = a + ib. Số phức z gọi là nhãn của điểm Z. Kể
từ đây một điểm trong mặt phẳng luôn luôn ký hiệu là một chữ cái
hoa và nhãn của nó là chữ cái thường tương ứng.
Trong mặt phẳng hệ tọa độ
vuông góc xOy, mỗi điểm Z với
nhãn z chúng ta đặt một vectơ
−→
OZ. Do đó số phức có thể biểu
diễn hình học như là những vectơ
trong mặt phẳng.

Sự biểu diễn số phức qua


vectơ hoàn toàn thích hợp khi ta Hình 2.1.
xem xét nguyên lý cộng và trừ
16 Chương 2. Độ đo góc của hai tia

các vectơ tương ứng với cộng và trừ các số phức (xem Hình 2.1).

Nếu Z1 và Z2 là hai điểm trên mặt phẳng với nhãn z1 và


z2 , khi đó tổng của chúng z3 = z1 + z2 biểu diễn bởi Z3 , mà
−−→ −−→ −−→ −−→ −−→
OZ3 = OZ1 +OZ2 . Còn hiệu z2 −z1 là vectơ OZ2 −OZ1 . Khoảng cách
−−−→
d của điểm Z1 đến Z2 hoặc độ dài Z1 Z2 là d = |Z1 Z2 | = |z1 − z2 |,
vậy d là môđun của số phức z2 − z1 .
Từ nguyên tắc cộng vectơ suy ra rằng nếu Z là trung điểm của
−→ 1 −−→ 1 −−→ −−→
Z1 Z2 , thì OZ = OZ3 = (OZ1 + OZ2 ), hoặc nhãn của Z biểu diễn
2 2
1
qua z1 , z2 là z = (z1 + z2 ).
2
Để tính góc định hướng α tạo
bởi hai tia đi qua điểm gốc của
tọa độ O, ta chọn z1 và z2 nằm
trên mỗi tia. Khi đó
z2
α = arg z2 − arg z1 = arg
z1
Trong trường hợp hai tia xuất
phát từ điểm Z00 , ta cũng làm
tương tự và có Hình 2.2.

z20 − z00
α = arg(z20 − z00 ) − arg(z10 − z00 ) = arg
z1 − z00
Một cách tổng quát, biểu diễn độ đo góc theo hướng dương của
hai vectơ bất kỳ theo nhãn của các số phức thì sao ? Cho hai vectơ
−−−→ −−−→
Z1 Z2 và U1 U2 với nhãn tại các điểm tương ứng z1 , z2 , u1 , u2 . Ta cần
−−−→
phải quay vectơ đơn vị của Z1 Z2 đi một góc φ theo chiều dương
nghĩa là
z2 − z 1 u2 − u1
(cos φ + i sin φ) =
|z2 − z1 | |u2 − u1 |
2.1. Góc định hướng 17

từ đó
u2 − u1 z2 − z1
cos φ + i sin φ = :
|u2 − u1 | |z2 − z1 |
u2 − u1 |u2 − u1 |
= : =p
z2 − z1 |z2 − z1 |

Vậy góc phải tìm cos φ = p+p p−p


2 , sin φ = 2i từ đó có

(z2 − z1 )(u2 − u1 ) + (u2 − u1 )(z 2 − z 1 )
cos φ =


2|z2 − z1 ||u2 − u1 |
(2.1)
−(z 2 − z 1 )(u2 − u1 ) + (u2 − u1 )(z 2 − z 1 )
sin φ =


2|z2 − z1 ||u2 − u1 |
−−−→ −−−→
Từ những đẳng thức trên suy ra vectơ Z1 Z2 , U1 U2 vuông góc với
nhau khi và chỉ khi

(z2 − z1 )(u2 − u1 ) + (u2 − u1 )(z 2 − z 1 ) = 0 (2.2)

và chúng song song với nhau khi và chỉ khi

(z2 − z1 )(u2 − u1 ) = (u2 − u1 )(z 2 − z 1 ) (2.3)

Nhận xét:
1. Do công thức (1), nếu z1 trùng với u1 và |z1 z2 | = |u1 u2 |, thì
khi biết nhãn z2 và góc φ với các giá trị đặc biệt thì u2 tính được
nhãn theo z2 như sau:
a) φ = 90◦ , thì u2 = iz2 .
√ !
1 3
b) φ = 60◦ , thì u2 = + i z2 .
2 2
√ !
3 1
c) φ = 30◦ , thì u2 = + i z2 .
2 2
18 Chương 2. Độ đo góc của hai tia

Các nhận xét trên rất có ích khi giải các bài toán hình học bằng
phương pháp số phức.
z2 − z0
2. Ký hiệu V (z2 , z1 , z0 ) = gọi là tỷ số đơn của các số
z1 − z0
phức z2 , z1 , z0 (viết theo thứ tự đã chỉ ra). Do đó, argumen của
−−−→ −−−→
V (z2 , z1 , z0 ) chính là góc định hướng giữa các vectơ Z0 Z1 và Z0 Z2 .
Điều kiện cần và đủ để 3 điểm z0 , z1 , z3 thẳng hàng là góc định
−−−→ −−−→
hướng giữa hai vectơ Z0 Z1 và Z0 Z2 bằng 0 hoặc ±π. Nghĩa là tỷ số
đơn V (z2 , z1 , z0 ) là một số thực.

2.2. VÍ DỤ
Ví dụ 2.1. Cho hình vuông ABCD. Điểm M là trung điểm của CD,
điểm P nằm trên đường chéo AC sao cho |P C| = 3|AP |. Chứng minh
rằng BP
\ M = 90◦ .

Lời giải. Lấy hệ tọa độ vuông góc


−−→
sao cho A là điểm gốc và vectơ AB
là vectơ đơn vị theo chiều dương
của trục hoành. Như vậy, nhãn của
những điểm A, B, C, D tương ứng là
a = 0, b = 1, c = 1 + i, d = i. Điểm M
1 1
có nhãn m = (c + d) = (1 + 2i).
2 2 Hình 2.3.
−→ 1 −→ 1 1
Vì AP = AC, nên p = c = (1 + i). Ta tính
4 4 4
1 1
m−p (1 + 2i) − (1 + i) 1 + 3i
V (m, b, p) = = 2 4 =
b−p 1 3−i
1 − (1 + i)
4
(1 + 3i)(3 + i) 10i
= = = i.
(3 − i)(3 + i) 10
2.2. Ví dụ 19
π
Bởi vì arg i = , do đó BP
\ M = 90◦ .
2
Hơn nữa, |i| = 1, chúng ta nhận được |BP | = |P M |, nên tam
giác BP M là vuông cân. J
Ví dụ 2.2. Cho ba hình vuông bằng nhau ABCD, BEF C, EP QF
(hình 2.4). Chứng minh rằng ACD
\ + AF\ \ = π.
D + AQD
2

Hình 2.4
−−→
Lời giải. Đưa vào hệ tọa độ vuông góc với A là điểm gốc và AB
là vectơ đơn vị theo chiều dương của trục hoành. Suy ra, nhãn của
A, B, C, D, E, F, P, Q lần lượt là a = 0, c = 1 + i, d = i, e = 2,
f = 2 + i, p = 3, q = 3 + i.
Vì ACD
\ = BAC
\ = arg(1 + i), AF
\ D = EAF
[ = arg(2 + i) và
AQD
\ = P[ AQ = arg(3 + i), thì
ACD
\ + AF
\ D + AQD
\ = arg(1 + i) + arg(2 + i) + arg(3 + i)
π
= arg(10i) = .
2
J
Ví dụ 2.3. Cho tam giác ABC. Trong nửa mặt phẳng bờ AB chứa
điểm C, dựng hình vuông ABDE.
Trong nửa mặt phẳng bờ BC chứa điểm A, dựng hình vuông BCF G.
Chứng minh GA vuông góc với CD và GA = CD.
20 Chương 2. Độ đo góc của hai tia

Lời giải. Lấy hệ tọa độ vuông G F


−−→
góc có gốc tại B, vectơ BC là
chiều dương của trục hoành. Ký
hiệu nhãn của các đỉnh của tam
A
giác ABC tương ứng là a, b, c.
Khi đó dễ dàng tính ra được
g = ic, d = −ia. Góc giữa GA
B E
và CD ký hiệu là ϕ thì C
−ia − c −ia − c
ϕ = arg : | | =
a − ic ia − c
π D
arg i =
2 Hình 2.5.
Vậy, GA vuông góc với CD và
|GA| = |a − ic| = | − ia − c| = |CD|. J
Ví dụ 2.4. Cho tam giác ABC F F
\ 6= 60◦ ), ở miền ngoài của
(BAC
E
E
tam giác vẽ các tam giác đều ABD D
D
và ACE. Dựng hình bình hành A
A
AEF D. Chứng minh tam giác
BF C là đều.
B C
Lời giải. Lấy hệ tọa độ vuông góc B C
có gốc tại A, nhãn của các đỉnh Hình 2.6.
của tam giác ABC lần lượt là a =
0, b, c. Do cách dựng tam giác đều
ta có
2.2. Ví dụ 21

1 3
e= c(cos 60◦+ i sin 60◦ )= c( + i )
2 √2
1 3
d = b(cos 60◦ − i sin 60◦ ) = b( − i )
√ 2 2
1 3
f = d + e = (b + c) + i(c − b)
2 2

1 3
|BF | = | (b + c) + i(c − b) − b| = |c − b| = |BC|
2 2

1 3
|CF | = | (b + c) + i(c − b) − c| = |c − b| = |BC|
2
do đó tam giác BF C đều.
2
J
Ví dụ 2.5. Nếu AB và CD là hai đoạn thẳng cắt nhau và P, Q
là những trung điểm tương ứng của các đoạn thẳng trên. Chứng
minh rằng, nếu AB là phân giác của góc CP D và P A2 = P B 2 =
|P C|.|P D|, thì CD là phân giác của góc AQB và QC 2 = QD2 =
|QA||QB|.

Lời giải. Từ sự bằng nhau của các góc BP D và CP B ta có thể viết


p−d p−d p−b p−b
:| |= :| |,
p−b p−b p−c p−c
từ đó
(p − d)(p − c) |p − d||p − c|
2
= =1
(p − b) |p − b|2
Suy ra
p2 − (c + d)p + cd = p2 − 2bp + b2
hoặc là ab + cd = 2pq.
Trong đẳng thức cuối cùng vai trò a, b và p tương ứng với c, d và
q là như nhau, nên điều kiện cần và đủ đeer AB là phân giác của
góc CP D và P A2 = P B 2 = |P C||P D|, cũng là điều kiện cần và đủ
đeer CD là phân giác của góc AQB và QC 2 = QD2 = |QA||QB|.
J
22 Chương 2. Độ đo góc của hai tia

2.3. BÀI TẬP


. 2.6. Cho hình vuông ABCD. Điểm M và N nằm tương ứng trên
2
các đường chéo BD và cạnh BC sao cho BM = BD và BN =
3
1 ◦
BC. Chứng minh rằng AM N = 90 .
\
3
. 2.7. Cho ngũ giác ABCDE. Nối các điểm trung bình của cạnh
AB và CD, và các điểm trung bình của cạnh BC và DE. Lấy H
và K lần lượt là các điểm trung bình của các đoạn nối trên. Chứng
1
minh rằng HK song song với AE và |HK| = |AE|.
4
. 2.8. Cho tam giác ABC, về phía ngoài của tam giác vẽ các hình
vuông ABEF và ACGH. Chứng minh rằng đường trung tuyến AM
vẽ từ đỉnh A của tam giác ABC là đường cao của tam giác AHF .
. 2.9. Về phía ngoài của tam giác ABC vẽ các hình vuông ABDE và
ACF G. Gọi H, K, L lần lượt là trung điểm của đoạn BE, BC, CG.
a) Chứng minh rằng tam giác HKL vuông cân.
b) Có nhận xét gì về vị trí thứ 4 của hình vuông có 3 đỉnh
H, K, L?.
. 2.10. Cho tam giác ABC, M là trung điểm của cạnh BC. Trên
cạnh AB lấy điểm D sao cho BD = 2AD. Các đoạn thẳng AM và
CD cắt nhau tại điểm I. Chứng minh rằng
a) I là trung điểm của đoạn AM .
b) CI = 3DI.
. 2.11. Về phía ngoài của tam giác ABC vẽ các tam giác cân
M AB, N AC và P CB theo thứ tự nhận các điểm M, N, P làm đỉnh
góc vuông. Chứng minh rằng các đoạn thẳng AP và M N bằng nhau
và vuông góc với nhau.
2.3. Bài tập 23

. 2.12. Cho tứ giác lồi ABCD. Gọi M và N theo thứ tự là trung


điểm của các cạnh AB và CD, E và F là giao lần lượt của các
đường thẳng AD và BC với đường thẳng M N . Chứng minh rằng
nếu AD = BC thì AEM
\ = BF \ M.

. 2.13. Về phía ngoài của một tứ giác ABCD trên các cạnh AB, BC,
CD, DA dựng các hình vuông, tâm lần lượt của các hình vuông này
là E, F, G, H. Chứng minh rằng EG = F H và EG⊥F H.

. 2.14. Chứng minh rằng đoạn thẳng nối trung điểm của các đường
chéo trong một tứ giác và những đoạn thẳng nối trung điểm của các
cạnh đối diện của tứ giác đó có một điểm chung.

. 2.15. Cho điểm M và N là trung điểm của các cạnh AB và BC


trên hình vuông ABCD. Đoạn thẳng CM và DN cắt nhau tại P .
Chứng minh rằng đoạn AP bằng cạnh hình vuông.
Chương 3
PHƯƠNG TRÌNH ĐƯỜNG THẲNG

3.1. Đường thẳng qua hai điểm . . . . . . . . . . . . . . . . . . . . . . . . . . 24


3.2. Phương trình tham số . . . . . . . . . . . . . . . . . . . . . . . . . . . . . . . . . 25
3.3. Ví dụ . . . . . . . . . . . . . . . . . . . . . . . . . . . . . . . . . . . . . . . . . . . . . . . . . . . 26
3.4. Bài tập. . . . . . . . . . . . . . . . . . . . . . . . . . . . . . . . . . . . . . . . . . . . . . . . . 32

3.1. ĐƯỜNG THẲNG QUA HAI ĐIỂM


Trong phần trước ta thấy điều kiện cần và đủ để 3 điểm khác
−−−→
nhau z0 , z1 , z2 nằm trên một đường thẳng là góc giữa hai vectơ Z1 Z2
−−−→
và Z0 Z2 bằng 0 hoặc ±π. Nói một cách khác tỷ số đơn V (z0 , z1 , z2 )
là một số thực. Do tính chất của số phức ta có thể biểu diễn dưới
dạng như sau:
z 0 − z2 z0 − z2
= .
z 1 − z2 z1 − z2
Từ đẳng thức trên ta thấy ngay, một đường thẳng đi qua hai điểm z1
và z2 là tập hợp các điểm Z sao cho
z − z2 z − z2
=
z1 − z2 z1 − z2
hoặc là
(z 1 − z 2 )z − (z1 − z2 )z + (z1 z 2 − z 1 z2 ) = 0
Vì nhãn của tất cả các điểm trên đường thẳng thoả mãn chỉ đẳng
thức trên, nên ta có thể gọi đó là phương trình đường thẳng.
3.2. Phương trình tham số 25

Nếu đặt B = z 1 − z 2 và C = z1 z 2 − z 1 z2 , thì


Bz − Bz + C = 0, B 6= 0.
vì C = z1 z 2 − z 1 z2 = −C, do vậy C hoàn toàn là số ảo. Ngược lại,
mọi phương trình có dạng trên biểu diễn một đường thẳng trên mặt
phẳng.

3.2. PHƯƠNG TRÌNH THAM SỐ


Ba điểm Z, Z1 , Z2 nằm trên một đường thẳng khi và chỉ khi tỷ
z − z2
số đơn V (z, z1 , z2 ) = là một số thực. Do đó với mỗi số thực
z1 − z2
λ, thì số phức z = z2 + λ(z1 − z2 ) = λz1 + (1 − λ)z2 là một nhãn của
một điểm trên đường thẳng đi qua Z1 Z2 và ngược lại. Như vậy, khi
λ chạy trên tập hợp số thực thì phương trình
z = z2 + λ(z1 − z2 ) = λz1 + (1 − λ)z2
gọi là phương trình tham số của đường thẳng đi qua Z1 Z2 .
Trong ứng dụng giải những bài tập hình học ta cần xem xét λ
khi biến đổi thì ảnh hưởng của z0 thế nào đối với z1 , z2 ?
z0 − z2 −−−→ −−−→
- Nếu số λ = là số thực dương, thì vectơ Z0 Z1 và Z1 Z2
z1 − z2
cùng chiều.
−−−→ −−−→
- Nếu số λ là âm, thì vectơ Z0 Z1 và Z1 Z2 ngược chiều nhau.
Đối với vị trí của điểm Z0 xác định như sau:
- Nếu 0 < λ < 1, thì Z0 nằm trong đoạn Z1 Z2 .
- Nếu λ > 1, thì Z0 nằm ngoài đoạn Z1 Z2 về phía Z1 .
- Nếu λ < 0, thì Z0 nằm ngoài đoạn Z1 Z2 về phía Z2 .
Giá trị tuyệt đối của λ bằng tỷ số đoạn thẳng |Z0 Z2 | và |Z1 Z2 |.
Trong thực tế, ta thường tìm trên đường thẳng Z1 Z2 một điểm Z sao
26 Chương 3. Phương trình đường thẳng

Z1 Z
cho = λ, đã cho trước (ở đây Z1 Z là độ dài đại số của Z1 Z).
ZZ2
z − z1 z1 + λz2 1 λ
Khi đó = λ, từ đó suy ra z = = z1 + z2
z2 − z 1+λ 1+λ 1+λ
. Nghĩa là một điểm Z nằm trên đuwờng nối Z1 Z2 có dạng trên với
λ là một số thực nào đó.

3.3. VÍ DỤ
Ví dụ 3.1. Cho hai hình vuông
C B
cùng hướng OABC và OA1 B1 C1 C B
có một điểm chung O. Chứng
minh rằng các đường thẳng
O A
AA1 , BB1 và CC1 đi qua một O A
điểm.
Lời giải. Chọn hệ toạ độ vuông E
C1
góc có gốc tại O. Khi đó
A1
c = ia, b = a + c = a(1 + i), A1
c1 = ia1 , b1 = a1 + c1 = a1 (1 + i).
Những đường thẳng AA1 , BB1
B1
và CC1 có phương trình tương B1
ứng Hình 3.1.
(a − a1 )z − (a − a1 )z + aa1 − aa1 = 0,
(b − b1 )z − (b − b1 )z + bb1 − bb1 = 0,
(c − c1 )z − (c − c1 )z + cc1 − cc1 = 0.
Nhưng chú ý rằng

cc1 − cc1 = (ai)(−a1 ) − (−ai)(a1 i) = aa1 − aa1

và bb1 − bb1 = a(1 + i)a1 (1 − i) − a(1 − i)a1 (1 + i) = 2(aa1 − aa1 ).


3.3. Ví dụ 27

Từ đó suy ra nếu cộng đẳng thức 1 và 3 thì nhận được 2.


Điều này chỉ ra rằng điểm chung của AA1 và CC1 nằm trên
đường thẳng BB1 . J
Ví dụ 3.2. Trên các cạnh AB và AC của tam giác đều ABC lấy các
|AD| |BE| 1
điểm E và D tương ứng sao cho = = . Chứng minh
|DC| |EA| 2
rằng, nếu P là giao điểm của BD và CE, thì AP
[ C = 900 .

Lời giải. Chọn hệ toạ độ vuông góc


với gốc tại A và chiều dương của
−−→
trục hoành là AB và b = 1. Khi đó
\ = 600 , suy
từ |AC| = |AB| và BAC

0 0 1 3
ra c = cos 60 +i sin 60 = +i .
2 2
|AE| 2
Từ điều kiện = 2, thì e = ,
|BE| 3 Hình 3.2.
1
tương tự d = c.
3
Những phương trình của đường
thẳng BD và CE tương ứng là
BD :(b − d)z − (b − d)z + bd − bd = 0,
CE :(c − e)z − (c − e)z + ce − ce = 0.
Thay những giá trị đã biết ở trên vào các đẳng thức, ta có
 √ ! √ ! √
 5 3 5 3 3
 6 +i 6 z− 6 −i 6 z−i 3 =0



√ ! √ ! √
 1 3 1 3 3
 − 6 + i 2 z − 6 − i 2 z − i2 3 =0


Giao điểm P có nhãn p là nghiệm của hệ phương trình trên. Giải hệ


28 Chương 3. Phương trình đường thẳng

9 3
phương trình, ta nhận được z = p = +i . Khi đó
14 14
√ √
1 3 9 3 √
c−p +i − −i 2 − i6 3
= 2 2 14
√ 14 = √ =
p 3 3 9+i 3
+i
14
√ 14 √ √
(2 − i6 3)(9 − i 3) i56 3 2 √
= √ √ = = i 3.
(9 + i 3)(9 − i 3) 84 3

Như vậy, số
c−p
p
là số hoàn toàn ảo. Suy ra AP
[ π
C= .
2
J

Ví dụ 3.3. Cho tam giác A1 A2 A3 . Trên cạnh A2 A3 và A3 A1 lấy các


A2 B 1 A3 B 2
điểm B1 và B2 sao cho = λ1 và = λ2 .
B 1 A3 B2 A1
Chứng minh rằng, nếu M là giao điểm của A1 B1 và A2 B2 thì

a2 + λ1 a3 + λ1 λ2 a1
m= .
1 + λ1 + λ1 λ2

Lời giải. Từ tỷ số đã cho, ta có A3


a2 + λ 1 a3
b1 = và
1 + λ1
a3 + λ 2 a1
b2 = .
1 + λ2 B2 B1
M
Khi đó, nếu đặt
a2 + λ1 a3 + λ1 λ2 a1
z = , thì A1
1 + λ1 + λ1 λ2 A2
chúng ta sẽ nhận được
a2 + λ 1 a3 + λ 1 λ 2 a1 Hình 3.3.
z= =
1 + λ1 + λ1 λ2
3.3. Ví dụ 29

λ1 λ2 1 + λ1 a2 + λ1 a3
= a1 + .
1 + λ1 + λ1 λ2 1 + λ1 + λ1 λ2 1 + λ1
λ1 λ2 1 + λ1
= a1 + b1
1 + λ1 + λ1 λ2 1 + λ1 + λ1 λ2
λ1 λ2 1 + λ1
nhưng + = 1, điều này có nghĩa là Z
1 + λ1 + λ1 λ2 1 + λ1 + λ1 λ2
với nhãn z nằm trong đoạn A1 B1 . Tương tự ta cũng có cách biểu
diễn
λ1 λ2 1 + λ1
z= a2 + b2
1 + λ1 + λ1 λ2 1 + λ1 + λ1 λ2
Suy ra Z cũng nằm trên đường thẳng A2 B2 . Vậy Z trùng với giao
điểm M của A1 B1 và A2 B2 . J
Chú ý: Từ cách chứng minh trên ta có thể tính
a2 + λ1 a3 + λ1 λ2 a1
− a1
A1 M m − a1 1 + λ1 + λ1 λ2
= =
M B1 b1 − m a2 + λ 1 a3 a2 + λ 1 a3 + λ 1 λ 2 a1

1 + λ1 1 + λ1 + λ1 λ2
(1 + λ1 )[(a2 − a1 ) + λ1 (a3 − a1 )] 1 + λ1
= =
λ1 λ2 [(a2 − a1 ) + λ1 (a3 − a1 )] λ1 λ2
A2 M
Tương tự ta có = λ1 (1 + λ2 ).
M B2
Trường hợp đặc biệt, λ1 = λ2 = 1, khi đó A1 B1 và A2 B2 là
đường trung tuyến của tam giác A1 A2 A3 và điểm M là trọng tâm
1
của tam giác A1 A2 A3 với nhãn m = (a1 + a2 + a3 ) và
3
A1 M A2 M A3 M 2
= = =
M B1 M B2 M B3 1
Tương tự ta có thể tính được những điểm đặc biệt khác trong
tam giác như tâm đường tròn ngoại tiếp, trực tâm,...

Ví dụ 3.4. [Định lý Cheva] Cho ba điểm B1 , B2 , B3 nằm trên các cạnh


30 Chương 3. Phương trình đường thẳng

tương ứng A2 A3 , A3 A1 , A1 A2 của tam giác A1 A2 A3 và


A2 B1 A3 B2 A1 B 3
= λ1 , = λ2 , = λ3 .
B1 A3 B 2 A1 B 3 A2
Chứng minh rằng, những đường thẳng A1 B1 , A2 B2 , A3 B3 cắt nhau
tại một điểm khi và chỉ khi λ1 λ2 λ3 = 1.

Lời giải. 1) Nếu λ1 λ2 λ3 = 1. Khi đó, với điểm B3 ta có


a1 + λ 3 a2 a2 + λ1 λ2 a1
b3 = = .
1 + λ3 1 + λ1 λ2
Ký hiệu M là giao điểm của A1 B1 và A2 B2 . Theo ví dụ trước ta có
a2 + λ1 a3 + λ1 λ2 a1
m= .
1 + λ1 + λ1 λ2
Ta cần kiểm tra xem M có nằm trên đường thẳng A3 B3 không ? Để
m − a3
khẳng định điều đó ta chứng minh là một số thực. Thật vậy,
b3 − m
a2 + λ1 a3 + λ1 λ2 a1
− a3
m − a3 1 + λ1 + λ1 λ2
=
b3 − m a2 + λ1 λ2 a1 a2 + λ1 a3 + λ1 λ2 a1

1 + λ1 λ2 1 + λ1 + λ1 λ2
(1 + λ1 λ2 )[(a2 − a3 ) + λ1 λ2 (a1 − a3 )] 1 + λ1 λ2
= = .
λ1 [(a2 − a3 ) + λ1 λ2 (a1 − a3 )] λ1
Suy ra điểm M nằm trên đường thẳng A3 B3 .
Do đó, A1 B1 , A2 B2 , A3 B3 cắt nhau hại điểm M .
2) Ngược lại, nếu A1 B1 , A2 B2 và A3 B3 cắt nhau tại một điểm
M . Theo cách làm của ví dụ trước ta tính được
a2 + λ1 a3 + λ1 λ2 a1
m=
1 + λ1 + λ1 λ2
vì M là giao điểm của A1 B1 và A2 B2 . Mặt khác, M còn là giao điểm
của những đường thẳng A2 B2 và A3 B3 , ta có
a3 + λ2 a1 + λ2 λ3 a2
m= .
1 + λ2 + λ1 λ3
3.3. Ví dụ 31

Sau khi so sánh giữa hai đẳng thức trên và nhóm lại, ta có

(1 − λ1 λ2 λ3 )[(a2 − a3 ) + λ2 (a2 − a1 )] = 0

Nhưng số (a2 − a3 ) + λ2 (a2 − a1 ) 6= 0, vì ba điểm A1 , A2 , A3 không


nằm trên một đường thẳng. Từ đó suy ra λ1 λ2 λ3 = 1. J
Ví dụ 3.5. [Định lý Menelai] Cho ba điểm B1 , B2 , B3 nằm trên các
cạnh tương ứng A2 A3 , A3 A1 , A1 A2 của tam giác A1 A2 A3 và
A2 B1 A3 B2 A1 B 3
= λ1 , = λ2 , = λ3 .
B1 A3 B 2 A1 B 3 A2
Chứng minh rằng, ba điểm B1 , B2 , B3 nằm trên cùng đường thẳng
khi và chỉ khi λ1 λ2 λ3 = −1.

Lời giải. 1) Nếu λ1 λ2 λ3 = −1. Xét tỷ số


a2 + λ1 a3 a1 + λ3 a2

b1 − b3 1 + λ1 1 + λ3
=
b3 − b2 a1 + λ3 a2 a3 + λ2 a1

1 + λ3 1 + λ2
1 + λ2 a2 − a1 + λ1 (a3 − a1 ) + λ1 λ3 (a3 − a2 )
= .
1 + λ1 a1 − a3 + λ3 (a2 − a3 ) + λ2 λ3 (a2 − a1 )
1 + λ2
=
(1 + λ1 )λ2 λ3
Đây là một số thực. Suy ra những điểm B1 , B2 , B3 nằm trên cùng
một đường thẳng.
2) Nếu những điểm B1 , B2 , B3 nằm trên một đường thẳng. Ta
chọn được một số λ03 sao cho λ1 λ2 λ03 = −1. Theo chứng minh ở ví
a1 + λ03 a2
dụ trước thì điểm B30 , có nhãn b03 = nằm trên B1 B2 . Như
1 + λ03
vậy, B30 trùng với giao điểm của đường thẳng A1 A2 và B1 B2 , đó là
B3 . Suy ra λ03 = λ3 , vậy λ1 λ2 λ3 = −1. J
32 Chương 3. Phương trình đường thẳng

3.4. BÀI TẬP


. 3.6. (Định lý Shainer) Chứng minh rằng giao điểm của hai đường
chéo một hình thang, giao điểm hai cạnh bên và những trung điểm
của hai cạnh đáy, nằm trên cùng một đường thẳng.

. 3.7. Điểm D chia cạnh AC của tam giác ABC với tỷ số |AD| :
|DC| = 1 : 4. Đoạn thẳng BD đã chia trung tuyến AE của tam giác
ABC với tỷ số nào ?

. 3.8. Cho hình thang ABCD, đáy lớn AB. M là trung điểm cạnh
BC và P là giao điểm của đường thẳng AM và đường thẳng song
song với BC xuất phát từ đỉnh D. Chứng minh rằng, nếu |AP | =
|P M | thì những đường thẳng AC, DP và M N cắt nhau tại một
điểm, ở đây lấy N là trung điểm của cạnh AD.

. 3.9. Qua điểm M nằm trong hình bình hành ABCD ta kẻ các
đường thẳng song song với các cạnh của chúng. Chúng cắt các cạnh
AB, BC, CD, DA lần lượt tại các điểm P, Q, R, S. Chứng minh nếu
đường thẳng P Q và RS cắt nhau, thì giao điểm của chúng nằm trên
AC.

. 3.10. Về phía ngoài của tam giác ABC vẽ các hình vuông ABEF
và ACGH, N là trung điểm của F H, M là giao điểm của các đường
BG và CE. Chứng minh rằng các điểm N, M, A thẳng hàng.

. 3.11. Cho hình thang ABCD đáy lớn là AB. Kẻ đường thẳng song
song với hai đáy, lần lượt cắt các cạnh bên BC, AD tại N và M , cắt
các dường chéo AC, BD tại P và Q. Chứng minh rằng M D = QN .

. 3.12. Trên cạnh AB và DC của tứ giác ABCD lấy điểm M và N


AM DN
sao cho = = k. Các điểm P, Q, S lần lượt là trung điểm
AB DC
3.4. Bài tập 33

của các đoạn thẳng AD, BC và M N . Chứng minh rằng P, Q, S cùng


PS
nằm trên một đường thẳng và = k.
PQ

. 3.13. Cho tam giác ABC, lấy K là trung điểm của AB, từ K kẻ
hai đuw ờng song song lần lượt với trung tuyến AA1 và BB1 của
tam giác ABC và cắt cạnh AC tại M , BC tại N . Chứng minh rằng
các đường trung tuyến AA1 , BB1 chia đường thẳng M N thành ba
phần bằng nhau.

. 3.14. Cho A1 , B1 , C1 là trung điểm các cạnh BC, CA và AB của


tam giác ABC, M là một điểm trong tam giác ABC, lấy M1 , M2 , M3
lần lượt là các điểm đối xứng của M qua các điểm A1 , B1 , C1 . Chứng
minh rằng những đường thẳng AM1 , BM2 , CM3 cắt nhau tại một
điểm.

. 3.15. Cho tam giác A1 A2 A3 có độ dài các cạnh |A2 A3 | =


d1 , |A3 A1 | = d2 , |A1 A2 | = d3 . Hãy biểu diễn nhãn của tâm đường
tròn nội tiếp tam giác theo số phức tạo bởi d1 , d2 , d3 và nhãn của các
đỉnh a1 , a2 , a3 .
Chương 4
PHƯƠNG TRÌNH ĐƯỜNG TRÒN

4.1. Phương trình tổng quát . . . . . . . . . . . . . . . . . . . . . . . . . . . . . . . 34


4.2. Đường tròn đơn vị . . . . . . . . . . . . . . . . . . . . . . . . . . . . . . . . . . . . 37
4.3. Giao điểm hai cát tuyến. . . . . . . . . . . . . . . . . . . . . . . . . . . . . . 38
4.4. Giao điểm hai tiếp tuyến . . . . . . . . . . . . . . . . . . . . . . . . . . . . . 40
4.5. Chân đường vuông góc ở dây cung . . . . . . . . . . . . . . . . 42
4.6. Bài tập. . . . . . . . . . . . . . . . . . . . . . . . . . . . . . . . . . . . . . . . . . . . . . . . . 44

4.1. PHƯƠNG TRÌNH TỔNG QUÁT


Chúng ta sẽ tìm điều kiện cần và đủ để 4 điểm Z0 , Z1 , Z2 , Z3
nằm trên một đường tròn. Ở đây ta có thể coi đường thẳng như là
đường tròn tâm vô tận (có giải thích sau).
Nếu Z0 , Z1 , Z2 , Z3 nằm trên đường
tròn (hình 4.1), thì hiệu giữa góc định
hướng Z\ 0 Z2 Z1 và Z0 Z3 Z1 là 0 hoặc ±π.
\
Suy ra tỷ số
V (z0 , z1 , z2 ) z0 − z2 z 0 − z3
= :
V (z0 , z1 , z3 ) z1 − z2 z 1 − z3
là một số thực. Hình 4.1.
V (z0 , z1 , z2 )
Ngược lại, nếu tỷ số là một số thực, thì z0 , z1 , z2 , z3
V (z0 , z1 , z3 )
là nhãn của những điểm trên đường tròn hoặc đường thẳng.
4.1. Phương trình tổng quát 35

V (z0 , z1 , z2 )
W (z0 , z1 , z2 , z3 ) = gọi là tỷ số kép của 4 điểm
V (z0 , z1 , z3 )
z0 , z1 , z2 , z3 (theo thứ tự này). Như vậy
Điều kiện cần và đủ cho 4 điểm Z0 , Z1 , Z2 , Z3 nằm trên đường
thẳng hoặc đường tròn là tỷ số kép của nhãn z0 , z1 , z2 , z3
V (z0 , z1 , z2 ) z0 − z2 z0 − z3
W (z0 , z1 , z2 , z3 ) = = :
V (z0 , z1 , z3 ) z1 − z2 z1 − z3
là một số thực, hoặc là
z 0 − z2 z0 − z 3 z0 − z2 z0 − z3
: = :
z 1 − z2 z1 − z 3 z1 − z2 z1 − z3
Từ phương trình trên, để một điểm Z nằm trên đường tròn ngoại
tiếp tam giác Z1 Z2 Z3 là phương trình sau thỏa mãn
z − z2 z − z3 z − z2 z − z3
: = :
z 1 − z2 z1 − z 3 z1 − z2 z1 − z3
Ta có thể gọi đây là phương trình đường tròn xác định bởi 3
điểm Z1 , Z2 , Z3 . Giải phóng mẫu số ta nhận được
(z −z2 )(z −z 3 )(z1 −z3 )(z 1 −z 2 )−(z −z3 )(z −z 2 )(z1 −z2 )(z 1 −z 3 ) = 0
hoặc là αzz + βz − βz + γ = 0, ở đây ta đặt
α = (z1 − z3 )(z 1 − z 2 ) − (z1 − z2 )(z 1 − z 3 ),
β = −z 3 (z1 − z3 )(z 1 − z 2 ) + z 2 (z1 − z2 )(z 1 − z 3 ),
γ = z2 z 3 (z1 − z3 )(z 1 − z 2 ) − z3 z 2 (z1 − z2 )(z 1 − z 3 ).

Rõ ràng, số α và γ là hoàn toàn ảo (α = −α và γ = −γ). Như


vậy, mọi đường tròn có phương trình dạng

αzz + βz − βz + γ = 0

ở đây α, γ là số hoàn toàn ảo. So với phương trình đường thẳng mục
trước, thì phương trình đường tròn là phương trình đuw ờng thẳng
36 Chương 4. Phương trình đường tròn

khi và chỉ khi α = 0. Điều này cũng phù hợp với điều kiện để ba
điểm Z1 , Z2 và Z3 thẳng hàng.
β
Bây giờ, nếu α 6= 0 chia phương trình trên cho α và đặt a = −
α
γ
và b = thì phương trình đường tròn có dạng
α
zz − az − az + b = 0.
Từ đó suy ra
|z − a|2 = |a|2 − b.
Do đó a là nhãn của điểm tâm đường tròn và bán kính là
p
R = |a2 | − b.
Trường hợp đặc biệt, tâm của đường tròn trùng với điểm gốc toạ
độ và bán kính là 1, thì phương trình đường tròn có dạng zz = 1 .
Đường tròn loại này gọi là đường tròn đơn vị.

Ví dụ 4.1. Trong mặt phẳng cho 4


đường tròn k1 , k2 , k3 , k4 . Cho X1 , Y1 là
giao điểm chung của k1 , k2 , X2 , Y2 – của
k2 , k3 , X3 , Y3 – của k3 , k4 , X4 , Y4 – của
k4 , k1 .
Chứng minh rằng nếu X1 , X2 , X3 , X4
nằm trên một đường tròn hoặc đường
thẳng , thì Y1 , Y2 , Y3 , Y4 cũng nằm trên
một đường tròn hoặc đường thẳng. Hình 4.2.

Lời giải. Do X1 , Y2 , X2 , Y1 nằm trên đường tròn k2 , thì tỷ số kép của


chúng
x 1 − x 2 x 1 − y1
W (x1 , y2 , x2 , y1 ) = :
y2 − x2 y2 − y1
là một số thực. Tương tự như vậy, các tỷ số kép khác
x 2 − x 3 x 2 − y2
W (x2 , y3 , x3 , y2 ) = :
y3 − x3 y3 − y2
4.2. Đường tròn đơn vị 37

x 3 − x 4 x 3 − y3
W (x3 , y4 , x4 , y3 ) = :
y4 − x4 y4 − y3
x 4 − x 1 x 4 − y4
W (x4 , y1 , x1 , y4 ) = :
y1 − x1 y1 − y4
là những số thực. Do đó
W (x1 , y2 , x2 , y1 )W (x3 , y4 , x4 , y3 )
=
W (x2 , y3 , x3 , y2 )W (x4 , y1 , x1 , y4 )
  
x1 − x2 x1 − x4 y1 − y2 y1 − y3
= : :
x3 − x2 x3 − x4 y3 − y2 y3 − y4
= W (x1 , x2 , x3 , x4 )W (y1 , y2 , y3 , y3 ).

là một số thực. Nhưng vì W (x1 , x2 , x3 , x4 ) là một số thực do giả thiết


những điểm X1 , X2 , X3 , X4 nằm trên cùng một đường tròn hoặc
đường thẳng, thì số W (y1 , y2 , y3 , y4 ) cũng là một số thực, nghĩa là
Y1 , Y2 , Y3 , Y4 nằm trên cùng một đường tròn hoặc đường thẳng.

4.2. ĐƯỜNG TRÒN ĐƠN VỊ


Có rất nhiều bài toán liên quan tới đường tròn khi ta chọn toạ
độ vuông góc với gốc chính là tâm đường tròn đó và coi đường tròn
là đường tròn đơn vị, thì một loạt các công thức tính toán trở nên
đơn giản, dễ nhớ và áp dụng được trong các bài toán cụ thể.
Như ta đã biết, sự vuông góc hoặc song song của hai đoạn thẳng
Z1 Z2 và U1 U2 được biểu diễn bằng công thức

(z2 − z1 )(u2 − u1 ) + (u2 − u1 )(z 2 − z 1 ) = 0


(z2 − z1 )(u2 − u1 ) = (u2 − u1 )(z 2 − z 1 )
Trong trường hợp Z1 , Z2 , U1 , U2 đều nằm trên đường tròn đơn vị,
1 1 1 1
thì những số liên hợp z 1 , z 2 , u1 , u2 có thể thay bằng , , , .
z1 z2 u1 u2
38 Chương 4. Phương trình đường tròn

Khi đó
   
1 1 1 1
(z2 − z1 ) − + (u2 − u1 ) − =0
u2 u1 z2 z1

Suy ra, Z1 Z2 và U1 U2 vuông góc với nhau khi và chỉ khi


z1 z2 + u1 u2 = 0 . Tương tự điều kiện cần và đủ để hai đoạn trên
song song là z1 z2 = u1 u2 .

Ví dụ 4.2. 2n-tứ giác A1 A2 ...A2n nội tiếp trong đường tròn. Từ điểm
bất kỳ B1 trên đường tròn kẻ dây cung B1 B2 song song hoặc vuông
góc với A1 A2 . Sau đó vẽ B2 B3 song song hoặc vuông góc với A2 A3 ,v.
v.. cuối cùng kẻ B2n−1 B2n song song hoặc vuông góc với A2n−1 A2n .
Chứng minh rằng B2n B1 song song hoặc vuông góc với A2n A1 .

Lời giải. Chọn hệ toạ độ vuông góc sao cho đường tròn là đơn vị.
Khi đó, theo công thức trên

a1 a2 = 1 b1 b2
b2 b3 = 2 a2 a3
......
a2n−1 a2n = 2n−1 b2n−1 b2n

ở đây 1 , 2 , ..., 2n−1 = ±1. Nhân từng vế các đawr ng thức trên và
giản ước đi ta được a1 a2n = b1 b2n , với  = 1 , 2 , ..., 2n−1 . Điều đó
chỉ ra rằng B1 B2n và A1 A2n song song hoặc vuông góc với nhau. J
4.3. GIAO ĐIỂM HAI CÁT TUYẾN
Điều kiện để ba điểm A, B và U nằm trên một đường thẳng cho
bởi phương trình
u−a u−a
= .
b−a b−a
4.3. Giao điểm hai cát tuyến 39

Nếu A và B là những điểm nằm trên đường tròn đơn vị, thì
1 1
a = , b = khi đó phương trình trên có thể viết
a b
a + b = u + abu

Đây cũng là điều kiện cần và đủ để U nằm trên đường thẳng AB.
Nếu Z1 Z2 và U1 U2 là hai cung của đường tròn đơn vị cắt nhau
thì giao điểm S của chúng cho bởi hệ
z1 + z2 = s + z1 z 2 s
u1 + u2 = s + u1 u2 s
sau khi loại s ta có công thức tính nhãn s của giao điểm:

(z1 + z2 )u1 u2 − (u1 + u2


s=
u1 u2 − z1 z2

Do Z1 Z2 và U1 U2 không song song nên u1 u2 − z1 z2 6= 0.


Ví dụ 4.3. Từ đỉnh A của một tứ giác ABCD nội tiếp trên đường tròn
dựng những đường vuông góc với các cạnh AB và AD lần lượt cắt các
cạnh CD và BC tại M và N .
Chứng minh rằng đường thẳng đi qua tâm của đường tròn chứa tứ
giác ABCD.

Lời giải. Chọn hệ toạ độ mà đường


tròn ngoại tiếp tứ giác ABCD làm
đường tròn đơn vị. Ta có M\ AB và
0
N AD bằng 90 . Gọi P và Q là giao
\
của AM và AN lần lượt với đường
tròn. Vì P
\ \ = 900 , vậy
AB = QAD
BP và DQ là những đuw ờng kính
Hình 4.3.
của đường tròn. Suy ra p = −b, q =
40 Chương 4. Phương trình đường tròn

−d.

Như vậy M, N là giao của các đường thẳng AP và CD; AQ và


BC. Do đó
(p + a)cd − (c + d)pa
m= =
cd − pa
(a − b)cd + (c + d)ab abc + abd + acd − bcd
= =
ab + cd ab + cd

(q + a)bc − (b + c)qa
n= =
bc − qa
(a − d)bc + (b + c)ad abc + abd + acd − bcd
= =
bc + ad ad + bc
m ad + bc
Khi đó = . Nhưng
n ab + cd
1 1 1 1
m a.d + b.c . + . ad + bc m
= = a d b c = = ,
n a.b + c.d 1 1 1 1 ab + cd n
. + .
a b c d
m −−→ −−→
hay là số thực, điều đó nghĩa là OM và ON cùng nằm trên một
n
trục hay là M, N, O thẳng hàng. J
4.4. GIAO ĐIỂM HAI TIẾP TUYẾN
Bây giờ, cho hai điểm Z, U trên đường tròn đơn vị với điều kiện
chúng không nằm trên cùng đường kính. Dựng hai đường thẳng tiếp
xúc với đường tròn tại hai điểm đó và chúng cắt nhau tại S. Ta tìm
cách biểu diễn nhãn s bởi z và u của hai điểm Z và U . Do SZ vuông
góc với OZ và SU vuông góc với OU ta có
z−s z−s u−s u−s
+ = 0 và + =0
z z u u
4.4. Giao điểm hai tiếp tuyến 41

hoặc là (z − s)z + (z − s)z = 0 và (u − s)u + (u − s)u = 0, ta có


sz + zs = 2 và su + su = 2 Từ đó suy ra

2zu
s= .
z+u

Ví dụ 4.4. Từ các đỉnh của tứ giác ABCD nội tiếp trong đường tròn
tâm O ta dựng các đường tiếp tuyến với đường tròn đó và chúng cắt
nhau tạo ra tứ giác P QRS.
Chứng minh rằng những điểm giữa các đường chéo của tứ giác P QRS
nằm cùng đường thẳng với tâm đường tròn.

Lời giải. Ta lấy đường tròn


đã cho làm đường tròn đơn
vị, theo công thức trên có
2ab 2bc
p= ,q= ,
a+b b+c
2cd 2da
r= ,s=
c+d a+d

Điểm trung bình M và N


của đường chéo P R và QS
có nhãn là Hình 4.4.
1 ab cd
m = (p + r) = +
2 a+b c+d
ab(c + d) + (a + b)cd abc + abd + acd + bcd
= =
(a + b)(c + d) (a + b)(c
1 bc da
n = (q + s) = + =
2 b+c d+a
bc(d + a) + (b + c)da bcd + abc + abd + acd
= =
(b + c)(d + a) (b + c)(d + a)
42 Chương 4. Phương trình đường tròn

m (b + c)(d + a)
Từ đó suy ra = . Nhưng
n (a + b)(c + d)
1 1 1 1
m (b + c)(d + a) ( + )( + )
= = b c a d = (b + c)(a + d) = m .
n (a + b)(c + d) 1 1 1 1 (a + b)(c + d) n
( + )( + )
a b c d
m
Do đó là một số thực, nghĩa là các điểm M, N nằm trên đường
n
thẳng đi qua O. J
4.5. CHÂN ĐƯỜNG VUÔNG GÓC Ở DÂY CUNG
Ta đi tìm công thức cho nhãn chân đường vuông góc hạ từ một
điểm M xuống đường thẳng AB, mà hai điểm A, B nằm trên đường
tròn đơn vị.
Như các phần trước ta có S nằm ở AB có công thức

a + b = s + abs

Mặt khác, M S vuông góc với AB, chúng ta có

(m − s)(a − b) + (m − s)(a − b) = 0

Từ đó suy ra s = m − (m − s)ab, thế s vào a + b = abs chúng ta có

1
s = (a + b + m − abm)
2

Ví dụ 4.5. Cho hình chữ nhật ABCD. Từ một điểm K bất kỳ trên
đường tròn ngoại tiếp hình chữ nhật hạ những đường thẳng vuông góc
xuống AB, CD, AD và BC và cắt các cạnh này lần lượt tại P, Q, R, S.
Chứng minh rằng P R vuông góc với QS và P S vuông góc với QR.
4.5. Chân đường vuông góc ở dây cung 43

Lời giải. Lấy đường tròn đơn vị R K


K S
R S
là đường tròn ngoại tiếp hình
D CC
chữ nhật ABCD chúng ta có D
c = −a và d = −b vì AC và BD Q
là đường kính của đường tròn.
Suy ra
1
p = (a + b + k − abk),
2 A
A P BB
1
q = (−a − b + k − abk)
2
1 Hình 4.5.
r = (a − b + k + abk),
2
1
s = (−a + b + k + abk).
2
Ta xét
1 1
p−r (a + b + k − abk) − (a − b + k + abk) a−k
= 2 2 = .
q−s 1 1 a+k
(−a − b + k − abk) − (−a + b + k + abk)
2 2

Tương tự có

p−s b−k
=
q−r b+k
Nhưng cả hai tỷ số là số hoàn toàn phức. Thật vậy,

1 1
p−r a−k −
= = a k = k − a = −p − r
q−s a+k 1 1 k+a q−s
+
a k

Điều khẳng định của bài toán là đúng. J


44 Chương 4. Phương trình đường tròn

4.6. BÀI TẬP


. 4.6. Cho O là một điểm trên mặt phẳng tam giác ABC. Những
điểm A1 , B1 , C1 là ảnh của đỉnh A, B, C đối xứng qua tâm O.
Chứng minh rằng những đường tròn ngoại tiếp các tam giác
ABC, A1 B1 C, A1 BC1 , AB1 C1 cắt nhau tại một điểm.

. 4.7. Trên một đường tròn có hai cung AB và CD. Nếu M là giao
điểm của các đường vuông góc dựng từ A đối với AB và từ C đối
với CD, còn N là giao điểm của các đường vuông góc dựng từ B
đối với AB và từ D đối với CD. Chứng minh rằng M N đi qua giao
điểm của hai đường thẳng BC và AD, nếu nó cắt nhau hoặc M N
song song với BC và AD, nếu chúng song song.

. 4.8. Trên đường tròn cho 4 điểm ABCD sao cho AB là đường
kính, còn CD thì không. Đường thẳng AC và BD cắt nhau tại điểm
X. Từ mỗi điểm C và D dựng các đường tiếp tuyến với đường tròn
và chúng cắt nhau tại Y . Chứng minh rằng đường thẳng XY vuông
góc với AB và đi qua giao điểm của hai đường thẳng AD và CB.

. 4.9. Trong đường tròn k kẻ hai đường kính AB và CD. M là điểm


bất kỳ trên đường tròn k, còn P và Q là chân đường vuông góc hạ
từ M xuống AB và CD. Chứng minh rằng độ dài đoạn P Q không
phụ thuộc vào vị trí của M trên đường tròn.

. 4.10. Từ trung điểm M của dây cung P Q trong một đường tròn,
dựng hai dây cung bất kỳ AB và CD. Dây cung AD và BC cắt P Q
lần lượt tại X và Y . Chứng minh rằng M là trung điểm của đoạn
XY .

. 4.11. Cho tứ giác ABCD nội tiếp trong đường tròn. Các đường
tiếp tuyến tại A và C cắt nhau tại điểm M ; còn các tiếp tuyến tại
4.6. Bài tập 45

B và D cắt nhau tại N . Chứng minh rằng các điểm A, C và N nằm


trên một đường thẳng khi và chỉ khi các điểm B, D và M nằm trên
một đường thẳng.

. 4.12. Cho hình bình hành ABCD. Dựng đường tròn với đường
kính AC. Gọi M và N là những điểm cắt thứ hai của đường tròn vừa
dựng với đường thẳng AB và AD. Chứng minh rằng đường thẳng
BD, M N và đường tiếp tuyến với đường tròn tại C cắt nhau tại một
điểm.

. 4.13. Cho P là một điểm trên nửa đường tròn S với đường kính
AB. Lấy hai cung bằng nhau trên S là _. BC. và _. CD.. Chứng
minh rằng nếu AC và BP cắt nhau tại E và AD và CP cắt nhau tại
F , thì EF vuông góc với AD.

. 4.14. Tại hai đầu đường kính AB của đường tròn dựng hai tiếp
tuyến tA và tB . Qua điểm C trên tA , không trùng với A dựng hai
cung bất kỳ D1 E1 và D2 E2 đối với đường tròn. Chứng minh rằng
các tia AD1 và AD2 cắt đường thẳng tB thành một đoạn thẳng có
độ dài bằng độ dài đoạn thẳng do hai tia AE1 và AE2 tạo ra cũng
trên tB .

. 4.15. Trên nửa đường tròn đường kính AB lấy hai điểm bất
kỳ C và D. Những điểm P, Q, R là trung điểm của những đoạn
AC, CD, BD. Qua mỗi điểm P và R dựng các đường thẳng vuông
góc lần lượt với AQ và BQ, chúng cắt tiếp tuyến tại A và B tại các
điểm S và T . Chứng minh rằng ST và CD song song.
Chương 5
ĐƯỜNG THẲNG
VÀ ĐƯỜNG TRÒN EULER

5.1. Nhãn các điểm đăc biệt trong tam giác . . . . . . . . . . . . 46


5.2. Ví dụ . . . . . . . . . . . . . . . . . . . . . . . . . . . . . . . . . . . . . . . . . . . . . . . . . . . 49
5.3. Bài tập. . . . . . . . . . . . . . . . . . . . . . . . . . . . . . . . . . . . . . . . . . . . . . . . . 52

Những ví dụ và bài tập hình học ở chương trước đã được giải


bằng cách dùng những tính chất của số phức. Trong chương này ta
dùng phương pháp số phức tiếp tục nghiên cứu và giải những bài
tập trong hình học phẳng theo một chủ đề riêng.

5.1. NHÃN CÁC ĐIỂM ĐĂC BIỆT TRONG


TAM GIÁC
Khi giải các bài toán nếu ta chọn hệ toạ độ vuông góc thích hợp
thì công việc giải sau đó rất đơn giản. Tất nhiên việc chọn như vậy
không mất đi tính tổng quát hay tính tự nhiên của bài toán. Việc
nghiên cứu các tính chất hình học có liên quan đến tam giác người
ta thường lấy toạ độ vuông góc có gốc tại tâm đường tròn ngoại tiếp
tam giác đó và chính đường tròn đó làm đường tròn đơn vị. Cho
tam giác A1 A2 A3 , đường tròn ngoại tiếp tâm O, nhãn của tâm O là
o = 0 và nhãn của các đỉnh có tính chất |a1 | = |a2 | = |a3 | = 1 và
5.1. Nhãn các điểm đăc biệt trong tam giác 47

a1 a1 = 1, a2 a2 = 1, a3 a3 = 1. Bài toán đặt ra là hãy biểu diễn nhãn


những điểm đặc biệt của tam giác A1 A2 A3 theo a1 , a2 , a3 với cách
chọn toạ độ trên.

Trong mục này ta chỉ tìm nhãn của những điểm trọng tâm, trực
tâm, ... đã quen thuộc với chúng ta. Gọi B1 , B2 , B3 lần lượt là trung
điểm các cạnh A2 A3 , A3 A1 , A1 A2 . Theo tính toán của những phần
a2 + a3 a3 + a1
trước b1 = , b2 = ,
2 2
a1 + a2
b3 = . Gọi G là trọng tâm của
2
tam giác. Nhận xét thấy rằng một
điểm có nhãn  
1 1 2 a2 + a3
(a1 +a2 +a3 ) = a1 +
3 3 3 2
thuộc đường thẳng nối từ A1 đến
B1 , theo phương trình đường thẳng
Hình 5.1.
có thông số.
Tương tự, do tính đối xứng của a1 , a2 , a3 nên điểm có nhãn
như trên cũng nằm trên A2 B2 và A3 B3 . Do đó điểm có nhãn
như vậy chính là trọ ng tâm của tam giác A1 A2 A3 hay là
1
g = (a1 + a2 + a3 )
3
Chúng ta lấy H3 là điểm đối xứng tâm đường tròn qua A1 A2 ,
vậy OA1 H3 A2 là hình thoi, do đó h3 = a1 + a2 . Mặt khác, xét H là
đỉnh hình bình hành A3 OH3 H. Khi đó h = h3 + a3 = a1 + a2 + a3 .
Do A3 H song song với OH3 , suy ra H hằm trên đường cao của tam
giác hạ từ A3 của tam giác A1 A2 A3 . Do tính đối xứng của h đối với
a1 , a2 , a3 dễ dàng thấy rằng H nằm trên đường cao hạ từ A1 và A2 .
Như vậy, điểm H có nhãn h = a1 + a2 + a3 là trực tâm của tam
48 Chương 5. Đường thẳng và đường tròn Euler

giác.
1 1
Ta có g = h, nghĩa là |OG| = |OH| và H, G cùng nằm trên
3 3
một đường thẳng qua O, mà O là tâm đường tròn ngoại tiếp tam
giác A1 A2 A3 . Đường thẳng đi qua ba điểm O, H, G gọi là đường
thẳng Euler.
Gọi P1 , P2 , P3 lần lượt là chân đường cao hạ từ A1 , A2 , A3 xuống
các cạnh tương ứng. Do a1 , a2 , a3 đều nằm trên đường tròn, theo
công thức từ chương trước ta có
1
p3 = (a1 + a2 + a3 − a1 a2 a3 )
2
1
p2 = (a1 + a2 + a3 − a1 a3 a2 )
2
1
p1 = (a1 + a2 + a3 − a2 a3 a1 )
2
Gọi C1 , C2 , C3 là trung điểm của đoạn thẳng nối từ đỉnh A1 , A2 ,
h + a1 a2 + a3
A3 tới trực tâm H của tam giác. Khi đó c1 = = a1 + ,
2 2
a1 + a3 a1 + a2
tương tự, ta có c2 = + a2 , c3 = + a3 .
2 2
Gọi E là trung điểm của M3 C3 , khi đó
a1 + a2 a1 + a2
+ a3 + 1
e= 2 2 = (a1 + a2 + a3 ).
2 2
Do tính đối xứng của e đối với a1 , a2 , a3 thì E cũng chính là điểm
giữa của các đoạn M1 C1 , M2 C2 . Mặt khác, dễ thấy rằng
1
|e − m1 | = |e − m2 | = |e − m3 | =
2

1
|e − c1 | = |e − c2 | = |e − c3 | = .
2
Do đó 6 điểm M1 , M2 , M3 , C1 , C2 , C3 nằm trên cùng một đường tròn
σ tâm E và có bán kính bằng nửa bán kính đường tròn ngoại tiếp
5.2. Ví dụ 49

tam giác A1 A2 A3 . Ta cũng thấy E nằm trên đường thẳng Euler và


1
|OE| = |OH|, nghĩa là E nằm giữa trực tâm và tâm đường tròn
2
ngoại tiếp.
Bây giờ, ta xét
1 1 1
|e − p3 | = | − a1 a2 a3 | = |a1 ||a2 ||a3 | =
2 2 2
Do đó P3 cũng nằm trên đường tròn σ tâm E. Tương tự những chân
đường cao P1 , P2 cũng nằm trên đường tròn σ. Đường tròn tâm E
đi qua chín điểm trên gọi là đường tròn Euler hoặc là đường tròn
9 điểm của tam giác.

5.2. VÍ DỤ
Ví dụ 5.1. Chứng minh rằng trong một tam giác bất kỳ A1 A2 A3 ,
những điểm Q1 ,Q2 ,Q3 đối xứng của trực tâm H lần lượt qua các
cạnh A2 A3 , A3 A1 , A1 A2 nằm trên đường tròn ngoại tiếp tam giác
A1 A2 A3 .

Lời giải. Vẫn hình vẽ và toạ độ như phần trên, p3 tính theo công
h + q3
thức cũ. Do Q3 là đối xứng của H qua P3 ta có p3 = , hay là
2
q3 = 2p3 −h = a1 +a2 +a3 −a1 a2 a3 −a1 −a2 −a3 = −a1 a2 a3 . Nhưng
|q3 | = | − a1 a2 a3 | = |a1 ||a2 ||a3 | = 1. Do đó Q3 nằm trên đường tròn
ngoại tiếp tam giác A1 A2 A3 . Tương tự Q1 , Q2 cũng có tính chất của
bài toán. J
Ví dụ 5.2. Chứng minh rằng trong mọi tam giác A1 A2 A3 , những
điểm đối xứng D1 , D2 , D3 của trực tâm tam giác lần lượt qua trung
điểm M1 , M2 , M3 của các cạnh A2 A3 , A3 A1 , A1 A2 nằm trên đường
tròn ngoại tiếp tam giác đã cho.
50 Chương 5. Đường thẳng và đường tròn Euler

Lời giải. Vẫn hình vẽ và cách chọn toạ độ như trên, ta có


a1 + a2
m3 = , h = a1 + a2 + a3 . Do D3 đối xứng của H qua M3 ,
2
h + d3 a1 + a2 a1 + a2 + a3 + d3
nên m3 = , hay là = , từ đó có
2 2 2
d3 = −a3 . Ta thấy ngay D3 có toạ độ đối xứng với A3 qua tâm O và
|d3 | = |a3 | = 1, do vậy D3 nằm trên đường tròn ngoại tiếp. Tương
tự D1 , D2 cũng vậy. J
Ví dụ 5.3. Chứng minh rằng tiếp tuyến qua điểm M3 (như các bài
trên) đối với đường tròn 9 điểm của tam giác A1 A2 A3 song song với
tiếp tuyến tại đỉnh A3 đối với đường tròn ngoại tiếp tam giác A1 A2 A3 .

Lời giải. Hệ toạ độ và hình vẽ chọn như hình trên. Chúng ta tìm cách
chứng minh được đường thẳng OA và EM3 song song với nhau thì
tiếp tuyến qua M3 đối với đường tròn 9 điểm và tiếp tuyến qua A
đối với đường tròn ngoại tiếp tam giác cũng song song với nhau, vì
EM3 và OA3 lần lượt là bán kính của các đường tròn. Thật vậy, theo
công thức đã biết, ta xét
1 1
(e − m3 )(a3 − o) = (a1 + a2 + a3 − a1 − a2 )a3 = a3 a3
2 2

1 1
(e − m3 )(a3 − o) = (a1 + a2 + a3 − a1 − a2 )a3 = a3 a3
2 2
Do đó (e − m3 )(a3 − o) = (e − m3 )a3 , nghĩa là EM3 song song với
OA3 . J
Ví dụ 5.4. Cho M là trọng tâm của tam giác ABC, P là chân đường
cao hạ từ A, còn Q là giao điểm của đường tròn ngoại tiếp tam giác
ABC và đường thẳng đi qua A đồng thời song song với BC. Chứng
|QM |
minh rằng điểm M nằm trên đoạn P Q và = 2.
|M P |
5.2. Ví dụ 51

Lời giải. Chọn đường tròn đơn vị ngoại


tiếp tam giác ABC. Ta có
1
p = (a + b + c − bca),
2
1
m = (a + b + c).
3
Do AQ//BC, suy ra aq = bc, hay là
q = bca. Khi đó
1 Hình 5.2.
m−q (a + b + c) − bca
= 3 =2
p−m 1 1
(a + b + c − bca) − (a + b + c)
2 3
Tỷ số đơn là một số thực, nghĩa là góc giữa hai đường thẳng này
bằng 0 hoặc ±π, do đó P, M, Q nằm trên đường thẳng. Ta thấy ngay
|QM |
|M P |
= 2. J
rằng B1 B2 vuông góc với OA3 .
Ví dụ 5.5. Cho B1 và B2 lần lượt là
chân đường cao hạ từ đỉnh A1 và A2
xuống các cạnh đối diện trong tam giác
A1 A2 A3 . O là tâm đường tròn ngoại
tiếp tam giác A1 A2 A3 . Chứng minh
Lời giải. Chọn đường tròn ngoại tiếp
tam giác là đường tròn đơn vị. Ta có
1 Hình 5.3.
b1 = (a1 + a2 + a3 − a1 a2 a3 ),
2
1
b2 = (a1 + a2 + a3 − a1 a2 a3 )
2
do đó
1 1
b2 − b1 = (a1 a2 a3 − a1 a2 a3 ) = a3 (a1 a2 − a1 a2 ).
2 2
52 Chương 5. Đường thẳng và đường tròn Euler

Nhưng số a1 a2 − a1 a2 hoàn toàn phức. Nghĩa là B1 B2 vuông góc


với OA3 . J
5.3. BÀI TẬP
. 5.6. Cho O là tâm của đường tròn ngoại tiếp tam giác A1 A2 A3
. Điểm B1 , B2 , B3 là những điểm đối xứng của O lần lượt qua các
cạnh A2 A3 , A3 A1 , A1 A2 . Chứng minh rằng hai tam giác A1 A2 A3
và B1 B2 B3 bằng nhau và đối xứng qua một điểm (đó là tâm của
đường tròn Euler).

. 5.7. Chứng minh rằng đường thẳng nối giữa điểm đối xứng của
trực tâm qua một đỉnh với điểm đối xứng của chính đỉnh này qua
điểm giữa của cạnh đối diện, đi qua tâm đường tròn ngoại tiếp tam
giác đó.

. 5.8. Trong tam giác A1 A2 A3 , cho |A1 A2 | = |A3 H|, ở đây H là trực
tâm của tam giác đã cho. Hãy tính góc của tam giác ở đỉnh A3 .

. 5.9. Trên đường tròn cho ba điểm A1 , A2 , A3 . Lấy điểm C3 đối


xứng của A3 qua trung điểm A1 A2 . Điểm D3 đối xứng của A3 qua
tâm đường tròn. Chứng minh rằng A1 A2 và C3 D3 vuông góc với
nhau.

. 5.10. Cho tam giác A1 A2 A3 với trực tâm H, vẽ đường tròn đường
kính A3 H, nó cắt các cạnh A2 A3 và A1 A3 tại P và Q. Chứng minh
rằng những tiếp tuyến tại điểm P và Q đối với đường tròn cắt nhau
tại điểm giữa của A1 A2 .

. 5.11. Cho đường tròn ngoại tiếp tam giác A1 A2 A3 , gọi N là giao
của đường kính của đường tròn đi qua đỉnh A3 và cạnh A1 A2 . Chứng
minh rằng trung điểm M của đoạn A3 N , chân đường cao P3 hạ từ
5.3. Bài tập 53

A3 và tâm E của đường tròn Euler tam giác A1 A2 A3 nằm trên một
đường thẳng.

. 5.12. Chứng minh rằng những điểm đối xứng của tâm đường
tròn ngoại tiếp tam giác qua các điểm giữa các trung tuyến nằm
trên đường cao có chung đỉnh của tam giác đó.

. 5.13. Cho O là tâm của đường tròn ngoại tiếp tam giác A1 A2 A3
và H là trực tâm của tam giác. Những đường thẳng A1 H, A2 H, A3 H
cắt đường tròn ngoại tiếp lần lượt tại Q1 , Q2 , Q3 . Chứng minh rằng
những đường thẳng đi qua Q1 , Q2 , Q3 lần lượt song song với OA1 ,
OA2 , OA3 cắt nhau tại một điểm.

. 5.14. Cho tam giác A1 A2 A3 , H là trực tâm của tam giác, D là


trung điểm cạnh A1 A2 . Đường thẳng HD cắt đường tròn ngoại tiếp
tam giác A1 A2 A3 tại K. Chứng minh rằng D là điểm giữa của đoạn
HK.

. 5.15. Cho tam giác ABC, trực tâm H chia đường cao BD với tỷ
số 3:1 (tính từ đỉnh B). Còn K là trung điểm của đường cao này.
\ = 900 .
Chứng minh rằng góc AKC
Chương 6
ĐƯỜNG THẲNG SIMSON

6.1. Ba điểm trên đường thẳng Simson . . . . . . . . . . . . . . . . . . . 54


6.2. Ví dụ . . . . . . . . . . . . . . . . . . . . . . . . . . . . . . . . . . . . . . . . . . . . . . . . . . . 55
6.3. Bài tập. . . . . . . . . . . . . . . . . . . . . . . . . . . . . . . . . . . . . . . . . . . . . . . . . 59

6.1. Ba điểm trên đường thẳng Simson


Cho tam giác A1 A2 A3 . Chúng ta chọn đường tròn ngoại tiếp tam
giác làm đường tròn đơn vị, do đó nhãn a1 , a2 , a3 của các đỉnh của
tam giác có đẳng thức |a1 |=|a2 |=|a3 |=1.

Một điểm P bất kỳ trên


đường tròn, từ P ta hạ lần
P2 P2
lượt các đường vuông góc P P1 ,
A3A3 AP
P P2 , P P3 xuống các cạnh
A2 A3 , A3 A1 , A1 A2 .
P1
Chúng ta sẽ chứng minh rằng O
chân ba đường vuông góc này O
A A2
nằm trên một đường thẳng. A11 P3 A2
Thật vậy, ta có
1
p1 = (a2 + a3 + p − a2 a3 p)
2 Hình 6.1.
1
p2 = (a3 + a1 + p − a3 a1 p)
2
6.2. Ví dụ 55

1
p3 = (a1 + a2 + p − a1 a2 p)
2
Ta xét
p1 − p3 a3 − a1 − a2 a3 p + a1 a2 p
V (p1 , p2 , p3 ) = = =
p2 − p3 a3 − a2 − a3 a1 p + a1 a2 p
(a3 − a1 )(1 − a2 p) (a3 − a1 )(p − a2 )
= =
(a3 − a2 )(1 − a1 p) (a3 − a2 )(p − a1 )
V (a3 , p, a1 )
= = W (a3 , p, a1 , a2 ).
V (a3 , p, a2 )

Vì các điểm A1 , A2 , A3 , P nằm trên đường tròn, nên tỷ số kép


W (a3 , p, a1 , a2 ) là số thực, suy ra tỷ số đơn V (p1 , p2 , p3 ) cũng là số
thực. Nghĩa là những điểm P1 , P2 , P3 nằm trên một đường thẳng.
(a3 − a1 )(p − a2 )
Có thể kiểm tra tỷ số là số thực trực tiếp qua số
(a3 − a2 )(p − a1 )
liên hợp của chúng cũng được. Đường thẳng đi qua ba điểm trên gọi
là đường thẳng Simson của điểm P đối với tam giác A1 A2 A3 .

6.2. Ví dụ
Ví dụ 6.1. Nếu P là một điểm trong mặt phẳng tam giác A1 A2 A3 và
chân đường vuông góc P1 ,P2 ,P3 xuống các cạnh của tam giác hạ từ
P nằm trên một đường thẳng, thì điểm P nằm trên đường tròn ngoại
tiếp tam giác A1 A2 A3 .

Lời giải. Do P1 ,P2 , P3 nằm trên một đường thẳng, nên tỷ số


(a3 − a1 )(1 − a2 p)
V (p1 , p2 , p3 ) =
(a3 − a2 )(1 − a1 p)
là một số thực. Nghĩa là
(a3 − a1 )(1 − a2 p) (a3 − a1 )(1 − a2 p)
=
(a3 − a2 )(1 − a1 p) (a3 − a2 )(1 − a1 p)
56 Chương 6. Đường thẳng Simson
  
1 1 p
− 1−
a3 a1 a2 (a − a1 )(a2 − p)
=  = 3
1 1 p (a3 − a2 )(a1 − p)
− 1−
a3 a2 a1
Từ đó suy ra (1 − a2 p)(a1 − p) = (1 − a1 p)(a2 − p). Do đó (1 −
|p|2 )(a1 − a2 ) = 0, suy ra |p|2 = 1. Điều này có nghĩa là P nằm trên
đường tròn đơn vị. J
Ví dụ 6.2. Hãy viết phương trình đường thẳng Simson của điểm P
đối với tam giác A1 A2 A3 thông qua nhãn p và a1 , a2 , a3 .

Lời giải. Gọi P1 ,P2 ,P3 là chân các đường vuông góc hạ từ P xuống
các cạnh của tam giác. Như ở chương 3, ta đã biết đường thẳng đi
qua hai điểm P1 , P2 có phương trình
(p2 − p1 )z − (p2 − p1 )z + (p1 p2 − p1 p2 ) = 0
Chia đẳng thức trên cho (p2 − p1 ), ta nhận được
p2 − p1
z− z+c=0
p2 − p1
Bởi vì
p2 − p1 a1 − a3 a1 p − a2 + a2 a3 p
=
p2 − p1 a1 − a3 a1 p − a2 + a2 a3 p
 
a3
(a1 − a2 ) 1 −
(a1 − a2 )(1 − a3 p) p
= =  
(a1 − a2 )(1 − a3 p) 1 1 p
− 1−
a1 a2 a3
a1 a2 a3
= = a1 a2 a3 p
p
Ta có z − a1 a2 a3 pz + c = 0
1
Để tính hệ số c, thay giá trị z = p3 = (a1 + a2 + p − a1 a2 p).
2
1 1 1 1
Chúng ta có c = a1 a2 a3 pp3 − p3 . Vì a1 = ,a2 = ,a3 = , p =
a1 a2 a3 p
6.2. Ví dụ 57

ta có
1 1
c = a1 a2 a3 p(a1 + a2 + a3 + p) − (a1 + a2 + a3 + p)
2 2

Vậy phương trình đường thẳng Simson cho điểm P đối với tam
giác A1 A2 A3 là

1 1
z − a1 a2 a3 pz + a1 a2 a3 p(a1 + a2 + a3 + p) − (a1 + a2 + a3 + p) = 0
2 2

Ví dụ 6.3. Cho P là một điểm trên đường tròn ngoại tiếp tam giác
A1 A2 A3 và Q1 , Q2 , Q3 là những điểm đối xứng của P với các cạnh
tam giác A2 A3 , A3 A1 , A1 A2 . Chứng minh rằng những điểm Q1 , Q2 ,
Q3 nằm trên đường thẳng chứa trực tâm của tam giác A1 A2 A3 .

Lời giải. Những điểm Q1 , Q2 , Q3 nằm trên đường thẳng do chúng


là vị tự của các điểm P1 , P2 , P3 với tâm ở P và hệ số 2. Mà P1 , P2 ,
P3 nằm trên đường thẳng Simson nên suy ra Q1 , Q2 , Q3 cũng nằm
trên một đường thẳng.
Chúng ta sẽ kiểm tra trực tâm H Q2
Q3 P3
có nằm trên đường thẳng Q1 , Q2 , Q3 P2
A3
không ?. Do P3 là trung điểm của A 3 PP

P Q3 , ta có P1
H
q3 = 2p3 − p H O
Q1
Q1 O
= a1 + a2 + p − a1 a2 p − p AA11 P3
A2
A2

= a1 + a2 − a1 a2 p
Tương tự, Q3
q2 = a3 + a1 − a3 a1 p, Q3
Hình 6.2.
q1 = a2 + a3 − a2 a3 p
58 Chương 6. Đường thẳng Simson

Ta tính
q3 − h a1 + a2 − a1 a2 p − a1 − a2 − a3
==
q1 − h a2 + a3 − a2 a3 p − a1 − a2 − a3
a3 + a1 a2 p a1 a2 + a3 p
= =
a1 + a2 a3 p a2 a3 + a1 p
Dễ dàng kiểm tra tỷ số này là một số thực. Nghĩa là H nằm trên
Q1 Q3 . J
Ví dụ 6.4. Cho tam giác nội tiếp đường tròn O và một điểm M thuộc
đường tròn đó. Gọi M 0 là một điểm đối xứng của M qua tâm O. Chứng
minh rằng hai đường thẳng Simson của các điểm M và M 0 vuông góc
với nhau.

Lời giải. Vẫn chọn hệ toạ độ và ký hiệu như các bài trên. Ta có
m = −m0 . Ta gọi các điểm hạ từ M xuống các cạnh của tam giác
A1 A2 A3 là P1 , P2 , P3 còn từ M 0 là P10 , P20 , P30 . Theo cách tính toán
như các bài trước ta có p2 − p1 = (a2 − a1 )(m − a3 ) và
p2 − p1 a3 − m
p02 − p01 = (a2 − a1 )(m0 − a3 ), do đó 0 0 = , nhưng
p2 − p1 a3 + m
1 1

p2 − p1 a3 − m a3 m a3 − m p2 − p1
0 0 = = =− =− 0
p2 − p1 a3 + m 1 1 a3 + m p2 − p01
+
a3 m
Đẳng thức này là một số hoàn toàn phức, nghĩa là đường thẳng P2 P1
và P20 P10 vuông góc với nhau. J
Ví dụ 6.5. Chứng minh rằng trên đường tròn ngoại tiếp tam giác có
một điểm mà đường thẳng Simson của điểm này đối với tam giác,
vuông góc với đường thẳng Euler của tam giác này.

Lời giải. Lấy đường tròn ngoại tiếp tam giác A1 A2 A3 làm đơn vị
cho hệ toạ độ. Điểm P ta phải tìm ? Nếu P tồn tại như vậy, P1 ,
6.3. Bài tập 59

P2 , P3 là chân đường vuông góc hạ từ P xuống các cạnh của tam


giác. Tính toán như bài trước ta có p2 − p1 = (a2 − a1 )(1 − a3 p) và
h = a1 + a2 + a3 . Nếu đường thẳng Simson của P đối với tam giác
vuông góc với đường thẳng Euler, thì
(a2 − a1 )(1 − a3 p) (a2 − a1 )(1 − a3 p)
=−
a1 + a2 + a3 a + a2 + a3
 1  
a3 1 1 p
(a2 − a1 )(1 − ) − 1−
p a2 a1 a3
=−
a1 + a2 + a3 1 1 1
+ +
a1 a2 a3
1 1
=−  
p(a1 + a2 + a3 ) 1 1 1
a1 a2 a3 + +
a1 a2 a3
a1 a2 + a2 a3 + a3 a1
p=−
a1 + a2 + a3
Đây chính là nhãn của điểm mà đường Simson vuông góc với đường
Euler. J
6.3. Bài tập
. 6.6. Cho điểm P trên đường tròn ngoại tiếp tam giác A1 A2 A3 . Từ
P hạ các đường vuông góc xuống các cạnh A2 A3 , A3 A1 , A1 A2 và
lần lượt cắt đường tròn tại các điểm Q1 , Q2 , Q3 . Chứng minh rằng
những đường thẳng A1 Q1 , A2 Q2 , A3 Q3 song song với đường thẳng
Simson của điểm P .

. 6.7. Cho P là một điểm trên đường tròn ngoại tiếp tam giác
A1 A2 A3 . Nếu đường thẳng Simson của điểm P cắt đường cao từ
đỉnh A1 tại điểm K và P chiếu vuông góc xuống cạnh A2 A3 tại P1 ,
chứng minh rằng P K song song với HP1 , ở đây H là trực tâm tam
giác A1 A2 A3 .
60 Chương 6. Đường thẳng Simson

. 6.8. Những đường cao của tam giác A1 A2 A3 cắt đường tròn ngoại
tiếp tại các điểm Q1 , Q2 , Q3 . Chứng minh rằng với điểm P bất kỳ
trên đường tròn, những đường thẳng Simson của P đối với tam giác
A1 A2 A3 và tam giác Q1 Q2 Q3 vuông góc với nhau.

. 6.9. Chứng minh rằng nếu đường thẳng Simson của P đối với tam
giác A1 A2 A3 đi qua điểm đối xứng qua tâm đường tròn của điểm
P , thì nó cũng đi qua trọng tâm của tam giác.

. 6.10. Chứng minh rằng nếu đường thẳng Simson của P đối với
tam giác A1 A2 A3 trong đường tròn ngoại tiếp tâm O song song với
OA1 , thì P A1 song song với A2 A3 .

. 6.11. Cho P là điểm trên đường tròn ngoại tiếp tam giác A1 A2 A3
và H là trực tâm của tam giác. Chứng minh rằng đường thẳng Sim-
son của điểm P chia đôi đoạn P H (và điểm này nằm trên đường
tròn Euler).

. 6.12. Chứng minh rằng góc giữa hai đường thẳng Simson của hai
điểm khác nhau P và Q bằng nửa góc dây cung giữa hai điểm này.

. 6.13. Cho đường tròn ngoại tiếp tam giác A1 A2 A3 tâm O. Những
điểm B1 , B2 , B3 lần lượt là các điểm đối xứng của A1 , A2 , A3 qua
tâm O. Chứng minh rằng những đường thẳng Simson của một điểm
bất kỳ trên đường tròn đối với tam giác A1 A2 A3 và B1 B2 B3 vuông
góc với nhau.

. 6.14. Cho A1 B1 , A2 B2 , A3 B3 là những đường cao của tam giác


A1 A2 A3 . Đường thẳng A1 B1 , A2 B2 , A3 B3 cắt đường tròn ngoại tiếp
tam giác A1 A2 A3 tại Q1 , Q2 , Q3 . Những đường thẳng Simson của
các điểm Q1 , Q2 , Q3 đối với tam giác A1 A2 A3 cắt nhau tạo thành
6.3. Bài tập 61

tam giác C1 C2 C3 . Chứng minh rằng trọng tâm của tam giác B1 B2 B3
và C1 C2 C3 trùng nhau.

. 6.15. Cho O là tâm đường tròn ngoại tiếp tam giác A1 A2 A3 , lấy
B1 , B2 , B3 lần lượt là các điểm đối xứng qua tâm O của các đỉnh tam
giác A1 , A2 , A3 . Chứng minh rằng những đường thẳng Simson của
các điểm B1 , B2 , B3 đối với tam giác A1 A2 A3 cắt nhau tại những
điểm trên đường tròn.
Chương 7
TỨ GIÁC NỘI TIẾP ĐƯỜNG TRÒN

7.1. Các điểm đặc biệt của tứ giác nội tiếp . . . . . . . . . . . . . 62


7.2. Ví dụ . . . . . . . . . . . . . . . . . . . . . . . . . . . . . . . . . . . . . . . . . . . . . . . . . . . 64
7.3. Bài tập. . . . . . . . . . . . . . . . . . . . . . . . . . . . . . . . . . . . . . . . . . . . . . . . . 67

7.1. CÁC ĐIỂM ĐẶC BIỆT CỦA TỨ GIÁC NỘI TIẾP


Cho tứ giác A1 A2 A3 A4 nội tiếp đường tròn tâm O. Chọn toạ độ
sao cho đường tròn tâm O làm đơn vị.
Nhãn ở đir nh của tứ giác lần lượt AA44
là a1 , a2 , a3 , a4 . Cứ lấy ba đỉnh của
tứ giác thì ta có một tam giác như
O A3
A1 A2 A3 , A1 A2 A4 , A2 A3 A4 , A3 A4 A1 . O A3
Mỗi tam giác này đều có những điểm A1
A1
và đường đặc biệt mà ta đã xét ở A
A2 2
những phần trước, ta ký hiệu M1 ,
M2 , M3 , M4 là trọng tâm của các tam Hình 7.1.
giác đã liệt kê ở trên. Tương tự cho các điểm trực tâm H1 , H2 , H3 ,
H4 . Tâm đường tròn Euler là E1 , E2 , E3 , E4 .
Như ta đã biết trung điểm của các cạnh A1 A2 , A2 A3 ,
A3 A4 , A4 A1 được ký hiệu là M12 ,M23 , M34 , M41 và tính được
a1 + a2 a2 + a3 a3 + a4 a4 + a1
m12 = , m23 = , m34 = , m41 = . Ta
2 2 2 2
7.1. Các điểm đặc biệt của tứ giác nội tiếp 63

cũng dễ dàng thấy rằng các đường nối hai trung điểm các cạnh đối
a1 + a2 + a3 + a4
diện cắt nhau tại điểm M và tính được nhãn m = .
4
Phải chăng đây là điểm trọng tâm của tứ giác ? Như ta đã biết trọng
a3 + a4 + a1
tâm của tam giác A3 A4 A1 có nhãn m3 = , khi đó ta có
3
3 a3 + a4 + a1 1 3 1
thể viết m = + a2 = m3 + a2 . Điều này nói lên
4 3 4 4 4
rằng điểm M nằm trên đường nối từ đỉnh A2 với trọng tâm tam giác
A3 A4 A1 (phương trình thông số). Tương tự M cũng nằm trên đường
nối từ một đỉnh tới trọng tâm của tam giác với 3 đỉnh còn lại.
Những đường thẳng nối một đỉnh của tứ giác với trọng tâm của
tam giác tạo bởi ba đỉnh còn lại cắt nhau tại một điểm, đó chính
là điểm M , gọi là trọng tâm của tứ giác nội tiếp A1 A2 A3 A4 . Như
chứng minh ở trên điểm M3 chia đoạn A3 M theo tỷ lệ A3 M3 :
M3 M = 3 : 1. Tương tự ta có A2 M2 : M2 M = A1 M1 : M1 M = 3 : 1
Cũng cách làm như trên, ta thấy rằng điểm có nhãn
1 1 1
h = (a1 + a2 + a3 + a4 ) = (a1 + a2 + a3 ) + a4 ,
2 2 2
nằm trên đường thẳng nối H4 và A4 ; do tính đối xứng của a1 ,
a2 ,a3 ,a4 suy ra điểm H nằm trên đường nối các đỉnh A1 với H1 ,
A2 với H2 , A3 với H3 . Nghĩa là, những đường thẳng nối một đỉnh
với trực tâm của tam giác tạo bởi ba đỉnh còn lại của một tứ giác cắt
nhau tại một điểm. Điểm này gọi là điểm trực tâm của tứ giác.
Như vậy, đường nối trực tâm, trọng tâm của tứ giác đi qua tâm
đường tròn ngoại tiếp tứ giác ấy (vì nhãn của tâm đường tròn là 0).
Đường thẳng như vậy đối với tứ giác ta cũng gọi là đường thẳng
Euler của tứ giác. Ta dễ dàng chứng minh được các tính chất sau:
- Trực tâm của tứ giác là tâm của đường tròn đi qua tâm các
đường tròn Euler của các tam giác tạo bởi ba đỉnh như trên.
64 Chương 7. Tứ giác nội tiếp đường tròn

Thật vậy,
1 1 1 1
|h − e4 | = | (a1 + a2 + a3 + a4 ) − (a1 + a2 + a3 )| = |a4 | =
2 2 2 2
1
do đó |h − e1 | = |h − e2 | = |h − e3 | = |h − e4 | = .
2
- Trên đường thẳng Euler của tứ giác có điểm G0 và H 0 với nhãn
1 1
g 0 = (a1 + a2 + a3 + a4 ) và h0 = (a1 + a2 + a3 + a4 ), lần lượt là
3 2
tâm đường tròn đi qua 4 trọng tâm và tâm của đường tròn đi qua
bốn trực tâm của những tam giác tạo bởi ba đỉnh. Dễ dàng kiểm tra
tính chất này và có thể tiếp tục những ý tưởng của tam giác cho tứ
giác. Chúng ta sẽ minh họa qua ví dụ và bài tập sau.

7.2. VÍ DỤ
Ví dụ 7.1. Từ mỗi đỉnh của tứ giác A1 A2 A3 A4 nội tiếp đường tròn
hạ những đường vuông góc xuống các cạnh và các đường chéo không
đi qua đỉnh đó. Chứng minh rằng chân những đường vuông góc trên
đường thẳng A1 A2 và A3 A4 nằm trên một đường tròn, chân đường
vuông góc trên A1 A4 và A2 A3 nằm trên đường tròn khác, tương tự
như vậy những điểm trên đường thẳng A1 A3 và A2 A4 nằm trên đường
tròn thứ ba, hơn nữa tâm của các đường tròn này trùng với trực tâm
của tứ giác.

Lời giải. Ta gọi axyz là nhãn của chân đường vuông góc hạ từ đỉnh
Ax xuống đường thẳng Ay Az ,x, y = 1, 2, 3, 4. Vậy ta có
1
a134 = (a1 + a3 + a4 − a3 a4 a1 ),
2
1
a234 = (a2 + a3 + a4 − a3 a4 a2 ),
2
1
a312 = (a3 + a1 + a2 − a1 a2 a3 ),
2
7.2. Ví dụ 65

1
a412 = (a4 + a1 + a2 − a1 a2 a4 ),
2
là nhãn của những chân đường vuông góc từ A1 , A2 xuống A3 A4
và từ A3 ,A4 xuống A1 A2 . Ta có
1 1
|h − a134 | = | (a1 + a2 + a3 + a4 ) − (a1 + a3 + a4 − a3 a4 a1 )
2 2
1 1
= |a2 + a3 a4 a1 | = |a1 a2 + a3 a4 |
2 2
Vì a1 nằm trên đuwờng tròn đow n vị , nên |a1 | = 1. Tương tự
|h − a234 | = |h − a312 | = |h − a412 | = 21 |a1 a2 + a3 a4 |. Do đó
a134 ,a234 ,a312 ,a412 nằm trên một đường tròn tâm H. J
Ví dụ 7.2. Cho A1 A2 A3 A4 là tứ giác nội tiếp đường tròn. Chứng minh
rằng đường thẳng Simson của bốn đỉnh đối với tam giác tạo bởi các
đỉnh còn lại, đi qua trực tâm của tứ giác đã cho.

Lời giải. Ta biết rằng với một điểm bất kỳ u trên đường tròn, thì
phương trình đường thẳng Simson đối với tam giác A1 A2 A3 có dạng
1 1
z − a1 a2 a3 uz + a1 a2 a3 u(a1 + a2 + a3 + u) − (a1 + a2 + a3 + u) = 0
2 2
1
Lấy u = a4 , dễ dàng kiểm tra được h = 2 (a1 + a2 + a3 + a4 ) thoả
mãn phương trình trên. Nghĩa là trực tâm của tứ giác nằm trên
đường thẳng Simson của A4 đối với tam giác A1 A2 A3 . Tương tự cho
3 đường thẳng Simson ứng với 3 đường còn lại. Do đó, H là giao
điểm của 4 đường thẳng Simson mô tả ở trên. J
Ví dụ 7.3. [Định lý Vail] Cho A1 A2 A3 A4 là một tứ giác nội tiếp đường
tròn. H1 , H2 , H3 , H4 lần lượt là trực tâm của các tam giác A2 A3 A4 ,
A1 A3 A4 , A1 A2 A4 , A1 A2 A3 . Khi đó đường thẳng Simson của A4 đối
với tam giác A1 A2 A3 cũng là đường thẳng Simson của H4 đối với tam
giác H1 H2 H3 .
66 Chương 7. Tứ giác nội tiếp đường tròn

Lời giải. Theo giả thiết ta có

h1 = a2 + a3 + a4 , h2 = a1 + a3 + a4
h3 = a1 + a2 + a4 , h4 = a1 + a2 + a3
h4 − h3 = −(a4 − a3 ) =⇒ |H4 H3 | = |A4 A3 |
h4 − h2 = −(a4 − a2 ) =⇒ |H4 H2 | = |A4 A2 |
h1 − h3 = −(a1 − a3 ) =⇒ |H1 H3 | = |A1 A3 |
h1 − h2 = −(a1 − a2 ) =⇒ |H1 H2 | = |A1 A2 |

Ta thấy rằng hai đa giác A1 A2 A3 A4 và H1 H2 H3 H4 có các cạnh


tương ứng bằng nhau và song song từng đôi một. Ta cũng thấy rằng
A4 đối với A1 A2 A3 cũng giống như H4 đối với H1 H2 H3 , vậy đường
Simson của A4 đối với A1 A2 A3 cũng giống như đường thẳng Simson
của H4 đối với H1 H2 H3 . J
Ví dụ 7.4. Nếu A1 ,A2 ,A3 ,A4 là bốn điểm khác nhau trên đường tròn,
đường thẳng Simson của A1 đối với A2 A3 A4 vuông góc với đường
thẳng Euler của tam giác. Chứng minh rằng đường thẳng Simson của
mỗi điểm còn lại với tam giác tạo bởi 3 điểm kia cũng vuông góc với
đường thẳng Euler của cùng tam giác ấy.

Lời giải. Điều kiện để đường thẳng Simson của điểm A1 đối với tam
giác A2 A3 A4 và đường thẳng Euler của tam giác A2 A3 A4 vuông góc
(a2 − a4 )(a1 − a3 )a1
với nhau là k + k = 0, ở đây k = (xem lại ví
a2 + a3 + a4
dụ 6.5), từ đó suy ra

(a2 − a4 )(a1 − a3 )a1 (a2 − a4 )(a1 − a3 )a1


=−
a2 + a3 + a4 a2 + a3 + a4
7.3. Bài tập 67

hay là
  
1 1 1 1
1 − − a1
a1 (a2 − a4 )(a1 − a3 ) a2 a4 a1 a3
=−
a1 (a2 + a3 + a4 ) 1 1 1
+ +
a2 a3 a4
Vậy, a1 a2 + a1 a3 + a1 a4 + a2 a3 + a2 a4 + a3 a4 = 0. Điều kiện này đối
xứng đối với a1 ,a2 ,a3 ,a4 . Do đó nó không thay đổi khi ta thay đỉnh
A1 bằng một trong các đỉnh A2 ,A3 ,A4 . J
Ví dụ 7.5. Cho tứ giác ABCD nội tiếp trong đường tròn. Chứng minh
rằng nếu U1 và U2 là trực tâm của những tam giác ABD và ACD thì
đường thẳng U1 U2 song song với BC.

Lời giải. Chọn đường tròn ngoại tiếp tứ giác làm đơn vị, các nhãn ở
đỉnh là a, b, c, d. Khi đó u1 = a + b + d và u2 = a + c + d, từ đó suy
ra u1 − u2 = b − c nghĩa là U1 U2 //BC, hơn nữa |U1 U2 | = |BC|. Vậy
BCU1 U2 là hình bình hành. J
7.3. BÀI TẬP
. 7.6. Cho tứ giác ABCD nội tiếp trong đường tròn. Từ các điểm A
và B hạ đường vuông góc AA2 và BB2 xuống CD; Từ các điểm B
và C hạ đường vuông góc BB1 và CC1 xuống DA; Từ các điểm C
và D hạ đường vuông góc CC2 và DD2 xuống AB; cuối cùng từ các
điểm D và A hạ đường vuông góc DD1 và AA1 xuống BC. Chứng
minh các đoạn thẳng A1 A2 , B1 B2 , C1 C2 , D1 D2 bằng nhau và đi qua
trực tâm của tứ giác.
. 7.7. Cho tứ giác ABCD nội tiếp đường tròn. Đường vuông góc
với BA từ đỉnh A cắt CD tại M ; đường vuông góc với DA từ đỉnh A
cắt BC tại N . Chứng minh rằng M N đi qua tâm đường tròn ngoại
tiếp tứ giác.
68 Chương 7. Tứ giác nội tiếp đường tròn

. 7.8. Cho tứ giác A1 A2 A3 A4 (không cần nội tiếp đường tròn). Lấy
các điểm B1 , B2 , B3 , B4 lần lượt nằm trên các cạnh A1 A2 , A2 A3 ,
A3 A4 , A4 A1 và chia theo cùng một tỷ số. Chứng minh rằng trọng
tâm của tứ giác A1 A2 A3 A4 và B1 B2 B3 B4 trùng nhau.

. 7.9. Cho tứ giác A1 A2 A3 A4 nội tiếp đường tròn. Chứng minh rằng
đường thẳng nối trung điểm của mỗi cạnh (hoặc của đường chéo)
với trực tâm tứ giác A1 A2 A3 A4 vuông góc với cạnh đối diện (hoặc
đường chéo kia).

. 7.10. (Định lý Ptolemei) Chứng minh rằng tích của độ dài đường
chéo của một tứ giác bất kỳ nhỏ hơn hoặc bằng tổng của tích độ dài
các cạnh đối diện của nó. Nếu tứ giác nội tiếp đường tròn thì ta có
đẳng thức.

. 7.11. Cho tứ giác A1 A2 A3 A4 nội tiếp đường tròn O và điểm P


bất kỳ trên mặt phẳng tứ giác. Ký hiệu P12 ,P13 , P14 , P23 , P24 , P34
là những điểm đối xứng của P lần lượt qua các cạnh A1 A2 , A1 A3 ,
A1 A4 , A2 A3 , A2 A4 , A3 A4 . Lấy R, S, T lần lượt là trung điểm của các
đoạn P12 P34 , P13 P24 , P14 P23 và cho O0 là điểm đối xứng của O qua
trọng tâm của tứ giác A1 A2 A3 A4 . Chứng minh rằng R, S, T, O0 nằm
trên một đường thẳng.

. 7.12. Cho tứ giác A1 A2 A3 A4 nội tiếp đường tròn O, về phía tâm


của đường tròn dựng những tam giác cân với đáy là mỗi cạnh A1 A2 ,
A1 A3 , A1 A4 , A2 A3 , A2 A4 , A3 A4 và trọng tâm tam giác chính là tâm
đường tròn O. Đỉnh lần lượt các tam giác cân mới dựng là A12 ,
A13 , A14 , A23 , A24 , A34 . Lấy những điểm đối xứng A012 , A013 , A014 ,
A023 , A024 , A034 qua các cạnh A3 A4 , A2 A4 , A2 A3 , A1 A4 , A1 A3 , A1 A2 .
Chứng minh rằng 6 điểm đối xứng trên nằm trên đường tròn S và
7.3. Bài tập 69

bán kính đường tròn bằng 4 lần khoảng cách từ tâm O đến trọng
tâm tứ giác.

. 7.13. Cho tứ giác ABCD nội tiếp đường tròn O sao cho AOB
\=
\ = 600 . Chứng minh rằng những trung điểm M ,P và N lần
COD
lượt của các đoạn thẳng AO, BC và DO là đỉnh của một tam giác
đều.

. 7.14. Mỗi đỉnh của một tứ giác ngoại tiếp đường tròn và những
điểm tiếp xúc đường tròn với các cạnh tứ giác có chung đỉnh này tạo
ra một tam giác. Chứng minh rằng những trực tâm của những tam
giác như trên tạo thành một hình bình hành và các cạnh hình bình
hành song song với đường nối những điểm tiếp xúc của các cạnh
đối diện của tứ giác.

. 7.15. Cho tứ giác ABCD nội tiếp trong đường tròn. Chứng minh
rằng nếu tiếp tuyến với đường tròn tại các điểm A và C cắt đường
chéo BD thì những tiếp tuyến với đường tròn tại những điểm B và
D cũng phải cắt đường chéo AC.
Chương 8
ĐƯỜNG TRÒN ĐƠN VỊ NỘI TIẾP

8.1. Tọa độ đơn vị mới . . . . . . . . . . . . . . . . . . . . . . . . . . . . . . . . . . . . 70


8.2. Ví dụ . . . . . . . . . . . . . . . . . . . . . . . . . . . . . . . . . . . . . . . . . . . . . . . . . . . 71
8.3. Bài tập. . . . . . . . . . . . . . . . . . . . . . . . . . . . . . . . . . . . . . . . . . . . . . . . . 78

8.1. TỌA ĐỘ ĐƠN VỊ MỚI


Những bài toán ở các phần trước đều giải bằng cách chọn hệ toạ
độ sao cho đường tròn ngoại tiếp tam giác hoặc tứ giác là đường
tròn đơn vị. Với cách đã chọn thì việc tính toán và giải các bài toán
trở nên đơn giản và dễ hiểu. Nhưng có những bài toán về đường
tròn nội tiếp hay đường phân giác, thì cách tốt hơn là chọn chính
đường tròn nội tiếp tam giác hoặc tứ giác làm đơn vị.
Trong phần này ta xét
một loạt thí dụ và bài tập
bằng cách chọn này. Cho tam
giác A1 A2 A3 , ta thường gán
nhãn của các đỉnh tam giác
là a1 ,a2 ,a3 rồi đi tìm những
mối liên hệ giữa chúng theo
dữ kiện của bài toán đã cho.
Hình 8.1.
Trong trường hợp đường tròn
8.2. Ví dụ 71

nội tiếp tam giác A1 A2 A3 tâm J và các điểm tiếp xúc T1 ,T2 ,T3 lần
lượt đối với các cạnh A2 A3 , A3 A1 , A1 A2 , người ta thường chọn
đường tròn nội tiếp tam giác trùng với đường tròn đơn vị và nhãn
t1 ,t2 , t3 là đã biết trên đường tròn, còn các đỉnh của tam giác có
nhãn tính theo công thức ở chương trước

2t2 t3 2t1 t3 2t1 t2


a1 = , a2 = , a3 = ,
t2 + t3 t1 + t3 t1 + t2
ở đây ta đưa thêm vào ký hiệu sau:

δ1 = t1 + t2 + t3 , δ2 = t1 t2 + t2 t3 + t3 t1 , δ3 = t1 t2 t3 .

Người ta cũng tìm được nhãn các điểm đặc biệt của tam giác biểu
diễn theo t1 ,t2 , t3 thông qua các ví dụ sau:

8.2. VÍ DỤ
Ví dụ 8.1. Cho tam giác A1 A2 A3 , đường tròn đơn vị J nội tiếp tam
giác tiếp xúc các cạnh A1 A2 , A2 A3 , A3 A1 lần lượt tại T1 , T2 , T3 . Hãy
tính các đại lượng sau theo t1 , t2 , t3 .
1. Nhãn o của tâm O đường tròn ngoại tiếp tam giác A1 A2 A3 .
2. Nhãn h trực tâm H của tam giác A1 A2 A3 .
3. Nhãn o9 của tâm O9 đường tròn 9 điểm của tam giác A1 A2 A3 .
4. Bán kính R của đường tròn ngoại tiếp tam giác A1 A2 A3 .

Lời giải. 1. Vì |OA1 | = |OA2 | = |OA3 |, suy ra o là nghiệm của hệ


(
(o − a1 )(o − a1 ) = (o − a2 )(o − a2 )
(o − a2 )(o − a2 ) = (o − a3 )(o − a3 )
72 Chương 8. Đường tròn đơn vị nội tiếp

hay là
−o(a1 − a2 ) − o(a1 − a2 ) = a2 a2 − a1 a1 (8.1)
−o(a2 − a3 ) − o(a2 − a3 ) = a3 a3 − a2 a2 (8.2)
Ta có
2 2 2(t1 − t2 )
a1 − a2 = − =
t2 + t3 t3 + t1 (t2 + t3 )(t3 + t1 )
2t2 t3 2t1 t3 2t23 (t2 − t1 )
a1 − a2 = − =
t2 + t3 t3 + t1 (t2 + t3 )(t3 + t1 )
4t1 t3 4t2 t3
a2 a2 − a1 a1 = 2

(t3 + t1 ) (t2 + t3 )2
t1 (t22 + 2t2 t3 + t23 ) − t2 (t23 + 2t1 t3 + t21 )
= 4t3
(t3 + t1 )2 (t2 + t3 )2
t1 t + t1 t3 − t2 t23 − t2 t21
2 2
= 4t3 2
(t3 + t1 )2 (t2 + t3 )2
t1 t2 (t2 − t1 ) − t23 (t2 − t1 ) (t2 − t1 )(t1 t2 − t23 )
= 4t3 = 4t3
(t3 + t1 )2 (t2 + t3 )2 t3 + t1 )2 (t2 + t3 )2
Từ (8.1) ta có
2(t2 − t1 ) 2t23 (t2 − t1 ) (t2 − t1 )(t1 t2 − t23 )
o −o = 4t3
(t3 + t1 )(t2 + t3 ) (t3 + t1 )(t2 + t3 ) t3 + t1 )2 (t2 + t3 )2
Hay là
2t3 (t1 t2 − t23 )
o − t23 o =
(t3 t1 )(t2 + t3 )
Từ (8.2) tương tự có
2t1 (t2 t3 − t21 )
o − t21 o =
(t3 t1 )(t1 + t2 )
Trừ hai đẳng thức sau cùng cho nhau được
2t1 (t2 t3 − t21 ) 2t3 (t1 t2 − t23 )
(t23 − t21 )o = −
(t3 t1 )(t1 + t2 ) (t3 t1 )(t2 + t3 )
8.2. Ví dụ 73

(t2 + t3 )(2δ3 − 2t33 ) − (t1 + t2 )(2δ3 − 2t33 )


=
(t3 + t1 )(t2 + t3 )(t1 + t2 )
2δ3 (t3 − t1 ) + 2t2 (t33 − t31 ) + 2t3 t1 (t23 − t21 )
=
δ1 δ2 − δ3
Khi đó
2δ3 + 2t2 (t23 + t3 t1 + t21 ) + 2t3 t1 (t3 + t1 )
(t3 + t1 )o =
δ1 δ2 − δ3
4δ3 + 2t2 t3 + 2t2 t21 + 2t23 t1 + 2t3 t21
2
=
δ1 δ2 − δ3
t2 t3 (t3 + t1 ) + t3 t1 (t3 + t1 ) + t1 t2 (t1 + t3 )
=2
δ1 δ2 − δ3
từ đó suy ra
2δ2
o=
δ1 δ2 − δ3
δ1 δ2
Do δ2 = , δ1 = ta nhận được
δ3 δ3
δ1
2
2δ2 δ3 2δ1 δ3
o= = =
δ1 δ2 − δ3 δ2 δ1 1 δ1 δ2 − δ3

δ32 δ3

2δ1 δ3
o=
δ1 δ2 − δ3
−−→ −−→ −−→ −−→
2. Do công thức tính tổng vectơ, ta có OA1 + OA2 + OA3 = OH.
Do đó a1 − o + a2 − o + a3 − o = h − o, hay là h = a1 + a2 + a3 − 2o.
h t2 t3 t3 t1 t1 t2 2δ1 δ3
= + + −
2 t2 + t3 t3 + t1 t1 + t2 δ1 δ2 − δ3
t2 t3 (t3 + t1 )(t1 + t2 ) + t3 t1 (t1 + t2 )(t2 + t3 ) + t1 t2 (t2 + t3 )(t3 + t1 )
=
(t1 + t2 )(t2 + t3 )(t3 + t1 )
2δ1 δ3

δ1 δ2 − δ3
74 Chương 8. Đường tròn đơn vị nội tiếp

t2 t3 (t21 + δ2 ) + t3 t1 (t22 + δ2 ) + t1 t2 (t23 + δ2 ) 2δ1 δ3


= −
(t1 + t2 )(t2 + t3 )(t3 + t1 ) δ1 δ2 − δ3
δ1 δ3 + δ2 2 2δ1 δ3 2
δ − δ1 δ3
= − = 2
δ1 δ2 − δ3 δ1 δ2 − δ3 δ1 δ2 − δ3

δ22 − δ1 δ3
h=2
δ1 δ2 − δ3
(h + o)
3. Do O9 là trung điểm của OH, nên e = , hay là
2
δ 2 − δ1 δ3 + δ1 δ3 δ22
e= 2 =
δ1 δ2 − δ3 δ1 δ2 − δ3
2 2
4. R = |o − a| = (o − a)(o − a) = oo + aa − ao − oa
4δ1 δ2 δ3 4 4δ1 δ3 1
= + −
(δ1 δ2 − δ3 )2 (t2 + t3 )(t2 + t3 ) δ1 δ2 − δ3 t2 + t3
R2 δ1 δ2 δ3 4 δ1 δ3
= + −
4 (δ1 δ2 − δ3 )2 (t2 + t3 )2 (δ1 δ2 − δ3 )(t2 + t3 )
t2 t3 δ2

(δ1 δ2 − δ3 )(t2 + t3 )
δ1 δ2 δ3 t2 t3 (δ1 δ2 − δ3 ) − (t2 + t3 )δ1 δ3 − t2 t3 (t2 + t3 )δ2
= 2
+
(δ1 δ2 − δ3 ) (t2 + t3 )(δ1 δ2 − δ3 )
δ1 δ2 δ3 t2 t3 (t1 + t3 )(t1 + t2 )δ1 δ2 − t2 t3 δ2
= +
(δ1 δ2 − δ3 )2 (t2 + t3 )(δ1 δ2 − δ3 )
δ1 δ2 δ3 t1 δ3 − δ1 δ3
= 2
+
(δ1 δ2 − δ3 ) (t2 + t3 )(δ1 δ2 − δ3 )
δ1 δ2 δ3 δ3 (t2 + t3 )
= 2

(δ1 δ2 − δ3 ) (t2 + t3 )(δ1 δ2 − δ3 )
δ1 δ2 δ3 δ3 δ32
= − =
(δ1 δ2 − δ3 )2 δ1 δ2 − δ3 (δ1 δ2 − δ3 )2
Như vậy, 2
4δ32

2 2δ3
R = =
(δ1 δ2 − δ3 )2 δ1 δ2 − δ3
8.2. Ví dụ 75

δ3
Ta sẽ chứng minh số λ = là số thực và dương, thật vậy
δ1 δ2 − δ3
1
δ3 δ3 δ3
λ= = = =λ
δ3 − δ1 δ2 1 δ δ
1 2 δ3 − δ1 δ2
− 2
δ3 δ3
Do đó λ là một số thực. Ta viết λ cách khác
δ3 1 1 1
= = =
δ3 − δ1 δ2 δ2 1 − δ1 δ1 1 − |δ1 |2
1 − δ1
δ3
Chúng ta sẽ chứng minh |δ1 | < 1. Thật vậy, ta nhận thấy các góc
của tam giác T1 T2 T3 luôn luôn nhọn. Suy ra trực tâm nằm trong
tam giác T1 T2 T3 , nhãn của trực tâm chính là δ1 , vậy nó nằm trong
đường tròn ngoại tiếp T1 T2 T3 , đó chính là đường tròn đơn vị, vì thế
|δ1 | < 1. Như vậy λ ≥ 0, từ R2 = 4λ2 , suy ra R = 2λ, nghĩa là
R=
2
1 − δ1 δ2
. J
Ví dụ 8.2. Cho tam giác A1 A2 A3 nội tiếp đường tròn tâm O bán kính
R và đường tròn nội tiếp tam giác A1 A2 A3 , tâm J bán kính r. Gọi
d là khoảng cách giữa hai tâm đường tròn O và J. Chứng minh rằng
d2 = R2 − 2Rr.

Lời giải. Gọi T10 , T20 , T30 là tiếp điểm của các cạnh A2 A3 ,A3 A1 ,A1 A2
và đường tròn nội tiếp tâm J. Chọn hệ toạ độ sao cho tâm đường
tròn đơn vị trùng với tâm của đường tròn nội tiếp. Gọi t1 , t2 , t3 nằm
trên đường tròn đơn vị và đường thẳng JT1 , JT2 , JT3 , do đó nhãn
của T10 , T20 , T30 là t01 = rt1 , t02 = rt2 , t03 = rt3 . Từ ví dụ trước ta có
2δ1 δ3
o= do đó
δ1 δ2 − δ3
2δ1 δ3 2δ3
o= r và R = r
δ1 δ2 − δ3 δ3 − δ1 δ2
76 Chương 8. Đường tròn đơn vị nội tiếp

Vì vậy o = −Rδ1 và o = −Rδ1 . Tính


δ1 δ2 δ3 − δ1 δ2
d2 = |OI|2 = oo = R2 δ1 δ1 = R2 = R2 (1 − )
δ3 δ3
2r
= R2 (1 − ) = R2 − 2Rr.
R

Ví dụ 8.3. Chứng minh rằng đường thẳng nối tâm O của đường tròn
ngoại tiếp với tâm J của đường tròn nội tiếp tam giác A1 A2 A3 là
đường thẳng Euler của tam giác T1 T2 T3 (T1 , T2 , T3 là tiếp điểm của
đường tròn nội tiếp và tam giác).

Lời giải. Tất cả các ký hiệu và chọn hệ toạ độ như các ví dụ trên. Do
đó theo ví dụ 1, ta có
2δ1 δ3
o=
δ1 δ2 − δ3
Vậy,
o 2δ3 2 2 o
= = = =
δ1 δ1 δ2 − δ3 δ2 δ1 δ1 − 1 δ1
δ1 − 1
δ3
mà trực tâm của tam giác T1 T2 T3 có nhãn chính là δ1 . Như vậy, IO
o
và IH cùng nằm trên một đường thẳng (do arg = 0 hoặc π).
δ1
J
Ví dụ 8.4. (Đề thi quốc tế lần thứ 23). Cho tam giác A1 A2 A3 với các
cạnh có độ dài khác nhau. Với mỗi k = 1, 2, 3 ký hiệu Mk là điểm giữa
của các cạnh đối diện với đỉnh Ak , Tk là điểm tiếp xúc của đường tròn
nội tiếp tam giác với cạnh đối diện với đỉnh Ak . Sk là điểm đối xứng
của Tk qua đường phân giác xuất phát từ đỉnh Ak .
Chứng minh rằng , những đường thẳng M1 S1 , M2 S2 , M3 S3 cắt nhau
tại một điểm.
8.2. Ví dụ 77

Lời giải. Chọn hệ toạ độ sao cho A3


đường tròn nội tiếp là đơn vị . Do
T1
cung T2 T3 và T1 S1 song song, từ đó T2
O
suy ra t2 t3 = t1 s1 hay là s1 = t2 t3 t1 . S2 S1
Tương tự s2 = t1 t3 t2 , s3 = t1 t2 t3 . J S3
A1 A2
Khi đó s2 − s3 = t1 (t3 t2 − t2 t3 ). T3
Nhưng số t3 t2 − t2 t3 hoàn toàn ảo,
Hình 8.2.
nghĩa là OT1 vuông góc với S2 S3 .
Từ đó suy ra S2 S3 song song với A2 A3 , mà M2 M3 //A2 A3 vậy
S2 S3 //M2 M3 .
Từ cách chứng minh trên suy ra S1 S2 S3 và M1 M2 M3 có các cạnh
tương ứng song song. Hai tam giác này không bằng nhau, vì đường
tròn đi qua S1 S2 S3 là đường tròn nội tiếp tam giác A1 A2 A3 có bán
R
kính r ≤ (đẳng thức xẩy ra khi tam giác A1 A2 A3 là đều). Mà bán
2
R
kính đường tròn ngoại tiếp tam giác M1 M2 M3 là . Suy ra theo
2
định lý Talet thì M1 M2 M3 và S1 S2 S3 vị tự, nghĩa là M1 S1 ,M2 S2 ,
M3 S3 cắt nhau tại một điểm (đó chính là tâm vị tự). J
Ví dụ 8.5. Các giả thiết như ví dụ cuối cùng. Chứng minh rằng điểm
cắt của ba đường thẳng trong cách chứng minh của ví dụ trên nằm
trên đường tròn nội tiếp tam giác A1 A2 A3 .

Lời giải. Ta gọi z là điểm cắt thứ hai của đường M1 S1 với đường
tròn đơn vị. Do đó có công thức

z + s1 = m1 + zs1 m1
1 2t3 t1 2t1 t2
Nhưng m1 = (a2 + a3 ), a2 = và a3 = do tính chất
2 t1 + t3 t1 + t2
78 Chương 8. Đường tròn đơn vị nội tiếp

của tiếp tuyến với đường tròn. Khi đó thay vào trên ta có
   
t2 t3 t3 t1 t1 t2 t2 t3 1 1
z+ = + +z +
t1 t3 + t1 t1 + t2 t1 t3 + t1 t1 + t2

Biến đổi đẳng thức về dạng


  
t2 t3 1 1 t3 t1 t1 t2 t2 t3
z 1− + = + −
t1 t3 + t1 t1 + t2 t3 + t1 t1 + t2 t1

z(t1 + t2 + t3 )(t21 − t2 t3 ) = (t1 t2 + t2 t3 + t3 t1 )(t21 − t2 t3 )
hoặc
t1 t2 + t2 t3 + t3 t1
z=
t1 + t2 + t3
(vì t21 − t2 t3 6= 0, nếu ngược lại thì T1 T2 T3 là tam giác cân, từ đó suy
ra A1 A2 A3 cũng cân vô lý).
Nhãn z của điểm Z phụ thuộc vào t1 , t2 , t3 có vai trò như nhau,
vậy đường thẳng M2 S2 và M3 S3 cũng đi qua điểm Z (mà Z đã nằm
trên đường tròn nội tiếp tam giác A1 A2 A3 ). J
8.3. BÀI TẬP
. 8.6. Điểm A1 , B1 , C1 lần lượt là điểm tiếp xúc của đường tròn nội
tiếp tam giác ABC với các cạnh BC, CA, AB. Chứng minh rằng
nếu N là giao điểm của OC1 và A1 B1 thì CN đi qua điểm giữa cạnh
AB.

. 8.7. Đường tròn tâm O tiếp xúc với các cạnh BC, CA, AB của
tam giác ABC lần lưowj t tại A1 , B1 , C1 . Chứng minh rằng nếu M
là trung điểm của BC thì đường thẳng OM đi qua trung điểm AA1 .

. 8.8. Cho hai tiếp tuyến P T và P B của một đường tròn cho trước.
Dựng đường kính AB đi qua B và dựng đường vuông góc từ T
8.3. Bài tập 79

xuống AB có chân là H. Chứng minh rằng đường thẳng AP đi qua


điểm giữa của T H.

. 8.9. Chứng minh rằng trong tam giác ABC, đường phân giác của
góc tại đỉnh A, đường trung bình song song với cạnh AC và đường
thẳng nối hai điểm tiếp xúc của đường tròn nội tiếp tam giác ABC
trên cạnh BC và CA, cắt nhau tại một điểm .

. 8.10. Đường tròn nội tiếp tam giác ABC tiếp xúc với cạnh AB và
AC lần lượt tại D và E. Chứng minh rằng những giao điểm N và
M của đường phân giác trong đỉnh B và C của tam giác ABC với
đường thẳng DE và đỉnh B, C cùng nằm trên một đường tròn.

. 8.11. (Định lý Newton). Trung điểm của các đường chéo trong
một tứ giác ngoại tiếp đường tròn và tâm đường tròn đó thẳng hàng.

. 8.12. Cho tứ giác ngoại tiếp đường tròn (O). Chứng minh rằng
những đường nối những điểm tiếp xúc của các cạnh đối diện và các
đường chéo tứ giác cắt nhau tại một điểm.

. 8.13. Tam giác ABC nội tiếp trong đường tròn tâm O. Tại các
điểm A, B, C kẻ những tiếp tuyến với đường tròn, chúng cắt nhau
tạo thành tam giác A0 B0 C0 (điểm A0 là điểm cắt của hai đường tiếp
tuyến tại B và C,...). Lấy P, Q, R tương ứng là những điểm cắt của
BC và B0 C0 , CA và C0 A0 , AB và A0 B0 . Chứng minh rằng những
điểm P, Q, R nằm trên một đường thẳng.

. 8.14. Cho I là tâm đường tròn nội tiếp tam giác A1 A2 A3 . Những
tia A1 I, A2 I, A3 I cắt đường tròn ngoại tiếp tam giác A1 A2 A3 tại các
điểm tương ứng B1 , B2 , B3 . Chứng minh rằng A1 B1 và B2 B3 vuông
góc với nhau.
80 Chương 8. Đường tròn đơn vị nội tiếp

. 8.15. Cho B1 và C1 là những điểm cắt của các đường phân giác
xuất phát từ B và C của tam giác ABC với đưowh ng tròn ngoại
tiếp tam giác này. Nếu O và I là tâm đường tròn ngoại tiếp và nội
−−→ −−→ −−→
tiếp, đồng thời có ON = OB1 + OC1 , chứng minh rằng IN và BC
vuông góc với nhau.
Chương 9
TAM GIÁC ĐỒNG DẠNG

9.1. Quan hệ đồng dạng của hai tam giác . . . . . . . . . . . . . . 81


9.2. Ví dụ . . . . . . . . . . . . . . . . . . . . . . . . . . . . . . . . . . . . . . . . . . . . . . . . . . . 82
9.3. Bài tập. . . . . . . . . . . . . . . . . . . . . . . . . . . . . . . . . . . . . . . . . . . . . . . . . 85

9.1. QUAN HỆ ĐỒNG DẠNG CỦA HAI TAM GIÁC


Cho hai tam giác đồng dạng A1 A2 A3 và B1 B2 B3 cùng hướng
nào đó (các cạnh tương ứng của hai tam giác có cùng hướng với
trục tọa độ).
Khi đó những góc A c1 ,A
c2 ,A
c3 A3 A3
tương ứng bằng các góc B c1 , B
c2 ,
A 1 A3 B 1 B3
B
c3 và tỷ số và là bằng
A1 A2 B1 B2
nhau. Nếu ta ký hiệu số phức A1
A1 A3 A1
k = (cos A1 + i sin A1 ) nghĩa B3 B3
A1 A2 A2

là A2
B1
B1 B3 B1
k= (cos B1 + i sin B1 )
B1 B2 B2
B2
hoặc nói cách khác
a3 − a1 b3 − b1 Hình 9.1.
k= và k = .
a2 − a1 b2 − b1
Do đó, nếu tam giác A1 A2 A3 và B1 B2 B3 đồng dạng và cùng
82 Chương 9. Tam giác đồng dạng

hướng, thì
a3 − a1 b3 − b1
=
a2 − a1 b2 − b1
Ngược lại, nếu đẳng thức sau cùng là đúng, thì hai tam giác
A1 A2 A3 và B1 B2 B3 đồng dạng và cùng hướng, bởi vì
A1 A3 B1 B3
= và A
c1 = B c1 .
A1 A2 B1 B2
Nếu hai tam giác A1 A2 A3 và B1 B2 B3 đồng dạng, nhưng ngược
a3 − a1 b3 − b1
hướng nhau thì tỷ số và có cùng môđun, còn argu-
a2 − a1 b2 − b1
men của chúng bằng nhau theo giá trị tuyệt đối, nhưng chỉ hướng
khác nhau. Hay là, mỗi tỷ số trên của tam giác này bằng số phức
liên hợp của tỷ số kia.

9.2. VÍ DỤ
Ví dụ 9.1. Nếu B1 , B2 , B3 là những điểm của mặt phẳng A1 A2 A3 ,
sao cho những tam giác A1 A2 B3 , A2 A3 B1 , A3 A1 B2 là đồng dạng và
cùng hướng. Chứng minh rằng những trọng tâm của các tam giác
A1 A2 A3 và B1 B2 B3 trùng nhau.

Lời giải. Từ giả thiết suy ra tồn tại số phức k sao cho
b3 − a1 b1 − a2 b2 − a3
= = =k
a2 − a1 a3 − a2 a1 − a3
suy ra

b3 = a1 + k(a2 − a1 ),
b1 = a2 + k(a3 − a2 ),
b2 = a3 + k(a1 − a3 ).

Cộng hai vế của các đẳng thức, ta có b1 + b2 + b3 = a1 + a2 + a3 . Từ


đó suy ra trọng tâm của A1 A2 A3 và B1 B2 B3 trùng nhau. J
9.2. Ví dụ 83

Ví dụ 9.2. Cho hai tam giác ABC và P QR, một điểm F trong tam
giác P QR. Những điểm A1 ,B1 ,C1 xác định sao cho những cặp tam
giác ABC1 và QP F , BCA1 và RQF , CAB1 và P RF đồng dạng và
cùng hướng. Chứng minh rằng tam giác A1 B1 C1 và P QR đồng dạng
và cùng hướng.

Lời giải. Chọn điểm gốc toạ độ là F , f = 0. Từ sự đồng dạng của


a − c1 q ap − bq
tam giác ABC1 và QP F ta có = từ đó suy ra c1 = .
b − c1 p p−q
bq − cr cr − ap
Tương tự ta có a1 = và b1 = suy ra
q−r r−p
cr − ap bq − cr

b1 − a1 r−p q−r q−p
= =
c1 − a1 ap − bq bq − cr r−p

p−q q−r
Đẳng thức sau cùng nói lên tam giác A1 B1 C1 và P QR đồng dạng
và cùng hướng. J
Ví dụ 9.3. Cho tam giác ABC vuông góc tại C. Chứng minh rằng
trung tuyến AM và CN tương ứng của tam giác ADC và DBC, ở
đây D là chân đường cao hạ từ đỉnh C, vuông góc với nhau.

Lời giải. Chúng ta biết rằng DBC và


DCA đồng dạng, chọn D là điểm gốc
của hệ toạ độ, d = 0. Khi đó c = bi tg α,
a = ci tg α và
1 1
n − c = b − bi tg α = b( − i tg α), Hình 9.2.
2 2
1 1
m − a = c − ci tg α = c( − i tg α).
2 2
84 Chương 9. Tam giác đồng dạng

từ đây suy ra các đường trung tuyến CN và AM vuông góc với


nhau. J
Ví dụ 9.4. Tam giác B1 B2 B3 nhận được từ tam giác A1 A2 A3 bằng
cách quay một góc φ 6= π quanh tâm của đường tròn ngoại tiếp.
Những đoạn B1 B2 , A1 A2 cắt nhau tại C3 ; B2 B3 , A2 A3 cắt nhau
tại C1 và B3 B1 , A3 A1 cắt nhau tại C2 . Chứng minh rằng tam giác
C1 C2 C3 đồng dạng với tam giác A1 A2 A3

Lời giải. Chọn hệ toạ độ sao cho đường tròn ngoại tiếp tam giác
A1 A2 A3 là đường tròn đơn vị. Theo
công thức tính giao điểm B1 B2 và
A1 A2
(a1 + a2 )b1 b2 − (b1 + b2 )a1 a2
c3 =
b1 b2 − a1 a2
vì b1 = ka1 và b1 = ka2 ,
với k = cos φ + i sin φ, thì Hình 9.3.

(a1 + a2 )k 2 a1 a2 − (a1 + a2 )ka1 a2 k


c3 = 2
= (a1 + a2 )
k a1 a2 − a1 a2 k+1
Tương tự
k k
c1 = (a2 + a3 ) và c2 = (a3 + a1 ).
k+1 k+1
Khi đó
k k
(a1 + a2 ) − (a2 + a3 )
c3 − c1
c2 − c1
= k+1
k
k+1
k
=
a1 − a3
a1 − a2
. J
(a3 + a1 ) − (a2 + a3 )
k+1 k+1
Ví dụ 9.5. Trong mặt phẳng tam giác ABC tồn tại hai điểm U và
V sao cho những tam giác AU V , V BU , U V C là đồng dạng và cùng
9.3. Bài tập 85

hướng. Chứng minh rằng những tam giác này đồng dạng với tam giác
ABC.

Lời giải. Từ điều kiện của bài toán suy ra tồn tại một số phức λ, sao
cho
u−a b−v v−u
= = =λ
v−a u−v c−u
u − λv 1
suy ra a = ,b = v + λ(u − v) và c = u + (v − u). Do đó
1−λ λ
u − λv
b−a v + λ(u − v) −
= 1−λ
c−a 1 u − λv
u + (v − u) −
λ 1−λ

(1 − λ + λ2 )v + (λ − λ2 − 1)u
(λ − λ2 − 1)u + (1 − λ + λ2 )v
= λ. J
9.3. BÀI TẬP
. 9.6. Cho hai tam giác đồng dạng ABC và A1 B1 C1 . Những điểm
A0 , B0 ,C0 chia các đoạn thẳng AA1 , BB1 và CC1 với tỷ số bằng
nhau. Chứng minh rằng A0 B0 C0 đồng dạng và cùng hướng với
những tam giác đã cho.

. 9.7. Hai tam giác ABC và A1 B1 C1 đồng dạng và cùng hướng.


−−−→ −−−→ −−−→
Chứng minh rằng, nếu vectơ M A0 ,M B0 ,M C0 tương ứng bằng vectơ
−−→ −−→ −−→
AA1 , BB1 , CC1 , thì tam giác A0 B0 C0 đồng dạng và cùng hướng với
các tam giác đã cho.

. 9.8. Trong mặt phẳng cho hai tam giác đồng dạng và cùng hướng
A1 A2 A3 và B1 B2 B3 . Ta chọn C1 , C2 , C3 sao cho những tam giác
A1 B1 C1 , A2 B2 C2 , A3 B3 C3 đồng dạng và cùng hướng. Chứng minh
rằng tam giác C1 C2 C3 đồng dạng với tam giác đã cho.
86 Chương 9. Tam giác đồng dạng

. 9.9. Cho tam giác A1 A2 A3 với trực tâm H. Cho G1 , G2 , G3 và G


là trọng tâm của các tam giác A2 A3 H, A3 A1 H, A1 A2 H, A1 A2 A3 .
Chứng minh rằng tam giác G1 G2 G3 và A1 A2 A3 đồng dạng với nhau
và G là trực tâm của tam giác G1 G2 G3 .

. 9.10. Cho tam giác ABC và điểm M sao cho C là trung điểm
của AM . Tam giác ACN xây dựng ra phía ngoài của tam giác ABC
sao cho CAN
\ = BAC \ và ACN
\ = ABC.\ Cho ABCL và N M BS là
những hình bình hành. Chứng minh rằng tam giác ABC và BLS
đồng dạng.

. 9.11. Những tam giác ABC và DEC vuông tại C, đồng dạng và
cùng hướng. Chứng minh rằng AD vuông góc với BE.

. 9.12. Những tam giác vuông ABC và AB1 C1 đồng dạng và ngược
chiều nhau, BCA
\ = B\ 0 \
1 C1 A = 90 ,CAB = C1 AB1 . Chứng minh
\
rằng nếu M là giao điểm của BC1 và CB1 , đường thẳng AM và
CC1 tồn tại, thì chúng vuông góc với nhau.

. 9.13. Cho ngũ giác lồi ABCDE, mà AB = BC = CA và


CD = DE = EC. Cho S là trọng tâm của tam giác ABC, còn
M và N lần lượt là trung điểm của BD và AE. Chứng minh rằng
tam giác SM E và SN D là đồng dạng.

. 9.14. Những đường thẳng đi qua đỉnh của tam giác đã cho sao
cho chúng tạo với cạnh đối diện những góc bằng nhau. Chứng minh
rằng tam giác tạo bởi những đường thẳng này đồng dạng với tam
giác đã cho và tâm của đường tròn ngoại tiếp của tam giác này là
trực tâm của tam giác đã cho.

. 9.15. Những điểm B,C và P nằm trên một đường tròn tâm O,
tiếp tuyến tại B và C với đường tròn cắt nhau tại A. Đường thẳng
9.3. Bài tập 87

vuông góc với AP tại điểm P cắt các đường thẳng OB và OC tương
ứng tại D và E. Từ D và E hạ đường vuông góc xuống AO và chân
đường vuông góc được ký hiệu là M và N . Chứng minh rằng a)
Các tam giác OAD và OEA đồng dạng . b) Những điểm M và N là
nghịch đảo của đường tròn đã cho.
Chương 10
ĐA GIÁC ĐỀU

10.1. Nhãn của đỉnh các đa giác đều . . . . . . . . . . . . . . . . . . . 88


10.2. Ví dụ . . . . . . . . . . . . . . . . . . . . . . . . . . . . . . . . . . . . . . . . . . . . . . . . . 89
10.3. Bài tập . . . . . . . . . . . . . . . . . . . . . . . . . . . . . . . . . . . . . . . . . . . . . . . 93

10.1. NHÃN CỦA ĐỈNH CÁC ĐA GIÁC ĐỀU


Trong chương 1 đa giác đều n đỉnh nằm trên đường tròn đơn
vị như là biểu diễn hình học những nghiệm của phương trình
xn = 1 trong số phức. Vậy mỗi đỉnh của n-đa giác đều có nhãn
2kπ 2kπ
wk = cos + i sin , k = 0, 1, 2, ..., n − 1. Ta biết rằng wk = w1k .
n n
Điều này làm công việc giải bài toán rất thuận tiện khi chọn đường
tròn đó làm đơn vị. Nhưng trong thực tế một bài toán có tham gia
một lúc nhiều đa giác đều, nên gặp khó khăn không phải là ít. Ngay
ở những ví dụ đầu tiên của tài liệu này cũng đã đề cập đến bài toán
có nhiều đa giác đều. Mỗi loại đa giác đều như tam giác đều, hình
vuông,... có một đặc trưng riêng, nên ta phải đưa ra các tiêu chuẩn
đặc trưng cho một loại đa giác đều cụ thể với hệ toạ độ bất kì. Vì
các đa giác đều nội tiếp trong đường tròn đi qua các đỉnh, nên trong
khi giải các bài tập ta luôn lấy các đường tròn này làm đơn vị, do đó
việc tính nhãn của các đỉnh trở nên đơn giản và các công thức biến
đổi được nhẹ nhàng.
10.2. Ví dụ 89

Thông dụng nhất là tam giác đều, cụ thể cho ba điểm A1 , A2 ,A3
là đỉnh của tam giác đều định hướng khi và chỉ khi A1 A2 = A1 A3 và
π
góc định hướng quay A1 A2 quanh A1 đến vị trí A1 A3 là , nghĩa là
6 √
π π 1 3
a3 −a1 = w(a2 −a1 ), ở đây ta ký hiệu w = cos +i sin = +i .
3 3 2 2
Ta viết lại a3 = a1 + w(a2 − a1 ) là nhãn của đỉnh thứ ba trong tam
giác đều định hướng dương A1 A2 A3 xây dựng trên cạnh A1 A2 . Như
ta đã biết w là nghiệm của phương trình w2 − w + 1 = 0 và phương
trình w3 = −1. Những ví dụ sau chỉ ra thế mạnh của phương pháp
số phức trong trường hợp các đa giác đều.

10.2. VÍ DỤ
Ví dụ 10.1. Trên các cạnh của một tam giác bất kỳ ABC về phía
ngoài dựng những tam giác đều ABC 0 ,BCA0 và CAB 0 . Chứng minh
rằng trọng tâm C1 , B1 và A1 của những tam giác mới dựng là đỉnh
của một tam giác đều.

Lời giải. Theo công thức tính đỉnh thứ ba theo hai đỉnh kia
c0 = b + w(a − b)
b0 = a + w(c − a)
a0 = c + w(b − c)
Do đó
1
c1 = (a + b + c0 )
3
1
= [(1 + w)a + (2 − w)b]
3
Tương tự ta có Hình 10.1.
1
b1 = [(1 + w)c + (2 − w)a]
3
90 Chương 10. Đa giác đều

1
a1 = [(1 + w)b + (2 − w)c]
3

Để biết tam giác A1 B1 C1 có phải là đều không ta phải kiểm tra


c1 = a1 + w(b1 − a1 ) ?. Chú ý w2 = 1, kiểm tra trực tiếp ta có đẳng
thức phải tìm. J
Chú ý : Kết luận còn đúng khi các tam giác dựng vào phía trong
của tam giác ABC .

Ví dụ 10.2. Trên cạnh của một tam giác ABC dựng những tam giác
đều BCA0 , ACB 0 ,ABC 0 sao cho A0 và A, B 0 và B nằm về hai phía
khác nhau đối với đường thẳng AB. Chứng minh rằng nếu điểm M
là trọng tâm của tam giác ABC 0 , thì tam giác A0 M B 0 là cân và góc ở

đỉnh M bằng .
3

Lời giải. Theo giả thiết ta có


a0 = c + w(b − c),
b0 = a + w(c − a),
c0 = b + w(a − b).
Từ đẳng thức sau cùng ta có
1
m = (a + b + c0 )
3
1
= [(2 − w)a + (1 + w)b]. Hình 10.2.
3
Khi đó
1
b0 − m = a + w(c − a) − [(2 − w)a + (1 + w)b]
3
1
= [(1 − 2w)a − (1 + w)b + 3wc]
3
0 1
a − m = c + w(b − c) − [(2 − w)a + (1 + w)b]
3
10.2. Ví dụ 91

1
= [(w − 2)a − (1 − 2w)b + 3(1 − w)c].
3
Vì (w − 2)w2 = 1 − 2w,(1 − 2w)w2 = −(1 + w) và (1 − w)w2 = w
suy ra b0 − m = w2 (a0 − m). Điều này chỉ ra rằng những đoạn A0 M
và B 0 M có độ dài bằng nhau và góc ở đỉnh là

3
. J
Ví dụ 10.3. Cho tam giác ABC . Hãy dựng tam giác A0 B0 C0 sao cho
A0 B0 C, B0 C0 A và C0 A0 B là những tam giác đều cùng hướng dương.

Lời giải. Giả sử tam giác A0 B0 C0 đã được dựng theo tính chất đã
cho, A0 B0 C, B0 C0 A và C0 A0 B là những tam giác đều cùng hướng
dương. Khi đó

a = b0 + w(c0 − b0 ), b = c0 + w(a0 − c0 ), c = a0 + w(b0 − a0 ).

Giải hệ phương trình đối với a0 , b0 , c0 ta nhận được


1
a0 = [a + b + w(c − b)],
2
1
b0 = [b + c + w(a − c)],
2
1
c0 = [c + a + w(b − a)].
2
Đặt

a0 = b + w(c − b), b0 = c + w(a − c), c0 = a + w(b − a).

Từ đây ta thấy rằng A0 ,B 0 ,C 0 là đỉnh của những tam giác BCA0 ,


CAB 0 , ABC 0 đều cùng hướng dương.
Như vậy, ta có cách dựng như sau: Chúng ta dựng những đỉnh
A0 ,B 0 ,C 0
như đỉnh thứ ba của những tam giác đều hướng dương
trên các cạnh tam giác đã cho. Còn các điểm A0 ,B0 ,C0 là những
điểm giữa của những đoạn thẳng AA0 ,BB 0 và CC 0 . J
92 Chương 10. Đa giác đều

Ví dụ 10.4. Cho M1 ,M2 và M3 là 3 đỉnh của một tam giác đều định
hướng dương và M là điểm bất kỳ trên mặt phẳng của tam giác .

Chúng ta quay điểm M một góc theo chiều dương quanh điểm
3

M1 ; điểm M 0 nhận được lại quay tiếp góc quanh điểm M3 ; nhận
3

đuw ợc M 00 lại quay tiếp một góc quanh điểm M2 . Điểm cuối cùng
3
nhận được ký hiệu là M 000 . Chứng minh rằng M 000 trùng với M .

Lời giải. Chọn hệ toạ độ sao cho điểm gốc trùng với trọng tâm
của tam giác M1 M2 M3 và m1 = 1, m2 = w, m3 = w2 với
2π 2π
w = cos + i sin . Khi đó
3 3
m0 − 1 = w(m − 1),
m00 − w2 = (m0 − w2 ),
m000 − w = w(m00 − w)

Sau khi thế lần lượt m0 và m00 và dùng w3 = 1 và w2 + w + 1 = 0


chúng ta tìm được m000 = m, nghĩa là M 000 trùng với M . J
Ví dụ 10.5. (Đề thi toán quốc tế lần thứ 27) Trong mặt phẳng cho
tam giác A1 A2 A3 và điểm P0 . Với mọi s ≥ 4 ta đặt As = As−3 . Dựng
dãy điểm P0 ,P1 ,P2 ,... sao cho điểm Pk+1 là ảnh của Pk với phép quay
tâm Ak+1 (k = 0, 1, 2, 3, ...) một góc 1200 theo chiều kim đồng hồ.
Chứng minh rằng nếu P1986 = P0 , thì tam giác A1 A2 A3 là đều.

Lời giải. Vì với mỗi k ≥ 0, tam giác Ak+1 Pk Pk+1 là cân với góc ở
đỉnh là 1200 , ta có
pk+1 − ak+1 = τ (pk − ak+1 ),
ở đây τ = cos 1200 + i sin 1200 . Đặc biệt với
p1 = a1 + τ (p0 − a1 ), p2 = a2 + τ (p1 − a2 ), p3 = a3 + τ (p2 − a3 )
10.3. Bài tập 93

Do đó
p2 = a2 − τ a2 + τ [a1 + τ (p0 − a1 )] = (1 − τ )a2 + τ (1 − τ )a1 + τ 2 p0

p3 = a3 − τ a3 + τ [(1 − τ )a2 + τ (1 − τ )a1 + τ 2 p0 ]
= (1 − τ )(a3 + τ a2 + τ 2 a1 ) + τ 3 p0
Nhưng τ 3 = 1, cuối cùng ta nhận được
p3 = (1 − τ )(a3 + τ a2 + τ 2 a1 ) + p0 .
Rõ ràng, số p0 ,p3 ,p6 ,... tạo thành cấp số cộng với số đầu tiên là p0 và
số dư là (1 − τ )(a3 + τ a2 + τ 2 a1 ). Do đó
P1986 = P3.662 = 662(1 − τ )(a3 + τ a2 + τ 2 a1 ) + p0 ,
vì P1986 = P0 suy ra (1 − τ )(a3 + τ a2 + τ 2 a1 ) + p0 = 0, hay là
(a3 + τ a2 + τ 2 a1 ) = 0. Nhưng τ = w2 = w − 1 và τ 2 = w4 = w3 .w =
−w với w = cos 600 + i sin 600 . Nghĩa là (a3 + (w − 1)a2 − wa1 ) = 0,
hoặc là a3 = a2 + w(a1 − a2 ), điều này chỉ ra rằng A1 A2 A3 là tam
giác đều. J
10.3. BÀI TẬP
. 10.6. Qua điểm P trên đường tròn ngoại tiếp tam giác đều ABC
dựng những đường thẳng song song với các cạnh BC,CA và AB
lần lượt cắt các cạnh CA,AB, BC tại M ,N ,Q. Chứng minh rằng
M ,N ,Q nằm trên một đường thẳng.

. 10.7. Trong mặt phẳng cho hai tam giác đều cùng hướng A1 A2 A3
và B1 B2 B3 . Từ một điểm bất kỳ O lấy những đoạn thẳng OC1 ,OC2 ,
−−→ −−−→ −−→ −−−→ −−→ −−−→
OC3 sao cho OC1 = A1 B1 ,OC2 = A2 B2 , OC3 = A3 B3 . Chứng minh
rằng C1 C2 C3 là tam giác đều.
94 Chương 10. Đa giác đều

. 10.8. Trong mặt phẳng ngũ giác đều A1 A2 A3 A4 A5 cho hai điểm
M , N . Các cặp (M1 , N1 ),(M2 , N2 ),...,(M5 , N5 ) là hình chiếu của
M ,N xuống những đoạn thẳng tương ứng A1 A2 ,A2 A3 ,A3 A4 ,A4 A5 ,
A5 A1 . Chứng minh rằng
−−−→ −−−→ −−−→ −−−→ −−−→ 5 −−→
M1 N1 + M2 N2 + M3 N3 + M4 N4 + M5 N5 = M N .
2
. 10.9. (Đề thi quốc tế lần thứ 19) Trong hình vuông ABCD dựng
những tam giác đều ABK,BCL,CDM ,DAN . Chứng minh rằng
trung điểm của 4 đoạn thẳng KL,LM ,M N ,N K và trung điểm của
8 đoạn thẳng AK,BK,BL, CL,CM ,DM ,DN ,AN là đỉnh của đa
giác đều 12 cạnh.

. 10.10. Cho ba tam giác đều cùng hướng A1 B1 C1


,A2 B2 C2 ,A3 B3 C3 và tam giác A1 A2 A3 cũng đều và cùng hướng với
những tam giác đã cho. Chứng minh rằng trung điểm của các đoạn
thẳng C1 B2 ,C2 B3 ,C3 B1 là đỉnh của một tam giác đều cùng hướng.

. 10.11. Cho hai tam giác đều cùng hướng A1 A2 A3 và B1 B2 B3 .


Trên các đoạn thẳng A1 B1 ,A2 B2 ,A3 B3 dựng những tam giác đều
A1 B1 C1 , A2 B2 C2 và A3 B3 C3 cùng hướng với những tam giác đã
cho. Chứng minh rằng tam giác C1 C2 C3 cũng đều và đồng hướng.

. 10.12. Những góc của tam giác ABC tạo thành cấp số nhân với
hệ số nhân 2. Chứng minh rằng trung điểm của các cạnh và chân
đuw ờng cao tạo thành lục giác đều.

. 10.13. Trên các cạnh A1 A3 và A2 A3 của tam giác A1 A2 A3 dựng


ra phía ngoài những tam giác đều A3 A2 B1 và A1 A3 B2 . Cho M1 và
M2 là các trọng tâm tương ứng của các tam giác vừa dựng, M là
trung điểm của A1 A2 . Chứng minh rằng |M M1 | ≥ |M M2 | khi và chỉ
khi |A2 A3 | ≥ |A1 A3 |.
10.3. Bài tập 95

. 10.14. (Đề thi Olympic toán quốc tế lần thứ 17) Trên cạnh của
một tam giác bất kỳ ABC dựng ra phía ngoài những tam giác BQC,
CP A và ARP , sao cho QBC
\ = CAP [ = 450 , BQC\ = P[ CA = 300 ,
ABR
[ = BAR[ = 150 . Chứng minh rằng P QR là tam giác vuông cân.
Chương 11
DIỆN TÍCH ĐA GIÁC

11.1. Công thức tính diện tích . . . . . . . . . . . . . . . . . . . . . . . . . . . . 96


11.2. Ví dụ . . . . . . . . . . . . . . . . . . . . . . . . . . . . . . . . . . . . . . . . . . . . . . . . . 98
11.3. Bài tập . . . . . . . . . . . . . . . . . . . . . . . . . . . . . . . . . . . . . . . . . . . . . . 102

11.1. CÔNG THỨC TÍNH DIỆN TÍCH


Cho tam giác OAB
định hướng dương. Nếu
chọn toạ độ vuông góc
có gốc tại O, thì diện
tích của tam giác là
1
Ra Rb sin(β − α)
2
với Ra = |a|,Rb = |b|
và α = arg a,β = arg b. Hình 11.1.
Ta thấy rằng
ba = Ra Rb [cos (β − α) + i sin (β − α)] suy ra diện tích của tam giác
1
OAB bằng Im(ba). Tổng quát hơn ta chứng minh định lý sau:
2
11.1. Công thức tính diện tích 97

Định lý. Cho A1 ,A2 ,...,An là đỉnh của một đa giác lồi trong mặt
phẳng phức và S ký hiệu là diện tích định hướng của đa giác . Khi đó
n
!
1 X
S = Im ak+1 ak
2
k=1

(chú ý đỉnh của đa giác đánh số theo vòng tròn, nghĩa là An+1 = a1 ,
An+2 = A2 ,...)
Chứng minh. Chúng ta sẽ chứng minh đẳng thức sau
" n # n
X X
Im (ak+1 + p)(ak + p) = Im( ak+1 ak ),
k=1 k=1

với p là một số phức bất kỳ.


Thật vậy, ta có
n
X n
X n
X n
X
(ak+1 + p)(ak + p) = ak+1 ak + p ak + p ak+1 + n|p|2 ,
k=1 k=1 k=1 k=1
Xn n
X
nhưng những số n|p|2 và p ak + p ak+1 là số thực, suy ra
k=1 k=1
n n
" # !
X X
Im (ak+1 + p)(ak + p) = Im ak+1 ak ,
k=1 k=1

điều này nghĩa là biểu thức không thay đổi, nếu đặt đỉnh của đa
giác tịnh tiến đi một số. Như vậy ta có thể giả thiết rằng điểm
gốc của hệ toạ độ nằm trong đa giác lồi. Khi đó nếu những đỉnh
Ak (k = 1, 2, 3, ...) có nhãn ak = Rk (cos φk + i sin φk ), thì ta có
n n
!
1 X 1 X
Im ak+1 ak = Im( Rk+1 Rk [cos(φk+1 − φk ) + i sin(φk+1 − φk )])
2 2
k=1 k=1
n
1X
= Rk+1 Rk sin(φk+1 − φk ).
2
k=1
98 Chương 11. Diện tích đa giác
n
1X
Nhưng số Rk+1 Rk sin(φk+1 − φk ) là diện tích định hướng của
2
k=1
n
1 X
tam giác OAk Ak+1 . Do đó số Im( ak+1 ak ) bằng tổng diện tích
2
k=1
định hướng những tam giác OA1 A2 ,...,OAn A1 . Nhưng đa giác là lồi
suy ra công thức của định lý là đúng.
Đặc biệt cho tam giác A1 A2 A3 ta có

1
S = Im(a2 a1 + a3 a2 + a1 a3 )
2

và S = 0 khi và chỉ khi ba điểm thẳng hàng. J


11.2. Ví dụ
Ví dụ 11.1. Cho một điểm P trên đường tròn tâm O ngoại tiếp tam
giác ABC. A1 , B1 , C1 lần lượt là những điểm đối xứng của P qua
OA, OB, OC. Chứng minh rằng diện tích tam giác A1 B1 C1 không
phụ thuộc vào việc chọn điểm P trên đường tròn.

Lời giải. Chọn hệ toạ độ gốc tại O.


Ký hiệu P 0 là điểm giữa của P C1 và
trên đường thẳng OC
1 1
p0 = (c − c + p − c2 p) = (p − c2 p).
2 2
1
p = (c1 + p) suy ra c1 = 2p0 − p
0
2
hay là c1 = −c2 p. Tương tự ta cũng
có a1 = −a2 p,b1 = −b2 p. Theo công
Hình 11.2.
thức trong định lý
11.2. Ví dụ 99

1 2
SA1 B1 C1 = Im(a2 pb p + b2 pc2 p + c2 pa2 p)
2
1
= Im((ab)2 + (bc)2 + (ca)2 )
2
Rõ ràng SA1 B1 C1 không phụ thuộc vào nhãn của P . J
Ví dụ 11.2. Hai cạnh đối diện AD và BC của tứ giác ABCD cắt
nhau tại O. Điểm P và S lần lượt là trung điểm của đường chéo BD
và AC. Chứng minh rằng diện tích của ABCD bằng 4 lần diện tích
của tam giác OSP .

Lời giải. Chọn O là gốc của hệ toạ độ. O


Vì O, A, D cùng nằm trên đường thẳng
nên d = λa, với λ là một số thực, cũng
như vậy c = µb, với µ là một số thực. C
D
Theo công thức tính diện tích của tứ
P
giác ta có S
1 A
SABCD = Im(ba + cb + dc + ad) B
2
1 Hình 11.3.
= Im(ba + dc)
2
vì các số cb và ad là số thực.
Mặt khác, diện tích của tam giác OSP là
1 b+d a+c 1 1
SOSP = Im( + ) = . Im(ba + dc)
2 2 2 4 2
do bc và da là các số thực. Do đó SABCD = 4SOSP . J
tích của tam giác A1 A2 A3 .
100 Chương 11. Diện tích đa giác

Ví dụ 11.3. Cho tam giác A1 A2 A3 và


điểm O. Ký hiệu M1 , M2 , M3 là những
trọng tâm của các tam giác OA2 A3 ,
OA3 A1 , OA1 A2 . Chứng minh rằng diện
1
tích của tam giác M1 M2 M3 bằng diện
9
Lời giải. Chọn hệ toạ độ có gốc tại O.
Hình 11.4.
Khi đó
1 1 1
m1 = (a2 + a3 ), m2 = (a3 + a1 ), m3 = (a1 + a2 )
3 3 3
Ta có
1
S M1 M2 M3 = Im(m2 m1 + m3 m2 + m1 m3 )
 2 
1 a3 + a1 a2 + a3 a1 + a2 a3 + a1 a2 + a3 a1 + a2
= Im . + . + .
2 3 3 3 3 3 3
1 1
= [ Im(a3 a2 + a1 a3 + a2 a1 )
9 2
1
+ Im(a1 a2 + a1 a3 + a2 a3 + a2 a1 + a3 a1 + a3 a2 )
2
1
+ Im(|a1 |2 + |a2 |2 + |a3 |2 )]
2
Nhưng những số a1 a2 + a1 a3 + a2 a3 + a2 a1 + a3 a1 + a3 a2 và
|a1 |2 + |a2 |2 + |a3 |2 là các số thực. Vậy
SM1 M2 M3 =
11
92
1
Im(a3 a2 + a1 a3 + a2 a1 ) = SA1 A2 A3
9
J
Ví dụ 11.4. Cho B1 , B2 , B3 , B4 , B5 , B6 lần lượt là trung điểm của
các cạnh A1 A3 , A2 A4 , A3 A5 , A4 A6 , A5 A1 , A6 A2 của một lục giác
lồi A1 A2 A3 A4 A5 A6 . Chứng minh rằng nếu lục giác B1 B2 B3 B4 B5 B6
là lồi, thì diện tích của nó nhỏ hơn bốn lần diện tích lục giác
A1 A2 A3 A4 A5 A6 .
11.2. Ví dụ 101

Lời giải. Ký hiệu S và S1 là diện tích của lục giác A1 A2 A3 A4 A5 A6


và B1 B2 B3 B4 B5 B6 . Khi đó
6 6
! !
1 X 1 X ak+1 + ak+3 ak + ak+2
S1 = Im bk+1 bk = Im .
2 2 2 2
k=1 k=1
6 6
! !
11 X 11 X
= Im ak+1 ak + Im (ak+1 ak+2 + ak+3 ak+2 )
42 42
k=1 k=1
6
!
11 X
+ Im ak+3 ak .
42
k=1

6
!
11 X
Dễ thấy, số Im (ak+1 ak+2 + ak+3 ak+2 ) và
42
k=1
6
J
!
11 X 1
Im ak+3 ak là những số thực. Do đó S1 = S.
42 4
k=1

Ví dụ 11.5. Cho P1 , P2 , P3 là trung điểm của những đường chéo


A1 A4 , A2 A5 , A3 A6 của lục giác lồi A1 A2 A3 A4 A5 A6 . Nếu với S1 , S2
và S ký hiệu là diện tích của lục giác với đỉnh là những trung điểm của
3
lục giác đã cho và diện tích của A1 A2 A3 A4 A5 A6 , thì S1 + S2 = S.
4

Lời giải. Áp dụng công thức của định lý


1
S1 = Im(p2 p1 + p3 p2 + p1 p3 )
2  
1 a2 + a5 a1 + a4 a3 + a6 a2 + a5 a1 + a4 a3 + a6
= Im . + . + .
2 2 2 2 2 2 2
11
= Im(a2 a4 + a3 a5 + a4 a6 + a5 a1 + a6 a2 + a1 a3 )
42
11 1 1
+ Im(a2 a1 + a3 a2 + a4 a3 + a5 a4 + a6 a5 + a1 a6 ) = S + B
42 4 4
102 Chương 11. Diện tích đa giác

Mặt khác,
 
1 a2 + a3 a1 + a2 a3 + a4 a2 + a3 a1 + a2 a3 + a4
S2 = Im . + . + .
2 2 2 2 2 2 2
11
= Im(a2 a1 + a3 a2 + a4 a3 + a5 a4 + a6 a5 + a1 a6 )
42
11
+ Im(a3 a1 + a4 a2 + a5 a3 + a6 a4 + a1 a5 + a2 a6 ).
42
Nhưng những số (a2 a4 + a3 a5 + a4 a6 + a5 a1 + a6 a2 + a1 a3 ) và
(a2 a1 + a3 a2 + a4 a3 + a5 a4 + a6 a5 + a1 a6 ) là liên hợp cùng nhau, suy
2 1
ra S2 = S − B, từ đó ta có S1 + S2 = S.
4 4
3
4
J
11.3. BÀI TẬP
. 11.6. Cho M và N là trung điểm của các đuw ờng trung tuyến
BD và CE của tam giác ABC. Hãy tìm tỷ số của diện tích tứ giác
BCM N và tam giác ABC.

. 11.7. Cho tam giác ABC. M và N là trung điểm của các cạnh
AC và AB, còn điểm P và Q chia cạnh BC làm ba phần bằng nhau.
Tìm tỷ số của diện tích tứ giác QP LK và tam giác ABC, ở đây K
và L là điểm cắt của M B tương ứng với AQ và N P .

. 11.8. Cho D là trung điểm cạnh BC của tam giác ABC và M là


điểm bất kỳ trên BD. Từ D dựng đường thẳng song song với AM
và cắt AC tại N . Chứng minh rằng diện tích tứ giác ABM N không
phụ thuộc vào việc chọn điểm M .

. 11.9. Chứng minh rằng nếu A1 , B1 , C1 là điểm cắt của chân các
đường cao trong tam giác ABC với đường tròn ngoại tiếp, thì diện
tích của lục giác AC1 BA1 CB1 bằng hai lần diện tích tam giác ABC.
11.3. Bài tập 103

. 11.10. Điểm A0 là giao của những đường thẳng vuông góc dựng
từ điểm B và C đối với các cạnh AB và AC của một tam giác nhọn
ABC. Những điểm B 0 và C 0 cũng dựng tương tự. Chứng minh rằng
diện tích của lục giác AC 0 BA0 CB 0 là gấp đôi diện tích tam giác
ABC.

. 11.11. Trong đường tròn tâm O dựng đường kính AB và một dây
cung không giao với AB là CD. Cho E và F lần lượt là chân đường
vuông góc hạ từ A và B xuống CD. Chứng minh rằng diện tích tứ
giác ABF E bằng tổng của diện tích tam giác ABC và ABD.

. 11.12. Cho B1 , B2 , B3 là ba điểm lần lượt trên các cạnh A2 A3 ,


A3 A1 , A1 A2 của tam giác A1 A2 A3 và
A2 B1 A3 B2 A1 B 3
= λ1 , = λ2 , = λ3 .
B1 A3 B 2 A1 B 3 A2
Hãy tính tỷ số của diện tích các tam giác B1 B2 B3 và A1 A2 A3 .

. 11.13. Cho B1 , B2 , B3 là ba điểm lần lượt trên các cạnh A2 A3 ,


A3 A1 , A1 A2 của tam giác A1 A2 A3 và
A2 B1 A3 B2 A1 B 3
= λ1 , = λ2 , = λ3 .
B1 A3 B 2 A1 B 3 A2
Ký hiệu C1 ,C2 ,C3 lần lượt là các giao điểm của cặp đường thẳng
A2 B2 và A3 B3 , A3 B3 và A1 B1 , A1 B1 và A2 B2 . Tính tỷ số của diện
tích tam giác C1 C2 C3 và A1 A2 A3 .

. 11.14. Cho tứ giác lồi ABCD có hai đường chéo cắt nhau ở K.
Gọi J là trung điểm của đoạn thẳng nối hai trung điểm các đường
chéo, O là điểm đối xứng của K qua tâm J. Chứng minh rằng bốn
đoạn thẳng nối điểm O với trung điểm các cạnh tứ giác thì chia tứ
giác thành bốn đa giác có diện tích bằng nhau.
104 Chương 11. Diện tích đa giác

. 11.15. Trong mặt phẳng đa giác lồi ABCD lấy một điểm bất kỳ
M . Xét đa giác lồi với những đỉnh là những điểm đối xứng của M
qua trung điểm của các cạnh tứ giác đã cho. Chứng minh rằng diện
tích của tứ giác mới bằng hai lần diện tích tứ giác đã cho.
Chương 12
LỜI GIẢI VÀ GỢI Ý BÀI TẬP

12.1. Lời giải và gợi ý bài tập chương 2 . . . . . . . . . . . . . . . . 105

12.2. Lời giải và gợi ý bài tập chương 3 . . . . . . . . . . . . . . . . 110

12.3. Lời giải và gợi ý bài tập chương 4 . . . . . . . . . . . . . . . . 115

12.4. Lời giải và gợi ý bài tập chương 5 . . . . . . . . . . . . . . . . 123

12.5. Lời giải và gợi ý bài tập chương 6 . . . . . . . . . . . . . . . . 128

12.6. Lời giải và gợi ý bài tập chương 7 . . . . . . . . . . . . . . . . 132

12.7. Lời giải và gợi ý bài tập chương 8 . . . . . . . . . . . . . . . . 139

12.8. Lời giải và gợi ý bài tập chương 9 . . . . . . . . . . . . . . . . 147

12.9. Lời giải và gợi ý bài tập chương 10 . . . . . . . . . . . . . . . 152

12.10. Lời giải và gợi ý bài tập chương 11 . . . . . . . . . . . . . . 158

12.1. LỜI GIẢI VÀ GỢI Ý BÀI TẬP CHƯƠNG 2


. 2.6. Lời giải. (Hình 12.1) Lấy toạ độ điểm gốc là B và a = −1,
1 2
d = i − 1, c = i. Dễ dàng thấy rằng n = i, b = 0, m = (i − 1).
3 3
106 Chương 12. Lời giải và gợi ý bài tập

2 D C
a−m −1 − (i − 1)
V (a, n, m) = = 3
n−m 1 2 M
i − (i − 1)
3 3
2i + 1 2i + 1 2 + i N
=− =−
−i + 2 2−i 2+i
4i + 2 − 2 + i
=− = −i A
4+1 B

Vậy AM ⊥ M N . J Hình 12.1.

. 2.7. Lời giải. (Hình 12.2) Dễ dàng D


tính E C
1 K
h = (a + b + c + d) và
4
1 H
k = (b + c + d + e)
4
1
từ đó suy ra h − k = (a − e). Đẳng A B
4
thức này chỉ ra rằng HK song song với
1
AE và |HK| = |AE|.
4
J Hình 12.2.

. 2.8. Lời giải. Chọn toạ độ có gốc tại A, do đó


b+c
a = 0, m = , h = ic và f = −bi.
2
Ta xét
b+c b+c
(m − a)(f − h) + (m − a)(f − h) = (bi + ic) + (−bi − ic)
2 2
b+c
=i (b + c − b − c) = 0
2
Do đó AM ⊥ F H. J
. 2.9. Lời giải. (Hình 12.3) Lấy toạ độ gốc tại A. Do đó a = 0,
b+c 1 1
k= , e = ib, g = −ic. Vậy h = b(1 + i), l = c(1 − i)
2 2 2
12.1. Lời giải và gợi ý bài tập chương 2 107

h−k
V (h, l, k) = E
l−k P
1 b+c D
b(1 + i) − G
= 2 2 H
1 b+c A
c(1 − i) −
2 2 L
ib − c F
= = −i
−ic − b C B
K
Vậy HK ⊥ KL
Hình 12.3.
và |HK| = |KL|.
Đỉnh thứ tư của hình vuông có ba đỉnh L, K, H là P trung điểm
1
của GE. Thật vậy, p = i(b − c)
2
1 1
h−p b(1 + i) − i(b − c) b + ic i
V (h, l, p) = = 2 2 = =i
l−p 1 1 c − ib i
c(1 − i) − i(b − c)
2 2
do đó P L ⊥ HP và |P H| = |P L|. J
. 2.10. Lời giải. (Hình 12.4) Chọn A là gốc toạ độ, ta có a = 0,
b+c 1 A
m= , d = b. Lấy I 0 là trung
2 3
1
điểm của AM , vậy i0 = (b + c) D
4
1 I
c − i0 c − (b + c)
0
V (c, d, i ) = = 4
d − i0 1 1
b − (b + c) B C
3 4 M
1 3c − b 1
= =− . Hình 12.4.
4 b − 3c 4
Do đó, D, I 0 , C thẳng hàng, vậy I 0
trùng với I và
|DI|
|CD|
= .
1
4
J
108 Chương 12. Lời giải và gợi ý bài tập

. 2.11.Lời giải. (Hình 12.5) Chọn A là gốc toạ độ, a = 0. Theo điều
kiện đã cho N
ib
0 − m = i(b − m) =⇒ m = , M
i−1
ic − b A
b − p = i(c − p) =⇒ p =
i−1
c B
c − n = i(0 − n) =⇒ n = C
i−1
ib − c
ta tính được m − n = và
i−1 P
ic − b
p−a= suy ra Hình 12.5.
i−1
|m − n| = |p − a| =⇒ |M N | = |P A| và
m−n
p−a
= i. Do đó M N ⊥ P A. J
a+b c+d
. 2.12.Lời giải. (Hình 12.6) Ta có m = ,n= . Theo công
2 2
thức tính góc A
D
\ = (a−d)(m−n)+(a−d)(m−n)
cos AEM 2|m−n||a−d| E F
(b−c)(m−n)+(b−c)(m−n) M
cos M
\ FB = 2|m−n||b−c| N
vì |a − d| = |b − c| nên C
B
(a − d)(a − d) = (b − c)(b − c).
Hình 12.6.

cos AEM
\ − cos BF
\ M=
(m − n)[(a − d) − (b − c)] (m − n)[(a − d) − (b − c)]
= +
2|m − n||b − c| 2|m − n||b − c|
[(a − d) + (b − c)][(a − d) − (b − c)]
=
4|m − n||b − c|
12.1. Lời giải và gợi ý bài tập chương 2 109

[(a − d) + (b − c)][(a − d) − (b − c)]


+
4|m − n||b − c|
(b − c)(a − d) − (a − d)(b − c) − (a − d)(b − c) + (a − d)(b − c)
=
4|m − n||b − c|
= 0.

Vậy AEM
\ = BF
\ M. J
. 2.13.Lời giải. (Hình 12.7) Các số a, b, c, d là nhãn của các đỉnh tứ
giác ABCD. Còn e, f , g, h là nhãn
của các tâm hình vuông. Ta có D G
H
a − e = i(b − e), b − f = i(c − f ),
C
c − g = i(d − g), d − h = i(a − h),
từ đó ta nhận được A
F
a − bi b − ci
e= ,f = , B
1−i 1−i
c − di d − ai E
g= ,h =
1−i 1−i
Chúng ta chỉ cần kiểm tra
Hình 12.7.
g − e = i(f − h). Thật vậy

i 1
i(f − h) = ((b − ci) − (d − ai)) = (bi + c − di − a)
1−i 1−i
=
1
1−i
((c − di) − (a − bi)) = g − e J
. 2.14. Gợi ý : Dễ dàng thấy rằng trung điểm của đoạn thẳng mô
1
tả trên đều có nhãn là (a + b + c + d). Đó chính là điểm chung của
4
các đoạn thẳng.
. 2.15. Lời giải. (Hình 12.8) Chọn hệ toạ độ có điểm gốc tại A và
1 1
b = 1, do đó d = i, c = 1 + i, m = , n = 1 + i. Vì P là giao điểm
2 2
110 Chương 12. Lời giải và gợi ý bài tập

của CM và DN ta có (ba điểm nằm trên đường thẳng)


p−m p−m D C
=
c−m c−m
p−n p−n P
=
d−n d−n
N
Thay những giá trị ở trên vào và rút
M
gọn được
A B
p(2i − 1) + p(2i + 1) = 2i
p(i + 2) + p(i − 2) = 4i Hình 12.8.
4 3
Giải hệ phương trình trên chúng ta tìm được p = + i nghĩa
J
5 5
là |p| = 1, bằng độ dài cạnh góc vuông.

12.2. LỜI GIẢI VÀ GỢI Ý BÀI TẬP CHƯƠNG 3


. 3.6. Lời giải. Ký hiệu hình thang ABCD. Gọi giao điểm hai
đuwờng chéo là Q, M, N là trung điểm AB và CD. Chọn điểm
đầu toạ độ là O giao của hai cạnh bên, vì AB//CD, nên d = λa và
c = λb với λ là một số thực nào đó. Khi đó những điểm O, M , và N
a+b c+d a+b
nằm trên một đường thẳng , vì m = và n = =λ .
2 2 2
Để chứng minh điểm Q nằm trên đường thẳng này, chúng ta chú ý
q với
λ 1 λ 1 λ
q= a+ c= b+ d= (a + b)
1+λ 1+λ 1+λ 1+λ 1+λ
Đẳng thức này chỉ ra rằng Q có nhãn q nằm trên AC và BD. Đồng
thời Q còn nằm trên đường thẳng OM . J
. 3.7. Lời giải. (Hình 12.9) Gọi M là giao của BD và AE. Chúng
b+c 1 |AD| |d − a| d−a
ta có e = và = = = từ đó suy ra
2 4 |DC| |c − d| c−d
12.2. Lời giải và gợi ý bài tập chương 3 111

4a + c 4a + b + c
d= . Đặt n = , một mặt
5 6
2 1 2 1 C
n = a + (b + c) = a + c
3 6 3 3
E
1 1 1 5
mặt khác n = b + (4a + c) = b + d D M
6 6 6 6
A B
vậy điểm N nằm trên AE, đồng thời nằm
trên BD, do đó nó chính là giao điểm của Hình 12.9.
hai đường trên. Để xác định tỷ số ta chỉ còn
tính
|AM |
|M E|
=
|n − a|
|e − n|
1
= .
2
J
. 3.8. Lời giải. (Hình 12.10) Chọn hệ toạ O

độ với điểm gốc là giao O của hai cạnh


bên AD và BC. Do DP//OM và P là
D C
trung điểm của AM suy ra D là trung
điểm của OA và |OA| = 2|OD|. N M

A P B
Do đó a = 2d và b = 2c. Còn điểm M
b+c 3
và P có m = = c và
2 2 Hình 12.10.
3
a+m 2d + c
p= = 2 = 4d + 3c .
2 2 4
Nếu X là điểm cắt của đường chéo AC và đường trung bình của
a+c 2d + c
hình thang, thì dễ thấy x = = , mặt khác
2 2
4d + 3c
2d + c d+2 d + 2p
x= = 4 = .
2 3 3
Điều này chứng tỏ X nằm trên đường thẳng DP . J
112 Chương 12. Lời giải và gợi ý bài tập

. 3.9. Lời giải. (Hình 12.11) Chọn điểm đầu toạ độ vuông góc là A.
p s
Đặt λ = và µ = . Khi đó q = b + s = b + µd và r = d + p = d + λb.
b d
λµ(b + d)
Điểm giao H của P Q và RS là h = .
λ+µ−1
Thật vậy, D R C
λµ(b + d)
h=
λ+µ−1 M
S Q
λ µ−1
= q+ p B
λ+µ−1 λ+µ−1
A P H
µ λ−1
= r+ s
λ+µ−1 λ+µ−1
Hình 12.11.
Ta cũng thấy H nằm trên đường AC.
Tất nhiên ta thấy rằng λ + µ − 1 6= 0, nếu ngược lại thì P S và QR
sẽ song song với nhau và nó không cắt nhau. J
. 3.10. Lời giải. (Hình 12.12) N H
Chọn toạ độ có gốc tại A, vậy G
F
a = 0, h = ic,
g = c(1 + i), A
E
f = ib,
e = b(1 − i), M C
i(c − b) B
n= ;
2
Hình 12.12.
M là giao của EC và BG.
EC : (c−b(1+i))m−(c−b(1−i))m+(c(b(1+i))−cb(1−i)) = 0
BG : (b−c(1+i))m−(b−c(1−i))m+(b(c(1+i))−bc(1−i)) = 0
Hoặc là

m m m m
= =⇒ =
i(b − c) i(b − c) n n
2 2
12.2. Lời giải và gợi ý bài tập chương 3 113

và a = 0 suy ra điểm M, A, N nằm trên một đường thẳng. J


. 3.11. Lời giải. (Hình 12.13) Do M N song song với AB và DC
nên D C
AM AP BQ BN
= = = =λ
MD PC QD NC M N
P Q
Vì vậy A B
a + λd b + λd
m= ,q = Hình 12.13.
1+λ 1+λ
a + λc b + λc
p= ,n =
1+λ 1+λ
Từ đó suy ra p − m =
λ(c − d)
1+λ
= n − q vậy |M P | = |QN |. J
k AM DN a+d
. 3.12. Lời giải. Đặt λ = , thì = = λ. p = ,
1−k MB NC 2
b+c a + λb d + λc a + d + λ(b + c)
q= ,m= ,n= suy ra s = .
2 1+λ 1+λ 2(1 + λ)

a + d a + d + λ(b + c)

p−s 2 2(1 + λ)
V (p, q, s) = =
q−s b + c a + d + λ(b + c)

2 2(1 + λ)
a + d + λ(a + d) − (a + d) − λ(b + c)
= = −λ
b + c + λ(b + c) − (a + d) − λ(b + c)

là một số thực, vậy 3 điểm P, S, Q nằm trên một đường thẳng và


|P S|
|SQ|
= λ hoặc là
PS
PQ
= k. J
a+c b+c
. 3.13. Lời giải. (Hình 12.14) Ta có b1 = , a1 = ,
2 2
a+b 3a + c 3b + c
k= ,m= ,n= , Ta lấy điểm P và Q có nhãn
2 4 4
114 Chương 12. Lời giải và gợi ý bài tập

a+c C
+b a + c + 2b
q= 2 =
2 4
b+c
+a b + c + 2a B1 A1
p= 2 =
2 4
3 M P Q N
m − n = (a − b)
4
1 A K B
m − p = (a − b)
4
1 Hình 12.14.
n − q = − (a − b)
4
1
p − q = (a − b)
4
Suy ra P Q song song với AB. Nhìn vào các đẳng thức trên ta
thấy P Q đi qua M , N và có tính chất như bài toán đặt ra. J
. 3.14. Lời giải. (Hình 12.15) Lấy m là nhãn của điểm M . Ta có
b+c a+c a+b C
a1 = , b1 = , c1 = M1
2 2 2
và m1 = b + c − m, m2 = a + c − m, M 2

m3 = a + b − m. Do đó M0
B1 A1
a − m2 = m − c = b − m1 và
m2 − m1 = a − b, suy ra AM2 song M
song với BM1 và M1 M2 song song A M3 C1 B
với AB. Vậy ABM1 M2 là hình bình
Hình 12.15.
hành.
Giao điểm của AM1 và BM2 là điểm giữa của các đoạn này, ký
a+b+c−m
hiệu là M 0 . Do đó m0 = . Với a, b, c có vai trò như
J
2
nhau, nên ta thấy ngay m0 đúng là giao của ba đường thẳng.
. 3.15. Lời giải. Trong tam giác A1 A2 A3 ta kẻ các đường phân giác
trong A1 L1 , A2 L2 ,A3 L3 . Điểm L3 nằm trong đoạn A1 A2 , suy ra
l3 = λa1 + (1 − λ)a2
12.3. Lời giải và gợi ý bài tập chương 4 115

l3 − a2 |L3 A2 |
ở đây 0 < λ < 1 và λ = = . Nhưng do tính chất của
a1 − a2 d3
|L3 A2 | d1
phân giác = , ta nhận được
|L3 A1 | d2
1
l3 = (d1 a1 + d2 a2 ).
d1 + d2
Chúng ta xét số
1 d1 + d2 d3
l= (d1 a1 + d = l3 a3
d1 + d2 + d3 d1 + d2 + d3 d1 + d2 + d3
Đây là nhãn tương ứng với điểm L và nằm trên đường thẳng A3 L3 .
Tương tự L nằm trên A2 L2 , A1 L1 . Suy ra L là giao của 3 đường
trên. J
12.3. LỜI GIẢI VÀ GỢI Ý BÀI TẬP CHƯƠNG 4
. 4.6. Lời giải. Ta chọn O là điểm gốc của hệ toạ độ vuông góc. Ta
có a1 = a, b1 = b, c1 = c. Cho z là một số phức bất kỳ. Chúng ta đặt
(z − b)(a − c) (z + b)(a + c)
= α; = β;
(z − c)(a − b) (z − c)(a − b)
(z − b)(a − c) (z + b)(a + c)
= γ; = δ;
(z + c)(a + b) (z + c)(a + b)
(β − 1)γ
Bằng cách kiểm tra trực tiếp thấy rằng = α và αδ = βγ.
1−γ
Nếu Z là điểm cắt thứ hai của các đường A
B
tròn ngoại tiếp tam giác A1 B1 C và tam
giác A1 BC1 khác A1 , khi đó giá trị của β N
và γ là số thực, suy ra α và δ cũng nhận P
D
M
giá trị thực. Nói một cách khác Z cùng
nằm trên các đường tròn ngoại tiếp tam
giác ABC và AB1 C1 . J C

Hình 12.16.
116 Chương 12. Lời giải và gợi ý bài tập

. 4.7. Lời giải. (Hình 12.16) Chọn hệ toạ độ lấy đường tròn làm
đường tròn đơn vị. Như bài 4.3 tính được
−abc + abd + acd − bcd
m= ,
cd − ab
abc − abd − acd + bcd
n= ,
cd − ab
Suy ra m + n = 0, nghĩa là M , N đối xứng qua tâm O và O,M ,
N nằm trên một đường thẳng.
Điểm P là giao điểm của đường thẳng BC và AD. Khi đó
(a + d)bc − (b + c)ad n bc − ad
p= và = .
bc − ad p cd − ab
Nhưng
11 11
n bc − ad − ad − bc n
= = b c ad = = .
p cd − ab 11 11 ab − cd p

cd ab
n
Hay là, là một số thực, nghĩa là P nằm trên đường thẳng M N .
p
Nếu BC và AD song song với nhau. Khi đó ad = bc. Trong
bc
trường hợp này thay d bằng . Ta có
a
bc bc bc
abc − abd − acd + bcd abc − ab − ac + bc
n= = a a a
cd − ab bc
c − ab
a
(a2 − ab − ac + bc)bc (c − a)(b − a)c (b − a)c
= =
(c2 − a2 )b (c − a)(c + a) c + a
n (b − a)c
Nhưng = và
b−c (c + a)(b − c)
1 1 1
n ( − ) (a − b)c n
= b a c = = .
b−c 1 1 1 1 (a + c)(c − b) b−c
( + )( − )
c a b c
12.3. Lời giải và gợi ý bài tập chương 4 117

nghĩa là ON (hay là M N ) song song với BC và AD. J


. 4.8. Lời giải. (hình 12.17) Chọn hệ
toạ đọ vuông góc sao cho đường tròn
chính là đơn vị và đường kính AB
nằm trên trục hoành. Khi đó a = −1,
b = 1, và điểm X là giao của AC và
BD
(−1 + c)d + (1 + d)c
x=
d+c
Hình 12.17.
2cd + c − d
=
c+d
2cd
Giao điểm Y của hai đường tiếp tuyến có công thức y = .
c+d
−−→
Vectơ XY được biểu diễn thành số phức

2cd 2cd + c − d d−c


− = .
d+c c+d c+d

1 1
d−c −
Nhưng = d c = − d − c , nghĩa là số d − c hoàn toàn phức.
c+d 1 1 c+d c+d
+
c d
−−→
Như vậy vectơ XY vuông góc với trục hoành, đó chính là đường
thẳng AB. Vì trong biểu thức điểm C và D tham gia đối xứng nhau
nên dễ dàng thấy rằng điểm cắt AD và BC cũng nằm trên đường
thẳng XY (hai đường thẳng đi qua điểm X và cùng vuông góc với
một đường thẳng nên nó trùng nhau.) J
. 4.9. Lời giải. (Hình 12.18) Chọn đường tròn k làm đường tròn đơn
vị . Vì AB và CD là những đường kính, do đó b = −a và d = −c.
Theo công thức tính nhãn của P và Q:
118 Chương 12. Lời giải và gợi ý bài tập

1
p = (a − a + m + a2 m)
2
1
q = (c − c + m + c2 m)
2
1
Vậy |p − q| = |m(a2 − c2 )|
2
1
= |a2 − c2 |,
2
độ dài P Q không phụ thuộc gì vào
Hình 12.18.
nhãn m của điểm M , trong khi a, c là
cố định. J
. 4.10.Lời giải. (Hình 12.19) Chọn hệ
toạ độ sao cho đường tròn đã cho là
đơn vị và trục hoành song song với P Q,
nghĩa là m là một số hoàn toàn ảo,
m = −m. Từ điều kiện M nằm trên AB
và CD, ta có
a + b = m + abm
c + d = m + cdm Hình 12.19.
Từ đó có
abm = m − a − b, cdm = m − c − d. Ngoài ra,

(p + q)ad − (a + d)pq 2mad + a + d


x= =
ad − pq ad + 1

tại vì, nếu q = cos φ + i sin φ, thì p = − cos φ + i sin φ và


pq = − sin2 φ − cos2 φ = −1. Tương tự

2mbc + b + c
y=
bc + 1
12.3. Lời giải và gợi ý bài tập chương 4 119

chúng ta xét
mad + a + d − m mbc + b + c − m
x+y−2m = (x−m)+(y−m) = +
ad + 1 bc + 1

Qui đồng mẫu số xét tử số:

(mad + a + d − m)(bc + 1) + (mbc + b + c − m)(ad + 1) =

= mabcd + abc + bcd − mbc + mad + a + d − m + mabcd

+abd + acd − mad + mbc + b + c − m =

= (m − a − b)cd + abc + bcd + (m − c − d) + abd + acd + a + b + c − 2m

= (mcd + c + d − m) + (mab + a + b − m) = 0 + 0 = 0.
J
. 4.11. Lời giải. Chọn đường tròn đơn vị là đường tròn ngoại tiếp
tứ giác ABCD. Khi đó
2ac 2bd
m= ,n = .
a+c b+d

Nếu A, C và N nằm trên một đường thẳng, khi đó


n−a n−a
=
c−a n−c

thay n vào biểu thức trên và a, b, c, d nằm trên đường tròn đơn vị ,
ta nhận được 2(ac + bd) = (a + c)(b + d). Biểu thức này không thay
đổi khi ta thay a bằng b và c bằng d. Do đó điều kiện cần và đủ để
các điểm A,C, N nằm trên đường thẳng có thể thay bằng B, D và
M. J
. 4.12. Lời giải. (Hình 12.20) Chọn hệ toạ độ có điểm gốc là giao
120 Chương 12. Lời giải và gợi ý bài tập

hai đường chéo hình bình hành và


đường tròn làm đơn vị. Không mất
tính tổng quát chọn a = −1, c = 1.
Ta thấy ngay b + d = 0. Tìm nhãn
điểm M , khi ta biết M nằm trên
đường tròn và trên đường AB :
1+b
m + a = b + mab =⇒ m =
1+b
(do a = −1). Hình 12.20.
1−b
Tương tự, ta có n = . Đường thẳng M N có phương trình
1−b
dạng m + n = z + mnz sau khi thế m và n, ta nhận được
2
(1 − b )z + (1 − b2 )z = 2(1 − bb)

Đường tiếp tuyến tại C có phương trình z + z = 2 =⇒ z = 2 − z.


Thay vào phương trình đường thẳng M N , ta tính ra nhãn của điểm
cắt của hai đường là
2(b2 − bb) 2
z= 2 = b.
b2 − b b+b
số b + b là một số thực, nghĩa là điểm cắt giữa M N và tiếp tuyến tại
C nằm trên đường OB, mà OB chính là DB, do vậy DB cũng đi
qua giao điểm hai đường trên. J
. 4.13.Lời giải. (Hình 12.21) Chọn hệ toạ độ có tâm của nửa đường
tròn và đường tròn đó làm đơn vị, hơn nữa a = −1, b = 1, ta tính
_ _
được bd = c2 (do BD= 2 BC) suy ra c2 = d. Khi đó
(a + c)bp − (b + p)ac (c − 1)p + (p + 1)c
e= =
bp − ac p+c
(c2 − 1)pc + (p + c)c2 (c2 − 1)p + (p + c)c
f= =
pc + c2 p+c
12.3. Lời giải và gợi ý bài tập chương 4 121


p(c2 − 1) + (p + c)c − (c − 1)p − (p + 1)c (p + 1)c(c − 1)
f −e = =
p+c p+c

từ đó suy ra D
f −e (p + 1)c(c − 1) D F C
=
d−a (p + c)(c2 + 1)
f −e (p + 1)c(c − 1) E B
A
=
d−a (p + c)(c2 + 1)
(p + 1)c(c − 1)
=−
(p + c)(c2 + 1)
Vậy EF và AD
vuông góc với nhau. J Hình 12.21.

. 4.14.Lời giải. (Hình 12.22) Lấy đường tròn đã cho làm đơn vị và
a = 1, b = −1. Ký hiệu nhãn của điểm
cắt AD1 , AD2 với tB lần lượt là −1+x1 i C
và −1 + x2 i với x1 , x2 là số thực.
Khi đó các điểm cắt tB
nằm trên đường thẳng nên D2
−1 + x1 i − 1 −1 − x1 i − 1 D1
= từ
1 − d1 1 − d1
2(d1 + 1)i B A
đó suy ra x1 = .
1 − d1
E1
2(d2 + 1) E2
Tương tự x2 = i. Như vậy tB tA
1 − d2
độ dài do hai tia AD1 và AD2 cắt ra trên Hình 12.22.
tB là

2(d2 + 1 2(d1 + 1) 4(d2 − d1 )
1 − d2 − 1 − d1 = (1 − d2 )(1 − d1 ) .

122 Chương 12. Lời giải và gợi ý bài tập

Tương tự độ dài do AE1 và AE2 cắt ra trên tB là


4(e2 − e1 )
| |. Ta biết rằng C nằm trên tiếp tuyến tại A vậy
(1 − e2 )(1 − e1 )
c + c = 2, nhưng
(e1 + d1 )e2 d2 − (e2 + d2 )e1 d1
c= chúng ta nhận được
e2 d2 − e1 d1
(e1 + d1 )e2 d2 − (e2 + d2 )e1 d1 −e1 − d1 + e2 + d2
+ =2
e2 d2 − e1 d1 e2 d2 − e1 d1

(e1 + d1 )e2 d2 − (e2 + d2 )e1 d1 − (e1 + d1 − e2 − d2 ) = 2(e2 d2 − e1 d1 )


hoặc là
e2 − e1 d2 − d1
+ =0
(1 − e1 )(1 − e2 ) (1 − d1 )(1 − d2 )
Do đó suy ra độ dài hai đoạn đã tính là bằng nhau. J
. 4.15.Lời giải. (Hình 12.23) Chọn
hệ toạ độ sao cho đường tròn là đơn D
Q T
vị và a = −1, b = 1. Ta ký hiệu −1 + C
si và 1 + ti là nhãn của S và T (s, t R
S
là các số thực). Do P S và AQ vuông P
A B
góc nhau, nên
−1 + si − p −1 − si − p Hình 12.23.
=− ,
q+1 q+1
2 + pq + pq + p + q + p + q
từ đó suy ra si = .
q−q
2 + rq + rq − r − q − r − q
Tương tự có ti = . Do đó
q−q

pq + pq − rp − rq + p + p + r + r + 2(q + q)
(−1 + si) − (1 + ti) = −2 + (∗)
q−q

c−1 c+d d+1


Nhưng p = ,q= ,r= , từ đó ta có p + r = q và
2 2 2
12.4. Lời giải và gợi ý bài tập chương 5 123

c−d
p−r = − 1. Do đó
2

c−d c−d
q( − 1) + q( − 1) + 3(q + q)
(∗) = −2 + 2 2
q−q
−2q + 2q + 2q + 2q
=
q−q
4q 4 4cd
= = = .
c+d c+d cd − 1 1 − cd
cd −
2 2

cd
Nhưng số là số thực, vì nó trùng với số liên hợp, vậy
(1 − cd)(d − c)
ST song song với CD. J
12.4. LỜI GIẢI VÀ GỢI Ý BÀI TẬP CHƯƠNG 5
. 5.6. Lời giải. Chọn điểm gốc hệ toạ độ là O và đường tròn ngoại
tiếp A1 A2 A3 là đơn vị. Do đó ta có b3 = a1 + a2 , b1 = a2 + a3 ,
b2 = a3 + a1 . Từ đẳng thức b3 − b2 = (a1 + a2 ) − (a3 + a1 ) = a2 − a3
suy ra B2 B3 và A2 A3 bằng nhau và song song với nhau. Ngoài ra
−−−→ −−−→
A2 A3 = −B2 B3 .
−−−→ −−−→ −−−→ −−−→
Tương tự ta cũng có A3 A1 = −B3 B1 và A1 A2 = −B1 B2 . Nghĩa là
hai tam giác A1 A2 A3 và B1 B2 B3 bằng nhau và có các cạnh song
a1 + a2 + a3
song tương ứng. Ta cũng thấy ngay E với nhãn e =
2
nằm trên đường A1 B1 , A2 B2 , A3 B3 và đó là tâm đối xứng của hai
tam giác này. Như các bài trên đã chỉ ra thì điểm E chính là tâm
đường tròn Euler của tam giác A1 A2 A3 . J
. 5.7. Lời giải. Chọn hệ toạ độ có đường tròn ngoại tiếp tam giác
A1 A2 A3 là đơn vị . Điểm P là đối xứng của H qua A1 , chúng ta có
124 Chương 12. Lời giải và gợi ý bài tập

h − a1 = a1 − p hoặc là

p = 2a1 − h = 2a1 − a1 − a2 − a3 = a1 − a2 − a3 .

Còn điểm Q đối xứng của A1 qua trung điểm A2 A3 , ta có


a2 + a3 a2 + a3
q− = −a1 , hoặc là q = a2 +a3 −a1 . Do đó p+q = 0.
2 2
Nghĩa là P Q đi qua tâm O và O chia P Q ra làm đôi. J
A3
. 5.8. Lời giải. Do đoạn thẳng A1 A2 và
A3 H vuông góc với nhau và bằng nhau thì
ta có thể tính được O
a1 − a2 = i(h − a3 ) = i(a1 + a2 ).
a2 1−i A1 M A2
Do đó = = −i, vì vậy
a1 1+i D3
A\ ◦ ◦
1 OA2 = 90 , nghĩa là A1 A3 A2 = 45 .
\ J C3
. 5.9. Gợi ý: (Hình 12.24) Chúng ta có
Hình 12.24.
c3 = a1 + a2 − a3 và d3 = −a3 .
c3 − d3 a1 + a2
Khi đó = . Tỷ số trên là
a1 − a2 a1 − a2
một số hoàn toàn phức, từ đó suy ra kết luận bài toán.
. 5.10.Lời giải. (Hình 12.25) Chọn hệ
toạ độ vuông góc với đường tròn ngoại
tiếp tam giác A1 A2 A3 là đơn vị . Do P
và Q thực chất là chân đường cao của
tam giác từ đỉnh A1 và A2 , ta có
1
p = (a1 + a2 + a3 − a2 a3 a1 )
2
1 Hình 12.25.
p = (a1 + a2 + a3 − a1 a3 a2 )
2
Tâm của đường tròn ta vẽ là c3 , đó là trung điểm của A3 H, nên
12.4. Lời giải và gợi ý bài tập chương 5 125


1 1
c3 = (a3 + h) = a3 + (a1 + a2 ).
2 2
a1 + a2
M3 là trung điểm của A1 A2 , ta có m3 = . Khi đó
2
1 1
m3 − p (a1 + a2 ) − (a1 + a2 + a3 − a2 a3 a1 )
= 2 2
c3 − p 1 1
a3 + (a1 + a2 ) − (a1 + a2 + a3 − a2 a3 a1 )
2 2
a2 a3 a1 − a3 a2 − a1
= =
a3 + a2 a3 a1 a2 + a1

1 1
m3 − q (a1 + a2 ) − (a1 + a2 + a3 − a1 a3 a2 )
= 2 2
c3 − q 1 1
a3 + (a1 + a2 ) − (a1 + a2 + a3 − a1 a3 a2 )
2 2
a1 a3 a2 − a3 a1 − a2
= = .
a3 + a1 a3 a2 a2 + a1
Những tỷ số trên là một số hoàn toàn ảo. Do đó M3 P ⊥ C3 P và
M3 Q ⊥ C3 Q, nghĩa là M3 P và M3 Q là tiếp tuyến tại P và Q đối với
đường tròn ta đang xét. J
. 5.11. Lời giải. (Hình 12.26) Gọi
A3 D3 là đường kính đã cho, điểm N

( nhãn thỏa mãn hai phương trình
a1 + a2 = n + a1 a2 n
a3 + d3 = n + a3 d3 n
vì d3 = −a3 , giải phương trình ta có
a1 + a2
n=
1 + a1 a2 a23
Hình 12.26.
Khi đó
1 1 a1 + a2
m = (a3 + n) = (a3 + ).
2 2 1 + a1 a2 a23
126 Chương 12. Lời giải và gợi ý bài tập

Ta xét
1 a1 + a2
(a3 + )
m−e 2 1 + a1 a2 a23
=
p3 − e 1 1
(a1 + a2 + a3 − a1 a2 a3 ) − (a1 + a2 + a3 )
2 2
(a1 + a2 )(−a1 a2 a23 ) (a1 + a2 )a3
= =
−a1 a2 a3 (1 + a1 a2 a23 ) a1 a2 + a23

Tỷ số trên là một số thực, thật vậy


1 1 1
( + )
m−e (a1 + a2 )a3 a a2 a3
= = 1
p3 − e a1 a2 + a23 1 1 1
+
a1 a2 a23
(a1 + a2 )a3 m−e
= 2 =
a1 a2 + a3 p3 − e
Nghĩa là M , E và P3 thẳng hàng. J
. 5.12. Lời giải. (Hình 12.27) Hình 12.27.
Lấy các ký hiệu như phần đầu chương 5, thì M 0 là trung điểm của
a3 + m3
A3 M3 nên m0 = . Theo giả thiết O0 là đối xứng của O qua
2
o0 + o
M 0 vậy = m0 hoặc là o0 = 2m0 ,
2
a3 + m3 a1 + a2
o0 = 2( ) = a3 + m3 = a3 + = c3 .
2 2
Đây chính là nhãn của điểm giữa A3 H, thuộc đường tròn Euler. J
. 5.13.Lời giải. (Hình 12.28) Ta ký A3
hiệu R1 , R2 , R3 là giao của đường
tròn và các đường vẽ song song với
OA1 , OA2 , OA3 đi qua Q1 Q2 Q3 . O
R3
Khi đó q1 r1 = (−a1 )a1 . Ta biết H
A1 A2
rằng q1 = −a2 a3 a1 , từ đó suy ra
r1 = a2 a3 a31 . Q3
Hình 12.28.
12.4. Lời giải và gợi ý bài tập chương 5 127

Tương tự, ta có r2 = a1 a3 a32 , r3 = a1 a2 a33 . Đường thẳng Q1 R1 và


Q2 R2 cắt nhau tại
(q1 + r1 )q2 r2 − (q2 + r2 )q1 r1
q2 r2 − q1 r1
(−a2 a3 a1 + a2 a3 a31 )(−a22 ) − (−a3 a1 a2 + a3 a1 a32 )(−a21 )
=
a21 − a22
a2 a2 + a22 a23 + a23 a21
= 1 2
a1 a2 a3

Biểu thức đối xứng với a1 , a2 , a3 . A3


Do đó Q1 R1 , Q2 R2 , Q3 R3 cắt nhau tại
điểm có nhãn như trên. J
. 5.14.Lời giải. (Hình 12.29) Chọn H O
đường tròn đơn vị như bài trước.
a1 + a2 A1 A2
h = a1 + a2 + a3 , d = . D
2 K
Ta lấy điểm đối xứng của H qua D, Hình 12.29.
0
ký hiệu là K , do đó
k0 + h a1 + a2
=d= ⇒ a1 + a2 + a3 + k 0 = a1 + a2 ⇒ k 0 = −a3 ,
2 0 2
nên |k | = |a3 | = 1.
Vậy K 0 nằm trên đường tròn và đường HD, đó chính là điểm K.
Hơn nữa, K là đối xứng của A3 qua tâm đường tròn ngoại tiếp. J
. 5.15. Lời giải. Chọn đường tròn đơn vị là đường tròn ngoại tiếp
1
tam giác ABC. Khi đó h = a + b + c, d = (a + b + c − acb) và
2
1 1 1 1
k = (b + d) = [b + (a + b + c − acb)] = (a + 3b + c − acb). Vì
2 2 2 4
BH h−b
= 3, nên = 3 =⇒ 4h = b + 3d, hay là
HD d−h
3
4(a + b + c) = b + (a + b + c − acb).
2
128 Chương 12. Lời giải và gợi ý bài tập

Từ đó suy ra 5ab + 3b2 + 5bc + 3ac = 0. Xét


1
k−c (a + 3b + c − acb) − c
= 4
k−a 1
(a + 3b + c − acb) − a
4
a + 3b − acb − 3c (a + 3b)(b − c)
= =
c + 3b − acb − 3a (c + 3b)(b − a)
1 3 1 1
k−c (a + 3b)(b − c) ( + )( − )
= = a b b c = (b + 3a)(b − c)
k−a (c + 3b)(b − a) 1 3 1 1 (b + 3c)(b − a)
( + )( − )
c b b a
b + 3a a + 3b (b + 3a)(c + 3b) + (a + 3b)(b + 3c)
Nhưng + =
b + 3c c + 3b (c + 3b)(b + 3c)
10ab + 6b2 + 10bc + 6ac
= =0
(c + 3b)(b + 3c)
Nghĩa là
k−c
k−a
\ = 90◦ .
hoàn toàn ảo, vậy AKC J
12.5. LỜI GIẢI VÀ GỢI Ý BÀI TẬP CHƯƠNG 6
. 6.6. Gợi ý: Như các ví dụ ở Chương 6, ta tính được q1 = −a2 a3 p.
a1 − q1 a1 + a2 a3 p
Ta xét =
p2 − p3 1 1
(a1 + a3 + p − a1 a3 p) − (a1 + a2 + p − a1 a2 p)
2 2
2(a1 p + a2 a3 )
=
(a3 − a2 )(p − a1 )

dễ dàng kiểm tra tỷ số trên là một số thực, nghĩa là A1 Q1 //P2 P3 .


Tương tự kiểm tra các đường còn lại đều song song với đường thẳng
Simson.

. 6.7. Lời giải. (Hình 12.30) Ký hiệu S là đỉnh thứ tư của hình bình
hành, xây dựng bởi P1 H và P1 P . Điểm S nằm trên đường cao và
12.5. Lời giải và gợi ý bài tập chương 6 129

P S là song song của HP1 . P


A3
Chúng ta sẽ chứng minh rằng S
nằm trên đường thẳng Simson, nghĩa P1
K
là S trùng với K.
Do P1 P SH là hình bình hành, O
H
nên s = h + p − p1 . A1 A2
D P3
Ta có
Hình 12.30.
s − p1 h + p − 2p1
=
p2 − p1 1 1
(a3 + a1 − a3 a1 p) − (a2 + a3 − a2 a3 p)
2 2
2(a1 + a2 + a3 + p − a2 − a3 − p)
=
(a1 − a2 )(1 − a3 p)
2(a1 + a2 a3 p) 2(a1 p + a2 a3 )
= = .
(1 − a3 p)(a1 − a2 ) (p − a3 )(a1 − a2 )
Tỷ số này là một số thực , nghĩa là S nằm trên đường thẳng Simson.
. 6.8. Lời giải. Như các bài trước thấy rằng vectơ song song với
đường thẳng Simson của điểm P đối với tam giác A1 A2 A3 là
(a3 − a2 )(p − a1 )p và q1 = −a2 a3 a1 ,q2 = −a1 a3 a2 , q3 = −a1 a2 a3 . Ta
xét tỷ số
(q3 − q2 )(p − q1 )p (−a1 a2 a3 + a1 a3 a2 )(p + a2 a3 a1 )
=
(a3 − a2 )(p − a1 )p (a3 − a2 )(p − a1 )
(a2 + a3 )(pa1 + a2 a3 )
=
a2 a3 (p − a1 )
Kiểm tra trực tiếp thấy rằng tỷ số này là một số hoàn toàn ảo. Do
đó góc giữa hai đường thẳng Simson của P đối với hai tam giác là
vuông góc. J
. 6.9. Lời giải. Nếu điểm đối xứng của P qua tâm đường tròn nằm
130 Chương 12. Lời giải và gợi ý bài tập

p1 + p
trên đường Simson, thì số sẽ là một số thực. Từ đó suy ra
p2 − p3
1
p1 + p (a2 + a3 + p − a2 a3 p) + p
= 2
p2 − p3 1 1
(a3 + a1 + p − a3 a1 p) − (a1 + a2 + p − a1 a2 p)
2 2
pa2 + pa3 + 3p2 − a2 a3
=
(a3 − a2 )(p − a1 )
p1 + p (pa2 + pa3 − p2 + 3a2 a3 )a1
=
p2 − p3 (a3 − a2 )(p − a1 )p
hai tỷ số trên bằng nhau, ta có
a1 + a2 + a3 p(a1 a2 + a2 a3 + a3 a1 ) + 3a1 a2 a3
p2 ( + p) = .
3 3
a1 + a2 + a3 a1 + a2 + a3
Ta lại có g = , từ đó g + p = + p; g +
3 3
1 1 1 1 1 1 p(a1 a2 + a2 a3 + a3 a1 ) + 3a1 a2 a3
p= ( + + )+ = =
3 a1 a2 a3 p 3 a1 a2 a3 p
p2 (g + p) p(g + p)
= . Từ đó dễ dàng suy ra trọng tâm G nằm trên
a1 a2 a3 p a1 a2 a3
đường Simson. J
. 6.10. Gợi ý: Theo giả thiết OA1 song song với đường thẳng Simson
a1
của điểm P , suy ra số là một số thực . Nghĩa là
p2 − p3
a1 a1
=
p2 − p3 p2 − p3
sau khi tính toán còn a2 a3 = a1 p, nghĩa là A2 A3 song song với A1 P .
. 6.11. Gợi ý: Gọi M là trung điểm của đoạn thẳng P H. Ta có
1 1
m = (p + h) = (p + a1 + a2 + a3 )
2 2
Ta sẽ chứng minh rằng M nằm trên đường Simson của điểm P . Thật
vậy,
m − p1 pa1 + a2 a3
=
p2 − p3 (a2 − a3 )(p − a1 )
12.5. Lời giải và gợi ý bài tập chương 6 131

đây là một số thực . Nghĩa là điểm M nằm trên đường thẳng đi qua
P1 P2 P3 .
. 6.12. Lời giải. (Bài này tổng quát ví dụ ở Chương 6) Góc phải
q2 − q3 1 − a1 q
tìm chính là agumen của số phức = . Nhưng ta
p2 − p3 1 − a1 p
có thể chọn hệ toạ độ số ảo cho a1 = 1 (mà không làm ảnh
hưởng tới kết luận bài toán). Nhưng vì agumen của 1 − q và 1 − p
bằng nửa argumen của q và p nên ta có kết luận bài toán (vì nếu
q = cos φ + i sin φ thì
1 φ φ
1 − q = 1 − cos φ + i sin φ = cos (cos + i sin )).
2 2 2
φ
2
J
. 6.13. Lời giải. Gọi chân đường cao từ P xuống các cạnh tam giác
A1 A2 A3 là P1 , P2 , P3 , và xuống các cạnh tam giác B1 B2 B3 là Q1 , Q2 ,
Q3 . Theo tính toán tương tự bài trên ta có p2 −p3 = (a3 −a2 )(1−a1 p)
và q2 − q3 = (b3 − b2 )(1 − b1 p). Ta xét
p2 − p3 (a3 − a2 )(1 − a1 p) p − a1
= =−
q2 − q3 (−a3 + a2 )(1 + a1 p) p + a1
1 1

p2 − p3 p a1 a1 − p p2 − p3
=− =− =−
q2 − q3 1 1 a1 + p q2 − q3
+
Do đó Q2 Q3 ⊥ P2 P3 .
p a1
J
. 6.14. Lời giải. Ta áp dụng phương trình đường thẳng Simson cho
điểm q3 = −a1 a2 a3 (do tính toán của các bài trước).
1
z − a1 a2 a3 (−a1 a2 a3 )z + a1 a2 a3 (−a1 a2 a3 )(a1 + a2 + a3 − a1 a2 a3 )−
2
1
− (a1 + a2 + a3 − a1 a2 a3 ) = 0
1 2 1
z + a23 z − a23 (a1 + a2 + a3 − a1 a2 a3 ) − (a1 + a2 + a3 − a1 a2 a3 ) = 0
2 2
Tương tự ta có đường thẳng Simson của điểm Q2 :
1 1
z + a22 z − a22 (a1 + a2 + a3 − a1 a2 a3 ) − (a1 + a2 + a3 − a1 a2 a3 ) = 0
2 2
132 Chương 12. Lời giải và gợi ý bài tập

Gọi c1 là nghiệm của hai đường thẳng trên, ta giải hệ bằng cách lấy
phương trình trên trừ dưới và tính nhóm lại còn
1 1
c1 = [a1 +a2 +a3 −a1 a2 a3 −a1 a3 a2 +a2 a3 a1 ] = b2 + (−a1 a2 a3 +a2 a3 a1 )
2 2
Tương tự ta cũng có
1 1
c2 = [a1 +a2 +a3 −a1 a2 a3 −a2 a3 a1 +a1 a3 a2 ] = b3 + (−a2 a3 a1 +a1 a3 a2 )
2 2
1 1
c3 = [a1 +a2 +a3 −a1 a3 a2 −a2 a3 a1 +a1 a3 a3 ] = b1 + (−a1 a3 a2 +a1 a2 a3 )
2 2
1 1
dễ kiểm tra thấy điểm trọng tâm (c1 + c2 + c3 ) = (b1 + b2 + b3 )
3 3
J
. 6.15. Lời giải. Ta có nhãn b1 = −a1 , b2 = −a2 , b3 = −a3 . Thay giá
trị vào phương trình đường thẳng Simson của các điểm
B3 : z + a1 a2 z − (a1 + a2 ) = 0
B2 : z + a1 a3 z − (a1 + a3 ) = 0
Giao điểm của hai đường thẳng trên là c1 , bằng cách trừ phương
trình trên cho dưới.

a1 c1 (a2 − a3 ) − (a2 − a3 ) = 0

a1 c1 − 1 = 0 =⇒ c1 = a1
Tương tự, ta có c2 = a2 , c3 = a3 , thực chất các điểm này trùng với
các đỉnh của tam giác A1 A2 A3 . J
12.6. LỜI GIẢI VÀ GỢI Ý BÀI TẬP CHƯƠNG 7
. 7.6. Lời giải. Lấy đường tròn ngoại tiếp tứ giác làm đơn vị . Khi
đó
1
a1 = (a + b + c − bca)
2
1
a2 = (a + c + d − cda)
2
12.6. Lời giải và gợi ý bài tập chương 7 133

1
h = (a + b + c + d)
2
Ta có
a1 − a2 (a − c)(b − d) a1 − a2
= =
h − a1 ad − bc h − a1
Từ đó suy ra H nằm trên đường thẳng A1 A2 , tương tự ta có H nằm
trên đường thẳng B1 B2 , C1 C2 , D1 D2 .
1 1
Mặt khác, |A1 A2 | = |a1 − a2 | = |(a − c)(b − d)a| = |AC||BD|.
J
2 2
Tương tự cho các đoạn thẳng B1 B2 , C1 C2 , D1 D2 .
. 7.7. Lời giải. (Hình 12.31) Gọi D0 B0
là giao điểm của AN với đường tròn. C
B 0 là điểm cắt của AM đối với đường D
M
tròn. Chọn hệ toạ độ để đường tròn
ngoại tiếp tứ giác làm đơn vị. Do O
\ = 90◦ và DAN \ = 90◦ nên BB 0 D0
BAM A N
và DD0 là đường kính, từ đó suy ra
d0 = d, b0 = b. Nhãn của M theo công B
Hình 12.31.
thức là giao của AB 0 và DC.
(
b + c = n + bcn b+c+d−a
=⇒ n =
( a − d = n − adn bc + ad
a − b = m − abm b+c+d−a
=⇒ m =
d + c = m + dcm dc + ab
n dc + ab n n
Ta có = , dễ dàng kiểm tra thấy = nghĩa là tỷ
m bc + ad m m
n
số là số thực , do đó góc giữa hai đường thẳng ON và OM là 0
m
hoặc π. Chứng tỏ M , N , O thẳng hàng. J
. 7.8. Lời giải. Ta có
a1 + λa2 a2 + λa3 a3 + λa4 a4 + λa1
b1 = , b2 = , b3 = , b4 =
1+λ 1+λ 1+λ 1+λ
134 Chương 12. Lời giải và gợi ý bài tập

Ta biết rằng trọng tâm của tứ giác A1 A2 A3 A4 là


a1 + a2 + a3 + a4
g= . Khi đó
4
a1 + λa2 a2 + λa3 a3 + λa4 a4 + λa1
b1 + b2 + b3 + b4 = + + +
1+λ 1+λ 1+λ 1+λ
= a1 + a2 + a3 + a4
Từ đó suy ra kết luận bài toán. J
. 7.9. Lời giải. Ta gọi góc giữa hai đường thẳng M12 E và A3 A4 là
φ12 , thì
e − m12
φ12 = arg
a3 − a4
Nhưng
a1 + a2 + a3 + a4 a1 + a2 a3 + a4
e − m12 = − =
2 2 2
Ta có
e − m12 (a3 + a4 ) (a3 − a4 ) a3 a3 + a4 a3 − a3 a4 − a4 a4
= =
a3 − a4 2(a3 − a4 ) (a3 − a4 ) 2(a3 a3 − a4 a3 − a3 a4 + a4 a4 )
a3 a4 − a3 a4
=
2(2 − a3 a4 − a3 a4 )
Dễ dàng thấy tỷ số hoàn toàn ảo, nghĩa là
a3 a4 − a3 a4 π
φ12 = arg =±
2(2 − a3 a4 − a3 a4 ) 2
Do đó M12 E ⊥ A3 A4 . Những đường thẳng vuông góc khác chứng
minh tương tự. J
. 7.10. Lời giải. Dễ dàng kiểm tra thấy rằng với những số phức bất
kỳ a1 , a2 , a3 , a4 ta có
(a2 − a1 )(a4 − a3 ) + (a3 − a2 )(a4 − a1 ) = (a4 − a2 )(a3 − a1 )
Lấy môđun hai vế ta nhận được
|a4 − a2 ||a3 − a1 | ≤ |a2 − a1 ||a4 − a3 | + |a3 − a2 ||a4 − a1 |
12.6. Lời giải và gợi ý bài tập chương 7 135

Nghĩa là
|A2 A4 ||A1 A3 | ≤ |A1 A2 ||A3 A4 | + |A2 A3 ||A1 A4 |
Nếu tứ giác A1 A2 A3 A4 nội tiếp đường tròn, thì ta chọn đường tròn
đó làm đơn vị. Khi đó nhãn ở đỉnh có dạng ak = cos φk + i sin φk ,
k = 1, 2, 3, 4. Ta sẽ chỉ ra rằng bất đẳng thức trên là đẳng thức. Ta
xét
a2 − a1 = (cos φ2 + i sin φ2 ) − (cos φ1 + i sin φ1 )
φ2 − φ1 φ1 + φ2 φ1 + φ2
= 2i sin (cos + i sin )
2 2 2
φ2 − φ1
Từ đó suy ra |a2 − a1 | = 2 sin do đó
2
φ1 + φ2 φ1 + φ2
a2 − a1 = |a2 − a1 |i(cos + i sin )
2 2
Ta có
|A1 A2 ||A3 A4 | + |A2 A3 ||A1 A4 | = |a2 − a1 ||a4 − a3 | + |a3 − a2 ||a4 − a1 |
a2 − a1 a4 − a3
= . +
φ1 + φ2 φ1 + φ2 φ3 + φ4 φ3 + φ4
i(cos + i sin ) i(cos + i sin )
2 2 2 2
a3 − a2 a4 − a1
.
φ2 + φ3 φ2 + φ3 φ1 + φ4 φ1 + φ4
i(cos + i sin ) i(cos + i sin )
2 2 2 2
(a2 − a1 )(a4 − a3 ) + (a3 −
=
φ1 + φ2 + φ3 + φ4 φ1 + φ2 + φ3 + φ4
i2 (cos + i sin )
2 2
a4 − a2 a3 − a1
= .
φ2 + φ4 φ2 + φ4 φ1 + φ3 φ1 + φ3
i(cos + i sin ) i(cos + i sin )
2 2 2 2
= |A2 A4 ||A1 A3 | J
. 7.11. Lời giải. Chọn đường tròn ngoại tiếp tứ giác là đường tròn
1
đơn vij, khi đó tâm của tứ giác g = (a1 + a2 + a3 + a4 ). Theo giả
4
136 Chương 12. Lời giải và gợi ý bài tập

o0 + o 1
thiết điểm O0 có nhãn = 2g =⇒ o0 = (a1 + a2 + a3 + a4 ). Gọi
2 2
0
P12 là chân đường vuông góc hạ từ P xuống A1 A2 . Theo công thức
1
p012 = (a1 + a2 + p − a1 a2 p).
2
p + p12
Nhãn p12 của P12 đối xứng của P qua A1 A2 là = p012 . Khi đó
2
p12 = 2p012 − p = a1 + a2 − a1 a2 p

Tương tự, ta có
p34 = a3 + a4 − a3 a4 p
Do R là trung điểm của P12 P34 , ta nhận được
1
r = [a1 + a2 + a3 + a4 − (ap]
2
Tương tự,
1
s = [a1 + a2 + a3 + a4 − (ap]
2
1
t = [a1 + a2 + a3 + a4 − (ap]
2
Ta xét
o0 − r a1 a2 + a3 a4 o0 − r
= =
o0 − s a1 a3 + a2 a4 o0 − s
0
Tỷ số này là một số thực, nghĩa là O , R, S thẳng hàng. Tương
tự ta có O0 , R, T thẳng hàng. J
. 7.12. Lời giải. Lấy đường tròn O làm đường tròn đơn vị , điểm giữa
a1 + a2
của A1 A2 là m12 = . Vì tam giác A1 A2 A3 là tam giác cân có
2
1
trọng tâm trùng với O nên (a12 +a1 +a2 ) = 0 =⇒ a12 = −(a1 +a2 ),
3
chân đường cao hạ từ P12 xuống A3 A4 là
1
p12 = (a3 + a4 + a12 − a3 a12 )
2
1
= (a3 + a4 − a1 − a2 + a3 a4 a1 + a3 a4 a2 ).
2
12.6. Lời giải và gợi ý bài tập chương 7 137

a12 + a012
Điểm A012 đối xứng của A12 qua A3 A4 , do đó = p12 hay là
2
−a1 − a2 + a012 1
= (a3 + a4 − a1 − a2 + a3 a4 a1 + a3 a4 a2 )
2 2
0
a12 = a3 + a4 + a3 a4 a1 + a3 a4 a2 = a3 a4 (a1 + a2 + a3 + a4 ) = a3 a4 δ1
ở đây δ1 = a1 + a2 + a3 + a4 . Từ đó ta có |OA012 | = |a012 | = |δ1 | (vì
|a3 | = |a4 | = 1 và |δ1 | = |δ1 |). Tương tự ta cũng có
|OA013 | = |OA014 | = |OA023 | = |OA024 | = |OA034 | = |δ1 |
Nghĩa là A012 ,A013 A014 , A023 , A024 , A034 nằm trên một đường tròn với
bán kính ρ = |δ1 | mà

ρ = |δ1 | = 4|
δ1
4
| = 4|OG| =⇒ ρ = 4|OG|. J
. 7.13. Lời giải. (Hình 12.32) Chọn hệ tọa độ điểm đầu là tâm
đường tròn và đường tròn ngoại tiếp tứ giác ABCD. Theo giả thiết
nhãn của các
√đỉnh C
1 3
a, b = ( + i)a,
2 2
√ P
D N
1 3
d, c = ( − i)d, O
2 2
B
1 1 1
m = d, n = a, p = (b + c).
2 2 2 M

Chúng ta tính được


√ A
1 1 3
p−m= ( + i)(a − d) Hình 12.32.
2 2 2
Vì vậy |p − m| = |m − n| . Tương tự
cũng có |p − n| = |m − n|. J
. 7.14. Lời giải. (Hình 12.33) Chọn đường tròn là đường tròn đơn
138 Chương 12. Lời giải và gợi ý bài tập

vị . T1 , T2 , T3 lần lượt là điểm tiếp xúc của đường tròn và các cạnh
AB, BC, CD, DA của tứ giác ABCD. D
Gọi hA , hB , hC , hD lần lượt là trực tâm T1
của tam giác tạo ra bởi các đỉnh tương HA
T2
ứng.
A O
Xét tam giác AT1 T4 . Ta thấy OT1 song
T4 C
song với T4 HA và OT4 song song với T3
B
T1 HA . Hình 12.33.

Do đó HA là đỉnh của hình bình hành, hA = t1 + t4 , tương tự ta


có hB = t1 + t2 , hC = t2 + t3 , hD = t3 + t4 .
Ta xét hA − hB = t4 − t2 , điều này chỉ ra rằng HA HB song song
với T2 T4 , mà T2 T4 là đoạn nối hai điểm tiếp xúc của cạnh đối diện.
Tương tự hD − hC = t4 − t2 , và HD HC //T2 T4 suy ra
HA HB //HC HD . Chứng minh tương tự ta có HA HC //HB HD và
song song với đoạn T1 T3 . J
. 7.15. Lời giải. Chọn hệ toạ độ sao cho đường tròn ngoại tiếp tứ
giác ABCD trùng với đường tròn đơn vị. Ký hiệu M là điểm cắt
của hai tiếp tuyến tại các điểm A và C. Khi đó theo công thức có
2ac
m = . Theo điều kiện bài toán M nằm trên đường BD, vậy
a+c
m−b
suy ra tỷ số là số thực.
d−b
2ac 2ac
m−b m−b −b −b
Khi đó = từ đó suy ra a + c = a+c , hoặc
d−b d−b d−b d−b

2ac − b(a + c) 2b − a − c
= d
(a + c)(d − b) −(d − b)(a + c)
Cuối cùng có
2(ac + bd) = (a + c)(b + d)
12.7. Lời giải và gợi ý bài tập chương 8 139

Biểu thức cuối cùng đối xứng giữa a và c tương ứng với b và d.
Như vậy đaa y chính là điều kiện cần và đủ để hai tiếp tuyến các
đỉnh đối diện cắt nhau tại một điểm ở đường chéo. Chúng ta thấy
vai trò của a thay cho b, c thay cho d và ngược lại. J
12.7. LỜI GIẢI VÀ GỢI Ý BÀI TẬP CHƯƠNG 8
. 8.6. Lời giải. (Hình 12.34) Chọn C
đường tròn nội tiếp tam giác ABC
làm đơn vị . Khi đó
2b1 c1 A1
a=
b1 + c1
, B1 N
2c1 a1 O
b= ,
c1 + a1
2a1 b1
c= . A C1 M B
a1 + b1
Hình 12.34.
Dây cung A1 B1 cắt đường kính
qua C1 tại N , vậy

(c1 − c1 )a1 b1 − (a1 + b1 a1 (a1 + b1 )


n= =
a1 b1 − c1 (−c1 ) a1 b1 + c21
Chỉ còn kiểm tra C, N và M là điểm giữa của AB nằm trên một
đường thẳng. Nhưng
1 b1 c1 c1 a1
m = (a + b) = +
2 b1 + c1 c1 + a1
Ta có
c21 (a1 + b1 ) 2a1 b1
n−c= −
c21 + a1 b1 a1 + b1
c2 (a2 + b21 ) − 2a21 b21
= 1 1
(a1 + b1 )(c21 + a1 b1 )
140 Chương 12. Lời giải và gợi ý bài tập


b1 c1 c1 a1 2a1 b1
m−c= + −
b1 + c1 c1 + a1 a1 + b1
b1 c1 a1 b1 c1 a1 a1 b1
= − + −
b1 + c1 a1 + b1 c1 + a1 a1 + b1
b21 (c1 − a1 ) a21 (c1 − b1 )
= +
(b1 + c1 )(a1 + b1 ) (a1 + c1 )(a1 + b1 )
c21 (a21 + b21 ) − 2a21 b21
=
(a1 + c1 )(a1 + b1 )(b1 + c1 )
n−c (a1 + c1 )(b1 + c1 )
Do đó = là số thực. Điều này suy ra từ tính
m−c c21 + a1 b1
chất a1 , b1 , c1 nằm trên đường tròn đơn vị. Vậy C, N, M thẳng hàng.
J
. 8.7. Lời giải. (Hình 12.35) Chọn C
hệ toạ độ và ký hiệu như bài toán
trước, ta có A1
1 B1
m = (b + c) M
2 Q
c1 a1 a1 b1 O
= +
c1 + a1 a1 + b1
a1 (2c1 b1 + a1 b1 + a1 c1 ) A C1 B
=
(a1 + b1 )(c1 + a1 ) Hình 12.35.
và nếu Q là điểm giữa của AA1 thì
1 b1 c1 1 (2c1 b1 + a1 b1 + a1 c1 )
q = (a + a1 ) = + a1 =
2 b1 + c1 2 2(b1 + c1 )
m
Kiểm tra Q nằm trên OM , nghĩa là kiểm tra xem có là số thực
q
không ?. Thật vậy,
m 2a1 (b1 + c1 )
=
q (a1 + b1 )(a1 + c1 )
12.7. Lời giải và gợi ý bài tập chương 8 141

Đây đúng là một số thực , do tính chất a1 , b1 , c1 nằm trên đường


tròn đơn vị. J
. 8.8. Lời giải. (Hình 12.36) Chọn hệ toạ độ vuông góc sao cho lấy
2t
đường tròn làm đơn vị và b = 1. Khi đó a = −1, t 6= ±1 và p = .
1 +t
2
 
1 1 1 t +1
Rõ ràng h = (t + t) = t+ = .
2 2 t 2t
1 3t2 + 1
Ký hiệu M là trung điểm của T H, ta có m = (t + h) = .
2 4t
Ta sẽ kiểm tra A, P và M nằm trên P
đường thẳng, thật vậy
3t2 + 1
m−a +1 (t + 1)2
= 4t = . T
p−a 2t 4t
+1
1+t
M
Dễ dàng kiểm tra tỷ số này trùng A B
H
với tỷ số liên hợp của nó, suy ra tỷ số
là một số thực , nghĩa là A, P, M thẳng
hàng. J Hình 12.36.

. 8.9. Lời giải. (Hình 12.37) Chọn đường tròn nội tiếp tam giác là
C
đường tròn đơn vị. A1 , B1 , C1 lần
lượt là điểm tiếp xúc của đường A1
tròn với cạnh BC, CA, AB. S
M
Đường phân giác từ A đi qua B1 L
O. Ký hiệu K, L là hai điểm do O
đường phân giác từ A cắt đường K
tròn thì k 2 = l2 = b1 c1 và giao A C1 N B
điểm của A1 B1 với KL là Hình 12.37.
142 Chương 12. Lời giải và gợi ý bài tập

(k + l)a1 b1 − (a1 + b1 )kl


s=
a1 b1 − kl
(a1 + b1 )b1 c1 (a1 + b1 )c1
= =
(a1 b1 + b1 c1 ) (a1 + c1 )

vì k + l = 0 và kl = −b1 c1 .
Trung điểm M và N của BC và AB có nhãn

1 a1 c1 a1 b1
m = (b + c) = +
2 a1 + c1 a1 + b1

1 b1 c1 a1 c1
n = (a + b) = +
2 b1 + c1 a1 + c1
Khi đó
a1 c1 a1 b1 (a1 + b1 )c1
+ −
m−s a + c1 a1 + b1 a1 + c1
= 1
n−s b1 c1 a1 c1 (a1 + b1 )c1
+ −
b1 + c1 a1 + c1 a1 + c1
a1 b1 b1 c1

a + b1 a1 + c1 (a2 − b1 c1 )(b1 + c1 )
= 1 = 1
b1 c1 b1 c1 c1 (a21 − b21 )

b1 + c1 a1 + c1

Dễ dàng kiểm tra tỷ số trên trùng với số liên hợp của nó. Vậy nó
là số thực, nghĩa là các điểm M , N và S thẳng hàng. Suy ra đường
A1 B1 , M N và phân giác đỉnh A của tam giác cắt nhau tại một điểm.
J
. 8.10. Lời giải. (Hình 12.38) Chọn J1 làm đường tròn đơn vị. E,
F , D lần lượt là điểm tiếp xúc của đường tròn với cạnh AC, CB,
BA. Ta có
12.7. Lời giải và gợi ý bài tập chương 8 143

2ed 2df 2ef C


a= ,b = ,c = .
e+d f +d e+f
Đường thẳng N
F
ED : e + d = z + edz,
CO : cz − cz = 0, E
BO : bz − bz = 0.
Điểm M là giao của ED và CO, A D M B
vậy
c 2ef Hình 12.38.
m= m=
c e+f
e+f 1
m= m
2ef ef
1 f +d f (e + d)
suy ra e + d = m + ed m = m =⇒ m =
ef f f +d
Điểm N là giao của ED và BO, vậy
b 2df d + f 1
n= n= n= n
b d + f 2df df

1 f +e f (e + d)
e + d = n + ed n=n =⇒ n =
df f f +e
Dùng các công thức trên tính và biến đổi tỷ số sau cho ta kết quả
n−b n−c (d − f )(e + f )
V (n, m, b, c) = : =
m−b m−c (d + f )(e − f )
Do e, f , d nằm trên đường tròn đơn vij, dễ dàng nhận thấy tỷ số trên
là một số thực . Vậy M , N , B, C nằm trên một đường tròn. J
. 8.11. Lời giải. Chọn đường tròn nội tiếp tứ giác làm đơn vị, các
điểm tiếp xúc lần lượt T1 , T2 , T3 , T4 trên các cạnh AB, BC, CD,
DA. M, N là trung điểm BD, AC. Ta có
2t1 t4 2t1 t2 2t2 t3 2t3 t4
a= ,b = ,c = ,d =
t1 + t4 t1 + t2 t2 + t3 t3 + t4
144 Chương 12. Lời giải và gợi ý bài tập

2t1 t4 2t2 t3
+
t + t4 t2 + t3 t1 t4 t2 t3
Tính n = 1 = +
2 t1 + t4 t2 + t3
t1 t2 t3 t4
Tương tự có m = +
t1 + t2 t3 + t4
t1 t2 t3 t4
+
o−m t + t2 t3 + t4 (t1 + t4 )(t2 + t3 )
V (o, m, n) = = 1 =
o−n t t
1 4 t t
2 3 (t1
+
t1 + t4 t2 + t3
Tỷ số đow n là một số thực, suy ra O, M, N nằm trên một đuwờng
thẳng. J
B L
. 8.12. Lời giải. (Hình 12.39) Chọn
C
đường tròn nội tiếp tứ giác là đường
tròn đơn vị . Vậy K
2kn 2lm
a= ,c = S
k+n l+m A
M
2kl 2nm
b= ,d = N
k+l n+m D
Hình 12.39.
và gọi S là giao của KM và N L thì
(n + l)km − (k + m)nl
s=
km − nl
Xét
(n + l)km − (k + m)nl 2nk
s−a −
V (s, a, c) = = km − nl k+n
s−c (n + l)km − (k + m)nl 2ml

km − nl m+l
n−km+l
=
m−ln+k
Dễ dàng, chỉ ra rằng tỷ số trên là số thực (vì do n, k, m, l nằm trên
đường tròn đơn vị ) suy ra S, A, C thẳng hàng. Nghĩa là đường chéo
12.7. Lời giải và gợi ý bài tập chương 8 145

AC đi qua giao điểm của KM và N L. Tương tự, ta cũng có


n−mm+n
V (s, b, d) =
k−l k+l
cũng là số thực và S, B, D thẳng hàng. J
. 8.13. Lời giải. Chọn đường tròn ngoại tiếp tam giác ABC làm
đơn vị. Điểm P nằm trên BC và trên đường tiếp tuyến tại A, do đó
p + bcp = b + c, p + a2 p = 2a. Từ đó suy ra
a2 (b + c) − 2abc
p=
a2 − bc
Chú ý nếu a2 = bc thì tiếp tuyến tại A song song với BC.
Tương tự ta có
b2 (c + a) − 2abc c2 (a + b) − 2abc
q= , r =
b2 − ca c2 − ab
Khi đó
a2 (b + c) − 2abc b2 (c + a) − 2abc
p−q 2 − bc

= 2 a b2 − ca
p−r a (b + c) − 2abc c2 (a + b) − 2abc

a2 − bc c2 − ab
(b − a)(a2 b2 + b2 c2 + c2 a2 − a2 bc − b2 ca − c2 ab)(c2 − ab)
=
(c − a)(a2 b2 + b2 c2 + c2 a2 − a2 bc − b2 ca − c2 ab)(b2 − ca)
(b − a)(c2 − ab)
=
(c − a)(b2 − ca)
Dễ dàng kiểm tra tỷ số trên là một số thực, vậy P, Q, R nằm trên
một đường thẳng. J
. 8.14. Lời giải. (Hình 12.40) Bài này tuy dùng tính chất đường
phân giác và đường tròn nội tiếp tam giác, nhưng ta chọn đường
tròn đơn vị là đường tròn ngoại tiếp tam giác đã cho A1 A2 A3 . Vì
B1 là điểm giữa của cung A2 A3 nên ta có b21 = a2 a3 tương tự có
b22 = a3 a1 , b23 = a1 a2 . Suy ra
146 Chương 12. Lời giải và gợi ý bài tập

(b2 b3 )2 = b22 b23 = a3 a1 a1 a2


A3
= a21 a2 a3 = a21 b21 = (a1 b1 )2
B1
Từ đó suy ra b2 b3 = ±a1 b1 . Nhưng
nếu b2 b3 = a1 b1 thì A1 B1 và B2 B3 B2

song song, điều này không thể được, I


vì B2 và B3 nằm hai phía khác nhau A1 A2
B3
của A1 B1 . Suy ra còn b2 b3 = −a1 b1 ,
điều này nói lên rằng A1 B1 và B2 B3 Hình 12.40.
vuông góc với nhau. J
. 8.15.Lời giải. Tương tự như bài trên chúng ta chọn đường tròn
ngoại tiếp tam giác ABC làm đơn vị. Khi đó b21 = ca, c21 = ab,
n = b1 + c1 . Tâm của đường tròn nội tiếp là điểm cắt của hai dây
BB1 và CC1 , nghĩa là

(c + c1 )bb1 − (b + b1 )cc1 (c + c1 )bb1 − (b + b1 )cc1 a


j= =
bb1 − cc1 bb1 − cc1 a
(a + c1 )b1 c21 − (b + b1 )b21 c1 (c + c1 )c1 − (b + b1 )b1
= =
b1 c21 − c1 b21 c1 − b1 )

(Nhãn của J ký hiệu là j không thì lẫn với đơn vị ảo!). Khi đó

(c + c1 )c1 − (b + b1 )b1
b1 + c1 −
n−j c1 − b1 bb1 − cc1
= =
c−b c−b (c − b)(c1 − b1 )
1 1 1 1
. − .
n−j b b1 c c1 cc1 − bb1 n−j
= = =−
c − b ( 1 − 1 )( 1 − 1 ) (b − c)(b1 − c1 ) c−b
c b c1 b1

Từ đó suy ra IN và BC vuông góc với nhau. J


12.8. Lời giải và gợi ý bài tập chương 9 147

12.8. LỜI GIẢI VÀ GỢI Ý BÀI TẬP CHƯƠNG 9


. 9.6. Lời giải. Từ sự đồng dạng của ABC và A1 B1 C1 ta có
c−a c1 − a1
k= =
b−a b1 − a1
vậy c − a = k(b − a) và c1 − a1 = k(b1 − a1 ). Còn những điểm A0 ,
B0 , C0 ta có
a + λa1 b + λb1 c + λc1
a0 = , b0 = , c0 =
1+λ 1+λ 1+λ
với λ là tỷ số đã cho. Khi đó
c + λc1 a + λa1
c0 − a0 −
= 1+λ 1 + λ = (c − a) + λ(c1 − a1 )
b0 − a0 b + λb1 a + λa1 (b − a) + λ(b1 − a1 )

1+λ 1+λ
k(b − a) + λk(b1 − a1 )
= = k.
(b − a) + λ(b1 − a1 )

. 9.7. Lời giải. Chọn điểm gốc là M , vậy m = 0 và


a0 = a1 − a, b0 = b1 − b, c0 = c1 − c
Từ sự đồng dạng của tam giác ABC và A1 B1 C1 ta có
c−a c1 − a1
= =k
b−a b1 − a1
Từ đó suy ra c − a = k(b − a) và c1 − a1 = k(b1 − a1 ). Khi đó
c0 − a0 c1 − a1 − (c − a) k(b1 − a1 ) − k(b − a)
= = = k.
b0 − a0 (b1 − a1 ) − (b − a) (b1 − a1 ) − (b − a)
J
. 9.8. Lời giải. Do A1 B1 C1 ' A2 B2 C2 ' A3 B3 C3 suy ra tồn tại một
số λ sao cho
c1 − a1 c2 − a2 c3 − a3
= = =λ
b1 − a1 b2 − a2 b3 − a3
148 Chương 12. Lời giải và gợi ý bài tập

Khi đó c1 = λb1 +(1−λ)a1 , c2 = λb2 +(1−λ)a2 , c3 = λb3 +(1−λ)a3 ,


từ đó suy ra
c3 − c1 λb3 + (1 − λ)a3 − λb1 − (1 − λ)a1
=
c2 − c1 λb2 + (1 − λ)a2 − λb1 − (1 − λ)a1
λ(b3 − b1 ) + (1 − λ)(a3 − a1 )
=
λ(b2 − b1 ) + (1 − λ)(a2 − a1 )
Mặt khác, do A1 A2 A3 ' B1 B2 B3 ta có
a3 − a1 b3 − b1
= =µ
a2 − a1 b2 − b1
Từ đó suy ra a3 − a1 = µ(a2 − a1 ) và b3 − b1 = µ(b2 − b1 ). Do đó
c3 − c1 λµ(b2 − b1 ) + (1 − λ)µ(a2 − a1 )
= =µ
c2 − c1 λ(b2 − b1 ) + (1 − λ)(a2 − a1 )
Điều này nghĩa là C1 C2 C3 đồng dạng với A1 A2 A3 , B1 B2 B3 và cùng
hướng. J
. 9.9. Lời giải. Chọn hệ toạ độ sao cho đường tròn ngoại tiếp tam
1
giác A1 A2 A3 là đơn vị . Khi đó h = a1 + a2 + a3 , g = (a1 + a2 + a3 ),
3
1 1 1
g1 = (a1 +2a2 +2a3 ), g2 = (2a1 +a2 +2a3 ), g3 = (2a1 +2a2 +a3 ).
3 3 3
Nhưng
g3 − g1 a3 − a1
= ,
g2 − g1 a2 − a1
nghĩa là tam giác G1 G2 G3 đồng dạng với A1 A2 A3 .
g − g1 a2 + a3
Ngoài ra = là một số hoàn toàn phức, hay là
g2 − g3 a2 − a3
G1 G vuông góc với G2 G3 . Tương tự, G2 G vuông góc với G3 G1 và
G3 G vuông góc với G1 G2 . Vậy là G là trực tâm tam giác G1 G2 G3 .
J
. 9.10. Lời giải. Do C là trung điểm của AM , ta có a + m = 2c, vì
ABCL là hình bình hành, nên l = a + c − b. Mặt khác từ giả thiết
12.8. Lời giải và gợi ý bài tập chương 9 149

ABC và ACN đồng dạng , cùng hướng, ta có


n−a c−a
=
c−a b−a
(c − a) 2
từ đó suy ra n = a + . Do đó
b−a
(c − a)2 (c − a)(a + c − 2b (c − a)(l − b)
n−m = a+ +(2c−a) = =
b−a b−a b−a
Vì N M BS là hình bình hành, nên n − m = s − b hay là s − b =
(c − a)(l − b) s−b c−a
hoặc = . Nghĩa là tam giác ABC ' BLS.
J
b−a l−b b−a

. 9.11. Lời giải. Chọn hệ toạ độ vuông góc có gốc tại C, do tính
đồng dạng và tam giác vuông ABC và DEC, ta có b = ia, e = λd
với λ là một số thực . Suy ra b − e = λ(d − a)i vậy AD và BE vuông
góc với nhau. J
. 9.12. Lời giải. Chọn hệ toạ độ có gốc tại A, do tính chất đồng
dạng của chúng, ta có b = c1 (1 + λi), b1 = c(1 − λi), với λ là số thực.
Ta viết phương trình BC1 và B1 C như sau
(b − c1 )m − (b − c1 )m + bc1 − bc1 = 0,
(c − b1 )m − (c − b1 )m + cb1 − cb1 = 0.
Cộng hai đẳng thức trên lại và thế giá trị b, b1 theo c, c1 . Ta có
m(c − c1 ) + m(c − c1 ) = 0
Điều này chỉ ra rằng AM và CC1 vuông góc với nhau. J
. 9.13. Lời giải. Chọn điểm gốc là trọng tâm S của tam giác ABC,
vì là tam giác đều, nên đặt w = cos 60◦ + i sin 60◦ , ta có bw2 = a
[ = 120◦ ).
(ASB
Mặt khác, ta có (b − c)w = a − c và (a − c)w = d − c, từ đó
suy ra (b − c)w = a − d = bw2 − d hay là b(w2 − w) = d − ew và vì
150 Chương 12. Lời giải và gợi ý bài tập

b+d
w2 −w+1 = 0, ta nhận được b+d = ew. Nhưng a+e = bw2 + =
w
bw3 + b + d d 1 1
= vì w3 = −1. Do đó m = (b + d) = ew và
w w 2 2
1 d |m| |e|
n = (a + c) = vậy, = , còn góc M
\ SE và N
\ SD bằng
2 2w |n| |d|

nhau 60 , nhưng khác hướng. Do đó tam giác M SE và tam giác
N SD đồng dạng. J
. 9.14. Lời giải. (Hình 12.41) Chọn
C1
hệ toạ độ sao cho đường tròn ngoại A3
B2
tiếp tam giác A1 A2 A3 là đường tròn
đơn vị . Đường thẳng qua A1 , A2 , B3
A3 tạo cùng một góc với các cạnh C2 A2
A1 C3
A2 A3 , A3 A1 , A1 A2 cắt đường tròn
B1
ngoại tiếp lần thứ 2 tại B1 , B2 , B3
tương ứng. Hình 12.41.
b3 − a3 b2 − a2
Khi đó số , cần phải là số thực vì hai tỷ số có
a1 − a2 a3 − a1
cùng argumen. Suy ra
b3 − a3 b2 − a2 b3 − a3 b2 − a2
: = :
a1 − a2 a3 − a1 a1 − a2 a3 − a1
Từ đó ta có a22 b2 = a23 b3 . Tương tự ta có a21 b1 = a22 b2 = s, ở đây s
là một số phức nào đó với môdd un 1. Gọi C1 , C2 , C3 là những giao
điểm của A2 B2 và A3 B3 ; A3 B3 và A1 B1 ; A1 B1 và A2 B2 ta có
(a3 + b3 )a2 b2 − (a2 + b2 a2 a3
c1 =
a2 b2 − a3 b3 a2 a3
s[(a3 + b3 )a3 − (a2 + b2 ) a3 b3 − a2 b2
= = a2 + a3 +
s(a3 − a2 ) a3 − a2
2 2
a3 sa3 − a2 sa2
= a2 + a3 + = a2 + a3 − sa2 a3
a3 − a2
12.8. Lời giải và gợi ý bài tập chương 9 151

Tương tự c2 = a3 + a1 − sa3 a1 và c3 = a1 + a2 − sa1 a2 . Do đó


c3 − c1 a1 + a2 − sa1 a2 − a2 − a3 + sa2 a3 a1 a2 a3
=
c2 − c1 a3 + a1 − sa3 a1 − a2 − a3 + sa2 a3 a1 a2 a3
(a1 − a3 )(a1 a2 a3 + s) a3 − a1
= =
(a1 − a2 )(a1 a2 a3 + s) a2 − a1
Vậy tam giác C1 C2 C3 đồng dạng với A1 A2 A3 . Ta có
|h − c1 | = |a1 + sa2 a3 | = |a1 a2 a3 + s| và tương tự
|h − c2 | = |h − c3 | = |a1 a2 a3 + s|, nghĩa là trực tâm H của tam giác
A1 A2 A3 cách đều các đỉnh C1 , C2 , C3 . Vì vậy đó chính là tâm đường
tròn ngoại tiếp tam giác C1 C2 C3 . J
. 9.15.Lời giải. (Hình 12.42) Chọn
D
đường tròn đã cho làm đơn vị và p =
1. AP cắt đường tròn tại Q. Ta có C
2bc
p + q = a + pqa. Vì a = ,p=1 O
R
b+c
2bc − b − c S1
S2
nên ta có q = . Điểm R
b+c−2 A Q P
là đối xứng với Q qua tâm O, suy ra
2bc − b − c B
r = −q = . Điểm D và E E
2−b−c
là giao điểm của đường thẳng P R và Hình 12.42.
đường kính đi qua B và C. Do đó

2bc − b − c
b2 (1
+ r) b2 (1 + )
d= = 2−b−c
r + b2 2bc − b − c
+ b2
2−b−c
2b2 (b − 1)(c − 1) 2b2 (c − 1)
= = −
−(b + c)(b − 1)2 (b + c)(b − 1)
2
2c (b − 1)
e=−
(b + c)(c − 1)
152 Chương 12. Lời giải và gợi ý bài tập

4b2 c2 d a
Khi đó de = 2
= a2 và suy ra = , nghĩa là OAD ' OEA.
(b + c) a e
Ký hiệu S1 S2 là đường kính qua A. Vì S1 và S2 là trung điểm của
_
BC, nên s21 = s22 = bc. Do đó
1 1
m = (s1 + s2 + d − s1 s2 d) = (d + s21 d)
2 2
2
b (b − c) c−1 c−1
= −[ + bc ] = −b
(b + c)(b − 1) (b + c)(b − 1) b−1
b−1
Tương tự có n = −c hoặc là mn = bc = s22 . Do đó
c−1
|OM ||ON | = |s22 | = 1, và vì M , N nằm trên một tia bắt đầu từ
O, suy ra nó nghịch đảo với nhau đối với đuwờng tròn đã cho. J
12.9. LỜI GIẢI VÀ GỢI Ý BÀI TẬP CHƯƠNG 10
. 10.6. Lời giải. (Hình 12.43) Chọn
đường tròn ngoại tiếp là đường tròn
đơn vị và vị trí tam giác sao cho a = 1,
b = τ , c = τ 2 . Đường thẳng qua P ,
song song với BC cắt đường tròn đơn
vị tại X, vì vậy px = bc =⇒ x = p.
Tương tự đường thẳng qua P song
song với CA và AB cắt đường tròn
đơn vị tại Y và Z, y = τ 2 p và z = τ p. Hình 12.43.
Suy ra
(c + a)px − (p + x)ca 1 + τ 2 − (p + p)τ 2
m= =
px − ca 1 − τ2
(a + b)py − (p + y)ab (1 + τ )τ 2 − (p + τ 2 p)τ
n= =
py − ab τ3 − τ
2
(1 + τ )τ − (p + τ p)
=
τ2 − 1
12.9. Lời giải và gợi ý bài tập chương 10 153

(b + c)pz − (p + z)bc (τ + τ 2 )τ − (p + τ p)
q= =
pz − bc τ −1
m−n
Với các giá trị trên kiểm tra tỷ số là một số thực , vậy ba
n−q
điểm thẳng hàng. J
. 10.7. Lời giải. Ta chọn O là điểm gốc của hệ toạ độ vuông góc
ta chọn O. Khi đó c1 = b1 − a1 , c2 = b2 − a2 , c3 = b3 − a3 và
vì a3 = a1 + w(a2 − a1 ) và b3 = b1 + w(b2 − b1 ), do đó suy ra
c3 = c1 + w(c2 − c1 ). J
. 10.8. Lời giải. Chọn hệ toạ độ sao cho A1 , A2 , A3 , A4 , A5 với nhãn
a1 , a2 ,a3 , a4 , a5 là nghiệm của phương trình x5 = 1, ta có
1
mk = (ak + ak+1 + m − ak ak+1 m)
2
1
nk = (ak + ak+1 + n − ak ak+1 n), a6 = a1
2
Suy ra
5 5
X 5 X
(nk − mk ) = (n − m) − (n − m) ak ak+1
2
k=1 k=1

Nhưng a1 = 1, a2 = τ , a3 = τ 2,a4 = τ 3, a5 = τ 4 và a1 a2 + a2 a3 +
a3 a4 + a4 a5 + a5 a1 = τ + τ 3 + τ 5 + τ 7 + τ 4 = 1 + τ + τ 2 + τ 3 + τ 4 =
5
τ5 − 1 −−−→ 5 −−→
J
X
= 0. Từ đó suy ra Mi N i = M N .
τ −1 2
i=1
. 10.9. Lời giải. Chọn hệ toạ độ vuông góc sao cho a = −1 − i,
b = 1 − i, c = 1 + i, d = −1 + i. Do đó
k = −1 − i + 2w kl = −i + (1 + i)w
l = 1 − i + 2iw lm = 1 + (i − 1)w
n = −1 + i − 2iw mn = i − (i + 1)w
m = 1 + i − 2w kl = −1 + (1 − i)w
154 Chương 12. Lời giải và gợi ý bài tập

ak = −1 − i + w bk = −i + w
bl = 1 − i + iw cl = 1 + iw
cm = 1 + i − w dm = i − w
dn = −1 + i − iw an = −1 − iw

Tâm của đa giác đều 12 cạnh này trùng với điểm đầu hệ tọa độ ta
đã chọn. J
. 10.10.Lời giải.

c1 = a1 + w(b1 − a1 )
c2 = a2 + w(b2 − a2 )
c3 = a3 + w(b3 − a3 )
a3 = a1 + w(a2 − a1 )

1 3
ở đây w = + i , hoặc w là nghiệm của phương trình bậc hai
2 2
2
w − w + 1 = 0. Xét số sau
c1 + b2 + w(c2 + b3 − c1 − b2 ) =
= a1 +w(b1 −a1 )+b2 +w[a2 +w(b2 −a2 )+b3 −a1 −w(b1 −a1 )−b2 ] =
(1−2w+w2 )a1 +(w−w2 )b1 +(w2 −w+1)b2 +(w−w2 )a2 +wb3 =
−wa1 + b1 + a2 + wb3
Mặt khác,

c3 + b1 = a3 + w(b3 − a3 ) + b1
= a1 + w(a2 − a1 ) + wb3 − w[a1 + w(a2 − a1 )] + b1
= −wa1 + a2 + wb3 + b1

Do đó
c3 + b1
2
=
c1 + b2
2
+ w(
c2 + b3 c1 + b2
2

2
) J
12.9. Lời giải và gợi ý bài tập chương 10 155

. 10.11. Lời giải. Từ giả thiết ta có


a3 = a1 + w(a2 − a1 ),
b3 = b1 + w(b2 − b1 ),
Ngoài ra
c1 = a1 + w(b1 − a1 ),
c2 = a2 + w(b2 − a2 ),
c3 = a3 + w(b3 − a3 ).
Do đó
c3 = a3 + w(b3 − a3 )
= a1 + w(a2 − a1 ) + w[b1 + w(b2 − b1 ) − a1 − w(a2 − a1 )]
= a1 + w(a2 − a1 ) + w[b1 + w(b2 − a2 ) − a1 − w(b1 − a1 )]
= c1 + w(c2 − c1 )
J
. 10.12.Lời giải. Từ điều kiện đều kiện đề bài suy ra góc của tam
π 2π 4π
giác là , , . Vì thế ta chọn hệ toạ độ sao cho a = 1, b = α2 ,
7 7 7
2π 2π
c = α3 , ở đây α = cos + i sin . Trung điểm A1 , B1 , C1 của
7 7
1 1
các cạnh BC, CA, AB tương ứng là a1 = (b + c) = (α2 + α3 ),
2 2
1 3 1 2
b1 = (1 + α ), c1 = (1 + α ), còn chân các đường cao a2 , b2 , c2
2 2
tính theo công thức đã biết
1 1
a2 = (α2 + α3 + 1 − α2 α3 ) = (1 + α2 + α3 − α5 )
2 2
1 1
b2 = (1 + α3 + α2 − α−2 α3 ) = (1 + α2 + α3 − α)
2 2
1 1
c2 = (1 + α2 + α3 − α2 α−3 ) = (1 + α2 + α3 − α6 )
2 2
156 Chương 12. Lời giải và gợi ý bài tập

1
E là tâm đường tròn 9 điểm có nhãn e = (1 + α2 + α3 ). Khi đó,
2
1 1 1 2 1 1
e − a1 = , e − b2 = α, e − b1 = α , e − c1 = α3 , e − a2 = α5 ,
2 2 2 2 2
1 6
e − c2 = α , điều này chỉ ra rằng những điểm trên là đỉnh của một
2
đa giác đều 6 cạnh, có tâm E và bán kính đuwờng tròn ngoại tiếp
1
là .
2
J
. 10.13. Lời giải. (Hình 12.44) Ta có
b1 = a2 + w(a3 − a2 ) và
b2 = a3 + w(a1 − a3 ) , do đó
a2 + a3 + b1
m1 =
3
(2 − w)a2 + (1 + w)a3
=
3
a3 + a1 + b2
m2 =
3 Hình 12.44.
(2 − w)a3 + (1 + w)a1
=
3
a1 + a2
Vì m = , nên
2
a1 + a2 (2 − w)a2 + (1 + w)a3
m − m1 = −
2 3
1
= [3(a1 − a3 ) + (2w − 1)(
6
1 √
= [3(a1 − a3 ) + i 3(a2 − a3 )],
6
a1 + a2 (2 − w)a3 + (1 + w)a1
m − m2 = −
2 3
1
= [(1 − 2w)(a1 − a3 ) + 3(
6
1 √
= [−i 3(a1 − a3 ) + 3(a2 − a3 )],
6
12.9. Lời giải và gợi ý bài tập chương 10 157

a2 − a3
Ta đặt z = . Khi đó
a1 − a3

|M M1 | |m − m1 | 3(a1 − a3 ) + i 3(a2 − a3 ) z−α
= = √ =
|M M2 | |m − m2 | −i 3(a1 − a3 ) + 3(a2 − a3 ) 1 − αz

3
ở đây α = i . Vì |α| < 1, bằng cách biến đổi chúng ta có thể dễ
3
dàng chứng minh được
z−α
1 − αz
≤ 1 khi và chỉ khi |z| ≤ 1. J
. 10.13.Lời giải. Ta chọn điểm đầu của hệ toạ độ là R, nghĩa là
r = 0. Để giải bài tập chúng ta phải chứng minh q = ip. Nếu U là
|CU | √
chân đường vuông góc hạ từ Q xuống AC thì ta có = 3. Suy
√ |U A|
c + 3a
ra u = √ , từ đó có
1+ 3

q = u + (a − u)(−i)
√ √
c + 3a c + 3a
= √ − i(a − √ )
1+ 3 1+ 3

(1 + i)c + ( 3 − i)a
= √
1+ 3
Tương tự ta có

(1 + i)c + ( 3 − i)b
p= √
1+ 3
chỉ còn chứng minh rằng
√ √
(1 + i)c + ( 3 − i)b (1 + i)c + ( 3 − i)a
√ i= √
1+ 3 1+ 3

a 3 1
Nhưng đẳng thức trên tương đương với = − + i là đúng, vì
b 2 2
tam giác RAB là cân với đỉnh R và có góc ở đỉnh là 150◦ . J
158 Chương 12. Lời giải và gợi ý bài tập

12.10. LỜI GIẢI VÀ GỢI Ý BÀI TẬP CHƯƠNG 11


. 11.6. Lời giải. Chọn điểm A là gốc toạ độ . Khi đó SABC =
1 1
Im(cb), còn SBCM N = Im(cb + nc + mn + bm). Nhưng m =
2 2
1 1 1 1 1
(c + d) = b + c và n = c + b. Khi đó
2 2 4 2 4

1 1 1 1 1 1 1 1 1
SBCM N = Im(cb + ( c + b)c + ( b + c)( c + b) + b( b + c)
2 2 4 2 4 2 4 2 4
1 1 1 1 1
= Im(cb + bc + bc + cb + bc)
2 4 4 16 4
1 5 3
= Im( cb + (cb + bc))
2 16 4
1
2 16
5 5
= Im cb = SABC
16
J

. 11.7. Lời giải. (Hình 12.45) Chọn điểm gốc của hệ toạ độ là điểm
1 1 2 1
B. Khi đó n = a, p = c, q = c, m = (a + c).
2 3 3 2
2 2 2 32 2 3
Vì a + c = a + c= a+ q
5 5 5 53 5 5
2 2
nên k = a + c, và
5 5
1 1 2 1 3 1 2 3
a + c = ( a) + ( c) = n + p,
5 5 5 2 5 3 5 5
1 1
nên l = a + c (do phương trình
5 5 Hình 12.45.
thông số suy ra được). Do đó
12.10. Lời giải và gợi ý bài tập chương 11 159

1
SQP LK = Im(qp + kq + lk + pl)
2
1 2 2 1 1 1
= Im( (a + c) c + c( a + c))
2 5 3 2 5 5
1 1 1 1
= Im( ac + (ac + ac)) = SABC .
2 5 15 5
. 11.8. Lời giải. (Hình 12.46) Chọn C là điểm gốc của hệ toạ độ.
a m
Khi đó b = 2d, m = λd, 1 ≤ λ ≤ 2. Theo định lý Talet = = λ,
n d
a
từ đó suy ra n = . Khi đó C
λ
1
SABM N = Im(ba + mb + nm + an)
2 D
1 a M
= Im(2da + (λb)) N
2 λ
1 1
= Im(da) = SABD = SABC A B
2 2
hoặc là SABM N không phụ thuộc vào λ. Hình 12.46. J
. 11.9. Lời giải. Chọn hệ toạ độ vuông góc sao cho đường tròn
ngoại tiếp tam giác ABC là đơn vị . Khi đó ta tính được c1 = −abc,
b1 = −cab, a1 = −bca và
1
SAC1 BA1 CB1 = Im(c1 a + bc1 + ca1 + b1 c + ab1 )
2
1
= Im(−2(bc + ca + ab))
2
1
= Im(2(ba + cb + ac)) = 2SABC
2
. 11.10.Lời giải. Theo cách dựng của bài toán thì A0 , B 0 , C 0 nằm
trên đường tròn ngoại tiếp tam giác ABC, vì có góc ABA \0 =
\0 = 90◦ ,... và mặt khác AA0 , BB 0 , CC 0 là đường kính của đường
ACA
tròn , do đó a0 = −a, b0 = −b, c0 = −c. Khi đó
1
SAC 0 BA0 CB 0 = Im(−ca − bc − ab − ca − bc − ab) = 2SABC
2
J
160 Chương 12. Lời giải và gợi ý bài tập

. 11.11. Lời giải. (Hình 12.47) Đặt a = −1, b = 1. Khi


1
đó e = (c + d − 1 + cd), D F
2
1 E C
f = (c + d + 1 − cd)
2

1
S ABF E = Im(ba + f b + ef + ae) A O B
2
1 1
= Im( (c + d + 1 − cd)
2 2
1
+ (c + d − 1 + cd)(c + d + 1 − cd) Hình 12.47.
4
1
− (c + d − 1 − cd))
2
1 1 1
= Im(c + d − c − d) = Im(c − c) + (d − d) = SABC + SABD
2 2 2
. 11.12. Lời giải. Chọn điểm gốc của hệ toạ độ vuông góc là A1 .
Khi đó
a2 + λ 1 a3 a3 λ 3 a2
b1 = , b2 = , b3 =
1 + λ1 1 + λ2 1 + λ3
Ta có
1
S B1 B2 B3 = Im(b2 b1 + b3 b2 + b1 b3 )
2
1 a3 a2 + λ1 a3 λ3 a2 a3 a2 + λ1 a3 λ3 a2
= Im( . + . + . )
2 1 + λ2 1 + λ1 1 + λ3 1 + λ2 1 + λ1 1 + λ3
1 + λ1 λ2 λ3 1
= . Im(a3 a2 )
(1 + λ1 )(1 + λ2 )(1 + λ3 ) 2
1 + λ1 λ2 λ3
= .SA1 A2 A3
(1 + λ1 )(1 + λ2 )(1 + λ3 )
Dễ dàng thấy rằng nếu B1 ,B2 ,B3 nằm trên một đường thẳng thì diện
tích tam giác B1 B2 B3 bằng không nghĩa là λ1 λ2 λ3 = −1, ngược lại
nếu λ1 λ2 λ3 = −1 thì những điểm B1 , B2 , B3 thẳng hàng. J
12.10. Lời giải và gợi ý bài tập chương 11 161

. 11.13. Gợi ý: Chọn A1 là điểm đầu của hệ toạ độ vuông góc . c1 ,


c2 , c3 tính được theo công thức ví dụ các chương trước
SC1 C2 C3 (λ1 λ2 λ3 − 1)2
=
SA1 A2 A3 (1 + λ1 + λ1 λ2 )(1 + λ2 + λ2 λ3 )(1 + λ3 + λ3 λ1 )

. 11.14. Gợi ý: Chọn K là điểm gốc của hệ toạ độ vuông góc . Dễ


1
dàng thấy j = (a + b + c + d), khi đó
4
k+o 1
= j =⇒ o = 2j =⇒ o = (a + b + c + d)
2 2
Thế các giá trị trên vào biểu thức
1
SAM OQ = Im(ma + om + qo + aq)
2
sẽ tính ra kết quả(M, Q là trung điểm AO, AB).
. 11.15. Lời giải. Chọn gốc toạ độ vuông góc là điểm M , nghĩa là
m = 0. Khi đó nhãn của tứ giác có đỉnh là những điểm đoos i xứng
với M qua trung điểm các cạnh là a + b, b + c, c + d, d + a. Diện tích
của tứ giác này là
1
S1 = Im((b + c)(a + b) + (c + d)(b + c) + (d + a)(c + d) + (a + b)(d + a))
2
1 1
= Im2(ba + cb + dc + ad) + Im(|a|2 + |b|2 + |c|2 + |d|2 )
2 2
1
+ Im(ca + db + ac + bd)
2
1
= 2 Im(ba + cb + dc + ad)
2
Do |a|2 + |b|2 + |c|2 + |d|2 và ba + cb + dc + ad là số thực . Vậy S1 = 2S.
J
Chương 13
VẤN ĐỀ TIẾP TỤC
VÀ BÀI TẬP TỰ GIẢI

13.1. Vai trò như nhau của các nhãn điểm . . . . . . . . . . . . . 162
13.2. Những định lý nổi tiếng trong hình học phẳng . . . . 165
13.3. Lời cuối cùng . . . . . . . . . . . . . . . . . . . . . . . . . . . . . . . . . . . . . . . 173
13.4. Bài tập tự giải . . . . . . . . . . . . . . . . . . . . . . . . . . . . . . . . . . . . . . 174

13.1. VAI TRÒ NHƯ NHAU CỦA CÁC NHÃN ĐIỂM


Nếu chúng ta để ý kỹ một số cách giải các bài toán ở chương
trước thì vai trò các nhãn của đỉnh của một tam giác hoặc tứ giác
là như nhau. Khi ta tính toán ra một biểu thức có tính đối xứng khi
thay thế chúng cho nhau thì đưa đến kết luận đối xứng nhau (hoặc
tương tự nhau). Thông qua mấy ví dụ sau chúng ta hiểu vấn đềt
một cách cụ thể hơn.

Ví dụ 13.1. Cho tứ giác ABCD nội tiếp trong đường tròn. Chứng
minh rằng đường phân giác các góc DAB và BCD cắt nhau trên
đuwờng chéo BD khi và chỉ khi đuwờng phân giác góc ABC và ADC
cắt nhau trên đuwờng chéo AC.

Lời giải. Ký hiệu P là điểm thứ hai mà đường phân giác góc DAB
cắt đường tròn ngoại tiếp tứ giác ABCD. Dễ thấy rằng điểm Q đối

162
13.1. Vai trò như nhau của các nhãn điểm 163

xứng qua tâm đường tròn của P là điểm cắt của đường phân giác
_
góc BCD. Vì P và Q là điểm giữa của các cung BD, nên p + q = 0
và p2 = q 2 = bd. K là điểm cắt của hai đuwờng phân giác. Khi đó
ap(c + q) − cq(a + p) a(c − p) + (a + p)c
k= =
ap − cq a+c
k−b k−b
Do điều kiện thẳng hàng của B, D và K : b−d
= b−d , thay giá trị k
vào ta nhận được
−2acp + ap2 − cp2 + abp + bcp = 2bpd − dbc + abd − adp − dcp
nhưng ap2 − cp2 = abd − bcd và đẳng thức trên có thể viết lại
2(ac + bd) = (a + c)(b + d) J
Biểu thức cuối cùng đối xứng giữa a và c, tương ứng với b và
d. Như vậy điều kiện cần và đủ để hai đuwờng phân giác góc đoos
i diện cắt đuw ờng chéo đuwợc chứng minh. Ta thấy vai trò của a
thay cho b, c thay cho d và ngược lại. Hơn nữa nhãn p không tham
gia trực tiếp vào công thức, do đó chúng ta có thể thay nó bằng q
khi chứng minh, từ đây ta có thể sinh ra loạt bài toán tương tự.

Ví dụ 13.2. Cho tứ giác ABCD nội tiếp đường tròn và M là điểm cắt
của những tiếp tuyến tại A và C với đường tròn, N là điểm cắt của
tiếp tuyến tại B và D. Cho AA0 , BB 0 , CC 0 , DD0 là các đường phân
giác trong của tứ giác với A0 , B 0 , C 0 , D0 nằm trên đường tròn. Những
đường thẳng AA0 và CC 0 cắt nhau tại K, AC 0 và A0 C cắt nhau tại
K 0 , BB 0 và DD0 cắt nhau tại L và B 0 D và D0 B cắt nhau tại L0 . Khi
đó mệnh đề sau tương đương:
1) Những điểm A, C, N thẳng hàng.
2) Những điểm B, D, M thẳng hàng.
3) Những điểm A, C, L thẳng hàng.
164 Chương 13. Vấn đề tiếp tục và bài tập tự giải

4) Những điểm B, D, K thẳng hàng.


5) Những điểm A, C, L0 thẳng hàng.
6) Những điểm B, D, K 0 thẳng hàng.

Bằng cách tương tự chúng ta giải các bài tập sau:

Ví dụ 13.3. Điểm giữa của các đường chéo AC và BD của một tứ


giác ABCD nội tiếp đường tròn tương ứng là M và N . Chứng minh
rằng BD là đường phân giác của góc AN C khi và chỉ khi AC là phân
giác của góc BM D.

Ví dụ 13.4. AB và CD là hai dây cung của một đường tròn. Chứng


minh rằng nếu những đường thẳng nối A với điểm giữa của CD và B
cũng với điểm giữa CD tạo thành những góc bằng nhau với CD, thì
những đường thẳng nối C và D với điểm giữa AB cũng tạo ra những
góc cùng số đo như nhau với AB.

Chúng ta xét thêm một ví dụ nữa.

Ví dụ 13.5. Tứ giác ABCD ngoại tiếp đường tròn k. Hai tiếp


tuyến bất kỳ l1 và l2 đối với đường tròn khác các cạnh của tứ giác.
Khoảng cách từ các đỉnh A, B, C, D đến l1 và l2 tương ứng ký hiệu là
a1 , b1 , c1 , d1 và a2 , b2 , c2 , d2 . Chứng minh rằng a1 b2 c1 d2 = a2 b1 c2 d1 .

Lời giải. Phương trình được chứng minh tương đương với
a1 c1 a2 c2
= . Gọi P1 và P2 là các tiếp điểm tương ứng của l1 và
b1 d1 b2 d2
l2 với k. Ta ký hiệu T1 , T2 , T3 , T4 là các điểm tiếp xúc của tứ giác
ABCD với đường tròn k. Khoảng cách a1 từ điểm A đến l1 sẽ bằng

1 1
a1 = (2p1 − a − p1 a) = |2p1 − a − p21 a|
2
2 2
13.2. Những định lý nổi tiếng trong hình học phẳng 165

2t1 t2
và vì a = , nên
t1 + t2

1 2t1 t2 2 2 (p1 − t1 )(p1 − t2 )
a1 = 2p1 − − p1 =
2 t1 + t2 t1 + t2 t1 + t2

Tương tự ta có

(p1 − t2 )(p1 − t3 )
b1 =
,
t2 + t3

(p1 − t3 )(p1 − t4 )
c1 = ,
t3 + t4

(p1 − t4 )(p1 − t1 )
d1 =
t4 + t1

a1 c1 (t2 + t3 )(t4 + t1 )
Suy ra =
, biểu thức này không phụ
b1 d1 (t1 + t2 )(t3 + t4 )
thuộc vào đường thẳng l1 . J
13.2. NHỮNG ĐỊNH LÝ NỔI TIẾNG TRONG
HÌNH HỌC PHẲNG
Phương pháp số phức đã chứng minh được rất nhiều định lý nổi
tiếng trong hình học tam giác mà ta đã gặp trong các chương trước,
như các đường và điểm Euler, đường thẳng Simson,.... Phần này
chúng ta đề cập đến một số định lý khó.

Ví dụ 13.6. [Định lý Brôca] Chứng minh rằng trong một tam giác
bất kỳ ABC tồn tại một điểm P sao cho

CP
\ B = AP
[ C = BP
\ A=ϕ

cotg ϕ = cotg α + cotg β + cotg γ,


ở đây α, β, γ là góc ở đỉnh của tam giác ABC tương ứng với A, B, C.
(Điểm P gọi là điểm Brôca)
166 Chương 13. Vấn đề tiếp tục và bài tập tự giải

Lời giải. Chọn hệ toạ độ sao cho đường tròn ngoại tiếp tam giác
ABC làm đơn vị. Các nhãn của đỉnh tam giác có tính chất như
trong các bài tập trước đã xét. Ở đây chúng ta dùng một tính chất
của số phức: nếu z là một số phức bất kỳ, khác không, thì zz bằng
cos 2ϕ + i sin 2ϕ, ở đây ϕ là argumen của z. Thực chất của định lý là
phải tìm được một số phức p và góc ϕ sao cho
p−a p−a p−b p−b p−c p−c
: = : = : = cos 2ϕ + i sin 2ϕ
b−a b−a c−b c−b a−c a−c
Những đẳng thức trên có thể viết lại
(p − a)(b − a) (p − b)(c − b) (p − c)(a − c)
= = = cos 2ϕ + i sin 2ϕ
(b − a)(p − a) (c − b)(p − b) (a − c)(p − c)

Mặt khác ta luôn có công thức sau (kiểm tra dễ dàng)


1 + cos 2ϕ + i sin 2ϕ
= i cotg ϕ
1 − cos 2ϕ − i sin 2ϕ
đồng thời áp dụng tính chất tỷ lệ thuận ta nhận được
(p − a)(b − a) + (b − a)(p − a) + (p − b)(c − b)+
...
(p − a)(b − a) − (b − a)(p − a) + (p − b)(c − b)−

(c − b)(p − b) + (p − c)(a − c) + (a − c)(a − c)


... = i cotg ϕ
(c − b)(p − b) + (p − c)(a − c) − (a − c)(a − c)
Từ đó suy ra

−a(b − a) − (b − a)a − b(c − b) − (c − b)b − c(a − c) − (a − c)c


a(b − a) − (b − a)a + b(c − b) − (c − b)b + c(a − c) − (a − c)c
= i cotg ϕ

Rút gọn công thức trên


(b − a)(b − a) + (c − b)(c − b) + (a − c)(a − c)
i cotg ϕ =
−ba − cb − ac + ab + bc + ca
13.2. Những định lý nổi tiếng trong hình học phẳng 167

Số ở tử là hoàn toàn ảo, ngoài ra công thức diện tích S của tam giác
ABC là S = 12 Im(ba + cb + ac). Suy ra
|b − a|2 + |c − b|2 + |a − c|2
cotg ϕ = .
4S
Đặt p = |c − b|, q = |a − c|, r = |b − a|. Ta có
cos α cos β cos γ
cotg α + cotg β + cotg γ = + +
sin α sin β sin γ
2qr cos α 2rp cos β 2pq cos γ
= + +
2qr sin α 2rp sin β 2pq sin γ
2 2 2 2
(q + r − p ) + (r + p − q 2
=
4S
p2 + q 2 + r 2 |b − a|2 + |c − b|2 + |a − c|2
= =
4S 4S
= cotg ϕ
Như vậy đẳng thức thứ nhất được chứng minh. Ta đặt
w = cos 2ϕ + i sin 2ϕ. Khi đó
(p − a)(b − a)(c − b)
= (p − a)(c − b)w
b−a
(p − b)(c − b)(a − c)
= (p − b)(a − c)w
c−b
(p − c)(a − c)(b − a)
= (p − c)(b − a)w
a−c
Cộng ba phương trình lại ta được
(p − a)(b − a)(c − b) (p − b)(c − b)(a − c) (p − c)(a − c)(b − a)
+ + =0
b−a c−b a−c
Từ đó suy ra
ab(c − b) + bc(a − c) + ca(b − a)
p=
(c − b)b + (a − c)c + (b − a)a
J
168 Chương 13. Vấn đề tiếp tục và bài tập tự giải

Trong hình học người ta đưa vào khái niệm đường thẳng đẳng
giác: Hai đường thẳng cùng xuất phát từ đỉnh của một góc và đối
xứng nhau qua đường phân giác của góc đó gọi là những đường
thẳng đẳng giác.
Ví dụ 13.7. Nếu ba đường thẳng xuất phát từ ba đỉnh của một tam
giác cắt nhau tại một điểm thì những đường đẳng giác của chúng cũng
cắt nhau tại một điểm.

Lời giải. Chọn đường tròn ngoại tiếp tam giác ABC làm đường
tròn đơn vị. P là điểm cắt của ba đường thẳng cho trước (giả thiết
P không nằm trên đườ ng tròn). Ký hiệu A0 , B0 , C0 là điểm cắt
của các phân giác trong với đường tròn. Vì các điểm A0 , B0 , C0 lần
_ _ _
lượt là các trung điểm của BC, CA và AB, nên a20 = bc, b20 = ca,
c20 = ab. Ngoài ra a20 a2 = bca2 = (ca)(ab) = b20 c20 từ đó suy ra
(a0 a − b0 c0 )(a0 a + b0 c0 ) = 0. Nhưng a0 a − b0 c0 6= 0 , vì nếu ngược
lại thì AA0 và B0 C0 song song với nhau, điều này không thể xảy ra.
Suy ra a0 a + b0 c0 = 0 nghĩa là AA0 và B0 C0 vuông góc với nhau.
Tương tự cho các đường phân giác khác cũng vậy nghĩa là I là trực
tâm của tam giác A0 B0 C0 . Nhưng cùng nằm trên một đường tròn
đơn vị vậy j = a0 + b0 + c0 với j là nhãn của I.
Ký hiệu PA là điểm đối xứng của P qua đường phân giác AA0 .
Khi đó công thức tính hai đường vuông góc có p + pA = a + a0 + p −
aa0 p, suy ra pA = a + a0 − aa0 p. Ta ký hiệu ZA là điểm thứ hai của
đường thẳng APA với đường tròn, ta có
bc(ap − 1)
zA + a = pA + azA pA =⇒ zA = .
a−p
Tương tự ta có thể chứng minh được điểm PB và ZB :
ca(bp − 1)
zB =
b−p
13.2. Những định lý nổi tiếng trong hình học phẳng 169

Những đường thẳng APA và BPB là đối xứng của AP và BP


qua đường phân giác cắt nhau tại điểm Q, nó có nhãn
(a + zA )bzB − (b + zB )azA
q=
bzB − azA
bc(ap−1) abc(bp−1)
[a + a−p ] b−p − [b + ac(bp−1)
b−p ]
abc(ap−1)
a−p
= abc(bp−1) abc(ap−1)
b−p − a−p
2
abcp − (ab + bc + ca)p − p + a + b + c
=
1 − pp
Biểu thức trong q đối xứng với a, b, c do đó đường thẳng qua C đối
xứng của CP đối với CC0 cũng đi qua điểm Q.
Trường hợp điểm P nằm trên đường tròn dẫn đến ba đường
thẳng APA , BPB , CPC song song với nhau chứng minh dễ dàng. J
Trong chứng minh trên hai điểm P và Q gọi là hai điểm đẳng
giác với nhau. Mỗi điểm trong mặt phẳng tam giác không nằm trên
đường tròn có duy nhất một điểm đẳng giác. Nhãn của điểm đẳng
giác được tính theo công thức sau
abcp2 − (ab + bc + ca)p − p + a + b + c
q=
1 − pp
Ta biến đổi công thức
p2 − (a + b + c)p − abcp + ab + bc + ca
q=
abc(1 − pp)
suy ra
1
p + q + abcpq = [p − p2 p + abcp2 − (ab + bc + ca)p
1 − pp
−p + a + b + c + pp2 − (a + b + c)pp = abcp2 + (ab + bc + ca)p]
=a+b+c
Công thức p + q + abcpq = a + b + c nổi tiếng với tên gọi F. Morley.
170 Chương 13. Vấn đề tiếp tục và bài tập tự giải

Ta xét một số cặp điểm đẳng giác điển hình có nhãn tính theo
công thức trên:
1) Điểm tâm đường tròn nội tiếp tam giác là điểm đẳng giác với
chính nó p = q theo công thức trên (gọi j là nhãn của I) và tính ra

1 2
j = (a + b + c − abcj )
2
2) Điểm đẳng giác của trực tâm tam giác là tâm đường tròn
ngoại tiếp tam giác đó, thật vậy, p = h = a + b + c theo công thức
trên
abc(a + b + c)2 − (ab + bc + ca)(a + b + c) − a − b − c + a + b + c
q=
1 − (a + b + c)(a + b + c)
(a + b + c)[abc(a + b + c) − (ab + bc + ca)]
= =0
1 − (a + b + c)(a + b + c)

3) Điểm đẳng giác với trọng tâm của tam giác là điểm Lemoan
và là giao điểm của ba đường đối trung. Nhãn của điểm này theo
công thức tính được
(ab + bc + ca)(a + b + c) − 3(a + b + c)
l=2
(a + b + c)(a + b + c) − 9

4) Bài trước ta có nhãn của điểm Brôca, gọi là điểm Brôca thứ
nhất, còn nhãn của điểm đẳng giác với nó cũng tính được theo công
thức trên.
Bằng cách tính tương tự ta tính được các nhãn của các điểm đặc
biệt khác và dùng nhãn này để chứng minh các bài tập có liên quan
đến chúng.

Ví dụ 13.8. [Định lý Feuebach] Đường tròn chín điểm của một tam
giác tiếp xúc với đường tròn nội tiếp của tam giác đó.
13.2. Những định lý nổi tiếng trong hình học phẳng 171

Lời giải. Ta lại chọn đường tròn ngoại tiếp tam giác ABC làm đơn
vị. Ký hiệu I, Ia , Ib , Ic tương ứng là tâm của đường tròn nội tiếp và
các đường tròn bàng tiếp tương ứng với các đỉnh của tam giác ABC,
và chúng có nhãn là j, ja , jb , jc . Ta ký hiệu A0 , B0 , C0 là những điểm
đường phân giác trong của các góc CAB, ABC, CBA cắt đường
tròn ngoại tiếp tam giác ABC. Dễ thấy rằng I là trực tâm của tam
giác A0 B0 C0 , như ta đã biết nó có nhãn j = a0 + b0 + c0 . Điểm Ia
nằm trên đường kéo dài AI và hai đường phân giác ngoài trên hai
đỉnh còn lại. Hai đường phân giác ngoài này cắt đường tròn ngoại
tiếp tại hai điểm tương ứng đối xứng với B0 và C0 qua tâm đường
tròn, khi đó Ia là trực tâm của tam giác có đir nh như vừa xác đij nh
nằm trên đường tròn đã cho và có nhãn ja = a0 − b0 − c0 . Tương tự
ta có jb = −a0 + b0 − c0 và jc = −a0 − b0 + c0 . Ta biểu diễn
j + ja j + jb j + jc
= a0 , = b0 , = c0 ,
2 2 2
nghĩa là trung điểm của IIa , IIb và IIc trùng với các điểm
A0 , B0 , C0 . Ngoài ra
jb + jc jc + ja ja + jb
= −a0 , = −b0 , = −c0 ,
2 2 2
nghĩa là trung điểm của các đoạn Ib Ic , Ic Ia và Ia Ib trùng với các
điểm đối xứng qua tâm của A0 , B0 , C0 . Cũng bằng cách dùng các
công thức đã tính ta sẽ chứng minh được IIa vuông góc với Ib Ic .
Thật vậy, j − ja = 2(b0 + c0 ) và jb − jc = 2(b0 − c0 ). Tương tự chứng
minh cho các đỉnh còn lại của tam giác Ia Ib Ic , như vậy trực tâm
của tam giác Ia Ib Ic trùng với tâm của đường tròn nội tiếp I.
Từ công thức Euler cho khoảng cách giữa hai tâm của đường
tròn ngoại tiếp và nội tiếp ta có

jj = OI 2 = R2 − 2rR = 1 − 2r.
172 Chương 13. Vấn đề tiếp tục và bài tập tự giải

Như vậy bán kính đường tròn nội tiếp r = 21 (1 − jj). Ta xét các điểm
F xác định theo nhãn của chúng:
1 j
f = (a + b + c − abc ).
2 j
1 2
Điểm chú ý 1) bài 13.7 chỉ ra rằng j = (a + b + c − abcj ). Khi đó
2
1 1 1 j j
f − j = abcj( − j) = abc (1 − jj) = abc r. Nghĩa là |F I| = r ,
2 j 2 j j
như vậy F là điểm nằm trên đường tròn nội tiếp. Hơn nữa
1 j
e − f = abc cũng có nghĩa là F nằm trên đường tròn chín điểm.
2 j
f −j r
Ngoài ra = là một số thực, vậy F nằm trên đường thẳng nối
e−j 2
hai điểm tâm đường tròn nội tiếp I và tâm đường tròn chín điểm E,
khả năng duy nhất xảy ra là hai đường tròn tiếp xúc với nhau tại F .
J
Trong cách chứng minh trên ta thấy vai trò các đường tròn bàng
tiếp cũng như đường tròn nội tiếp, nên ta có

Ví dụ 13.9. Đường tròn Euler chín điểm của một tam giác tiếp xúc
với mọi đường tròn bàng tiếp của tam giác này.

Lời giải. Chứng minh hoàn toàn tương tự như bài tập trên, nhưng
lần này ta lấy các điểm Fa , Fb , Fc theo
1 ja 1 jb 1 jc
fa = (a+b+c−abc ), fb = (a+b+c−abc ), fc = (a+b+c−abc ),
2 ja 2 jb 2 jc
và tâm của các đường tròn bàng tiếp tính theo điểm tự đẳng giác
OIa2 = ja ja = R2 + 2ra R = 1 + 2ra ,
OIb2 = jb jb = R2 + 2rb R = 1 + 2rb ,
OIc2 = jc jc = R2 + 2rc R = 1 + 2rc ,
13.3. Lời cuối cùng 173

Rất nhiều điểm và đường đặc biệt trong tam giác ta có thể chứng
minh được bằng phương pháp số phức, đây cũng là một chuyên đề
rất hay mà ta cần làm và khảo sát đến cùng. Để kết thúc phần này
ta có thể nêu lên vài định lý nổi tiếng, cách chứng minh không khó.
J
Ví dụ 13.10. [Định lý Pascal] Nếu một lục giác nội tiếp trong đường
tròn và các điểm L, M, N tương ứng là các điểm cắt của các cạnh AB
và DE, BC và EF , CD và F A, thì các điểm L, M và N nằm trên
một đường thẳng.

Ví dụ 13.11. [Định lý Brianston] Nếu lục giác ABCDEF ngoại tiếp


đường tròn, thì các đường chéo AD, BE và CF là song song hoặc cắt
nhau tại một điểm.

Ví dụ 13.12. [Định lý Newton] Những điểm giữa của các đường chéo
trong một tứ giác đầy đủ thẳng hàng.(Tứ giác đầy đủ là hình tạo bởi
điểm cắt bốn đường thẳng, mà không ba đường thẳng nào cắt nhau
tại một điểm).

13.3. Lời cuối cùng


Mỗi phương pháp giải bài tập chỉ thực sự mạnh với một lớp bài
toán nào đó. Lớp bài toán chúng ta đã xét có thể chứng minh bằng
cách khác có thể dễ hơn hoặc có thể có bài khó hơn cách chứng
minh ở đây. Trong chứng minh bằng phương pháp số phức ta phải
luôn chọn một hệ toạ độ cho thuận tiện tính toán và nhiều khi
trong chứng minh cũng dùng cách phân tích mà ta thường dùng,
đã học. Thế thì phương pháp này nằm ở đâu trong số các phương
pháp chứng minh hình học phẳng mà ta đã biết như phương pháp
phân tích theo tiên đề, phương pháp hệ toạ độ Đề các, phương pháp
174 Chương 13. Vấn đề tiếp tục và bài tập tự giải

véc tơ, ... theo tôi nghĩ đây là phương pháp nằm giữa phương pháp
toạ độ Đề các và phương pháp véc tơ. Chính điều kết hợp này mà
phương pháp có nhiều thế mạnh trong chứng minh như ta đã thấy.
Những chủ đề chúng tôi liệt kê trên đây còn chưa đầy đủ, mong
các bạn góp ý và tìm thêm các ví dụ và chứng minh hay cho phương
pháp này, vì khuôn khổ cuốn sách nên còn nhiều bài tập trong các
tài liệu tôi có chưa đưa vào đây được, hẹn các bạn dịp khác. Có
bạn sẽ tự hỏi thế thì có loại số phức nào chứng minh cho các bài
toán hình học không gian ? Theo chúng tôi biết có một loại số khác
gần giống số phức trong không gian, đó là các số Katerion, được
Hamilton khai sinh, phát triển ở đầu thế kỷ 20 và ngày nay cũng có
rất nhiều ứng dụng. Nhưng rất tiếc để áp dụng giải các bài toán hình
học không gian thì rất hạn chế và không đẹp như số phức chúng ta
vừa khảo sát.

13.4. Bài tập tự giải


Các bài tập dưới đây đã có lời giải, nhưng để bạn đọc tích cực
áp dụng phương pháp số phức tôi không liệt kê ra đây.

. 13.13. Qua tâm của một đa giác đều n đỉnh dựng một đường
thẳng bất kỳ. Chứng minh rằng tổng S các bình phương khoảng
cách từ các đỉnh của đa giác đến đường thẳng không phụ thuộc vào
vị trí đường thẳng.

. 13.14. Cho tam giác ABC, Cb = 90◦ . Gọi D là chân đường vuông
góc hạ từ đỉnh C. Chứng minh rằng các đường trung tuyến của các
tam giác ADC và DBC vuông góc với nhau.

. 13.15. Cho đa giác đều n đỉnh A1 , A2 , . . . , An nội tiếp trong đường


tròn tâm O , bán kính r. P là một điểm kéo dài trên OA1 . Chứng
13.4. Bài tập tự giải 175

minh rằng P A1 + P A2 + · · · + P An = OP n − rn .

. 13.16. Cho P nằm trên đường tròn đơn vị và các đỉnh đa giác đều
n cạnh A1 , A2 , . . . , An cũng nằm trên đường tròn này. Chứng minh
rằng P A21 + P A22 + · · · + P A2n = 2n .

. 13.17. Cho A0 , A1 , A2 , A3 , A4 chia đường tròn đơn vị ra 5 phần


bằng nhau. Chứng minh rằng những dây cung A0 A1 , A0 A2 thỏa
mãn
(A0 A1 .A0 A2 )2 = 5.

. 13.18. Cho P nằm trên đường tròn đơn vị và các đỉnh đa giác đều
n cạnh A1 , A2 , . . . , An cũng nằm trên đường tròn này. Chứng minh
rằng P A41 + P A42 + · · · + P A4n là hằng số (không phụ thuộc vào vị
trí của P trên đường tròn).

. 13.19. Cho G là trọng tâm của tam giác ABC. Chứng minh rằng
3(GA2 + GB 2 + GC 2 ) = AB 2 + BC 2 + CA2 .

. 13.20. Cho lục giác đều ABCDEF trong đường tròn bán kính r.
Chứng minh rằng nếu AB = CD = EF = r, thì những điểm giữa
của BC, DE và F A là đỉnh của một đa giác đều.

. 13.21. Cho các điểm D, E, F trong mặt phẳng tam giác ABC sao
cho các tam giác BDC, CEA và AF B đồng dạng. Chứng minh rằng
1) Trọng tâm của các tam giác DEF trùng với trọng tâm tam
giác ABC.
2) Nếu tam giác ABC không phải là tam giác đều, thì tam giác
DEF là tam giác đều khi và chỉ khi ba tam giác BDC, CEA và
AF B là tam giác cân đáy là các cạnh của tam giác ABC và góc ở
đỉnh cân là 30◦ .
176 Chương 13. Vấn đề tiếp tục và bài tập tự giải

. 13.22. Trên các cạnh AB và CD của đa giác bất kỳ ABCD dựng


ra phía ngoài những hình vuông ABM N và DCKL. Chứng minh
rằng trung điểm của những đường chéo tứ giác ABCD và M N KL
trùng nhau hoặc là đỉnh của một hình vuông.

. 13.23. Trong mặt phẳng hình chữ nhật A1 A2 A3 A4 lấy điểm O bất
kỳ. Ký hiệu Oj (j = 1, 2, 3, 4) là những điểm đối xứng của O qua các
cạnh Aj Aj+1 (j = 1, 2, 3, 4, A5 = A1 ). Quay điểm Oj quanh điểm Aj
theo chiều kim đồng hồ một góc 90◦ và nhận được điểm Bj . Chứng
minh rằng diện tích tứ giác B1 B2 B3 B4 không phụ thuộc vào vị trí
của điểm O.

. 13.24. Cho P nằm trên đường tròn ngoại tiếp tam giác ABC. Tại
P quay các đỉnh của tam giác đi một góc ϕ biến đổi A, B, C thành
A1 , B1 , C1 . Chứng minh rằng những đường thẳng AA1 , BB1 , CC1
cắt nhau tại một điểm M . Tìm quĩ tích điểm M khi 0 ≤ ϕ ≤ 2π.

. 13.25. Cho tam giác ABC. Cạnh BC chia làm ba phần bằng nhau
bởi các điểm A1 và A2 . Chứng minh rằng hiệu của những bình
phương các đoạn AA1 và AA2 bằng 3 lần hiệu của bình phương
các cạnh tương ứng xuất phát từ A.

. 13.26. Trong tứ giác lồi ABCD những cạnh AB và CD cắt nhau


tại điểm O. Trên cạnh BC và AD lấy các điểm M, N, K, L sao cho
BM = M N = N C và AK = KL = LD. Chứng minh rằng diện tích
của tam giác OKM và OLN bằng nhau.

. 13.27. Cho tứ giác lồi A1 A2 A3 A4 . Điểm B1 là đối xứng của A1


qua A2 , B2 là đối xứng của A2 qua A3 , B3 - của A3 qua A4 và B4 -
của A4 qua A1 . Chứng minh rằng diện tích của tứ giác B1 B2 B3 B4
bằng 5 lần diện tích của tứ giác A1 A2 A3 A4 .
13.4. Bài tập tự giải 177

. 13.28. Từ một điểm M kẻ hai đường tiếp tuyến với đường tròn
(k) cho trước (A ∈ k, B ∈ k). Tìm tất cả những điểm P khác A, B
sao cho những điểm cắt thứ hai của các đường thẳng P A và P B với
đường tròn tạo thành đường kính.

. 13.29. Những đường tiếp tuyến tại các đỉnh của một tam giác
ABC đối với đường tròn ngoại tiếp tam giác này, cắt nhau tạo thành
tam giác A0 B0 C0 . Chân đường cao của tam giác ABC ký hiệu tương
ứng là A1 , B1 , C1 . Chứng minh rằng những tam giác A0 B0 C0 và
A1 B1 C1 là vị tự của nhau.

. 13.30. Cho tam giác ABC bất kỳ và điểm P nằm trên đường tròn
ngoại tiếp tam giác. Qua điểm A dựng đường thẳng song song với
OP (O là tâm đường tròn), mà nó cắt đường tròn ở điểm thứ hai A0 .
Lấy Q là điểm đối xứng của A0 qua đường kính của đường tròn mà
nó song song với BC. Chứng minh rằng bằng cách xây dựng tương
tự như vậy đối với các đỉnh B và C, kết quả đều dẫn tới điểm Q.

. 13.31. Qua một điểm bất kỳ P trên đường tròn ngoại tiếp tam
giác ABC, dựng những đường thẳng song song với các cạnh của
tam giác BC, CA, AB. Ký hiệu A0 , B 0 , C 0 là các điểm cắt thứ hai
của các đường thẳng trên đối với đường tròn. Lấy A00 , B 00 , C 00 là
những điểm đối xứng của các đỉnh A, B, C đối với các đường thẳng
B 0 C 0 , C 0 A0 , A0 B 0 tương ứng.
1) Chứng minh rằng những tam giác ABC và A00 B 00 C 00 đồng
dạng và ngược hướng.
2) Chứng minh rằng OO00 = P H, với O và O00 là tâm của đường
tròn ngoại tiếp tam giác ABC và A00 B 00 C 00 , còn H là trực tâm tam
giác ABC.
178 Chương 13. Vấn đề tiếp tục và bài tập tự giải

. 13.32. Trong hình chữ nhật ABCD đường phân giác góc B cắt
đường chéo AC và cạnh AD lần lượt tại E và F . Qua E dựng đường
thẳng song song với AB, mà nó cắt đường chéo BD tại K. Chứng
minh rằng đường thẳng F K vuông góc với AC.

. 13.33. Đường thẳng song song với cạnh BC của tam giác ABC
cắt AB tại F và AC tại E. Chứng minh rằng những điểm cắt của
hai đường tròn đường kính BE và CF nằm trên đường cao của tam
giác ABC hạ từ đỉnh A.
TÀI LIỆU THAM KHẢO

[1] Iv. Tonov, Prilojenie na komplex trisla v geometriata, Sofia 1988

[2] I. M. Iaglom,Komplesni trisla i ix prilojenie v geometrii, Moskva


1964.

[3] P. S. Modenov, Problems in geometry, Moscow 1981.

[4] Loren C. Larson, Problem-solving through problems, Springer-


Verlag 1981

[5] L.Davidov,V. Petkov, Iv. Tonov, Vl. Chukanov, Matematicheski


konkursi, Sofia 1977.

179
MỤC LỤC

Lời nói đầu . . . . . . . . . . . . . . . . . . . . . . . . . . . . . . . . . . . . . . . . . . . . . . . . . . . 3


Chương 1. Khái niệm về số phức . . . . . . . . . . . . . . . . . . . . . . . . . . . . 5
1.1. Định nghĩa số phức. . . . . . . . . . . . . . . . . . . . . . . . . . . . . . . . . . . . . 5
1.2. Biểu diễn đại số của số phức. . . . . . . . . . . . . . . . . . . . . . . . . . . . 7
1.3. Dạng lượng giác của số phức . . . . . . . . . . . . . . . . . . . . . . . . . . . 9
1.4. Công thức Moa vrơ. . . . . . . . . . . . . . . . . . . . . . . . . . . . . . . . . . . . 12
Chương 2. Độ đo góc của hai tia . . . . . . . . . . . . . . . . . . . . . . . . . . . 15
2.1. Góc định hướng . . . . . . . . . . . . . . . . . . . . . . . . . . . . . . . . . . . . . . . 15
2.2. Ví dụ . . . . . . . . . . . . . . . . . . . . . . . . . . . . . . . . . . . . . . . . . . . . . . . . . 18
2.3. Bài tập. . . . . . . . . . . . . . . . . . . . . . . . . . . . . . . . . . . . . . . . . . . . . . . . 22
Chương 3. Phương trình đường thẳng . . . . . . . . . . . . . . . . . . . . . 24
3.1. Đường thẳng qua hai điểm . . . . . . . . . . . . . . . . . . . . . . . . . . . . 24
3.2. Phương trình tham số . . . . . . . . . . . . . . . . . . . . . . . . . . . . . . . . . 25
3.3. Ví dụ . . . . . . . . . . . . . . . . . . . . . . . . . . . . . . . . . . . . . . . . . . . . . . . . . 26
3.4. Bài tập. . . . . . . . . . . . . . . . . . . . . . . . . . . . . . . . . . . . . . . . . . . . . . . . 32
Chương 4. Phương trình đường tròn . . . . . . . . . . . . . . . . . . . . . . . 34
4.1. Phương trình tổng quát . . . . . . . . . . . . . . . . . . . . . . . . . . . . . . . 34
4.2. Đường tròn đơn vị . . . . . . . . . . . . . . . . . . . . . . . . . . . . . . . . . . . . 37
4.3. Giao điểm hai cát tuyến . . . . . . . . . . . . . . . . . . . . . . . . . . . . . . . 38

180
Phương pháp số phức và hình học phẳng 181

4.4. Giao điểm hai tiếp tuyến . . . . . . . . . . . . . . . . . . . . . . . . . . . . . . 40


4.5. Chân đường vuông góc ở dây cung . . . . . . . . . . . . . . . . . . . . 42
4.6. Bài tập. . . . . . . . . . . . . . . . . . . . . . . . . . . . . . . . . . . . . . . . . . . . . . . . 44
Chương 5. Đường thẳng và đường tròn Euler . . . . . . . . . . . . . . 46
5.1. Nhãn các điểm đăc biệt trong tam giác . . . . . . . . . . . . . . . . 46
5.2. Ví dụ . . . . . . . . . . . . . . . . . . . . . . . . . . . . . . . . . . . . . . . . . . . . . . . . . 49
5.3. Bài tập. . . . . . . . . . . . . . . . . . . . . . . . . . . . . . . . . . . . . . . . . . . . . . . . 52
Chương 6. Đường thẳng Simson . . . . . . . . . . . . . . . . . . . . . . . . . . . 54
6.1. Ba điểm trên đường thẳng Simson . . . . . . . . . . . . . . . . . . . . . 54
6.2. Ví dụ . . . . . . . . . . . . . . . . . . . . . . . . . . . . . . . . . . . . . . . . . . . . . . . . . 55
6.3. Bài tập. . . . . . . . . . . . . . . . . . . . . . . . . . . . . . . . . . . . . . . . . . . . . . . . 59
Chương 7. Tứ giác nội tiếp đường tròn . . . . . . . . . . . . . . . . . . . . 62
7.1. Các điểm đặc biệt của tứ giác nội tiếp . . . . . . . . . . . . . . . . . 62
7.2. Ví dụ . . . . . . . . . . . . . . . . . . . . . . . . . . . . . . . . . . . . . . . . . . . . . . . . . 64
7.3. Bài tập. . . . . . . . . . . . . . . . . . . . . . . . . . . . . . . . . . . . . . . . . . . . . . . . 67
Chương 8. Đường tròn đơn vị nội tiếp . . . . . . . . . . . . . . . . . . . . . 70
8.1. Tọa độ đơn vị mới . . . . . . . . . . . . . . . . . . . . . . . . . . . . . . . . . . . . . 70
8.2. Ví dụ . . . . . . . . . . . . . . . . . . . . . . . . . . . . . . . . . . . . . . . . . . . . . . . . . 71
8.3. Bài tập. . . . . . . . . . . . . . . . . . . . . . . . . . . . . . . . . . . . . . . . . . . . . . . . 78
Chương 9. Tam giác đồng dạng . . . . . . . . . . . . . . . . . . . . . . . . . . . . 81
9.1. Quan hệ đồng dạng của hai tam giác . . . . . . . . . . . . . . . . . . 81
9.2. Ví dụ . . . . . . . . . . . . . . . . . . . . . . . . . . . . . . . . . . . . . . . . . . . . . . . . . 82
9.3. Bài tập. . . . . . . . . . . . . . . . . . . . . . . . . . . . . . . . . . . . . . . . . . . . . . . . 85
182 Mục lục

Chương 10. Đa giác đều . . . . . . . . . . . . . . . . . . . . . . . . . . . . . . . . . . . 88


10.1. Nhãn của đỉnh các đa giác đều . . . . . . . . . . . . . . . . . . . . . . . 88
10.2. Ví dụ . . . . . . . . . . . . . . . . . . . . . . . . . . . . . . . . . . . . . . . . . . . . . . . . 89
10.3. Bài tập . . . . . . . . . . . . . . . . . . . . . . . . . . . . . . . . . . . . . . . . . . . . . . 93
Chương 11. Diện tích đa giác . . . . . . . . . . . . . . . . . . . . . . . . . . . . . . 96
11.1. Công thức tính diện tích . . . . . . . . . . . . . . . . . . . . . . . . . . . . . 96
11.2. Ví dụ . . . . . . . . . . . . . . . . . . . . . . . . . . . . . . . . . . . . . . . . . . . . . . . . 98
11.3. Bài tập . . . . . . . . . . . . . . . . . . . . . . . . . . . . . . . . . . . . . . . . . . . . . 102
Chương 12. Lời giải và gợi ý bài tập . . . . . . . . . . . . . . . . . . . . . . 105
12.1. Lời giải và gợi ý bài tập chương 2 . . . . . . . . . . . . . . . . . . . 105
12.2. Lời giải và gợi ý bài tập chương 3 . . . . . . . . . . . . . . . . . . . 110
12.3. Lời giải và gợi ý bài tập chương 4 . . . . . . . . . . . . . . . . . . . 115
12.4. Lời giải và gợi ý bài tập chương 5 . . . . . . . . . . . . . . . . . . . 123
12.5. Lời giải và gợi ý bài tập chương 6 . . . . . . . . . . . . . . . . . . . 128
12.6. Lời giải và gợi ý bài tập chương 7 . . . . . . . . . . . . . . . . . . . 132
12.7. Lời giải và gợi ý bài tập chương 8 . . . . . . . . . . . . . . . . . . . 139
12.8. Lời giải và gợi ý bài tập chương 9 . . . . . . . . . . . . . . . . . . . 147
12.9. Lời giải và gợi ý bài tập chương 10 . . . . . . . . . . . . . . . . . . 152
12.10. Lời giải và gợi ý bài tập chương 11. . . . . . . . . . . . . . . . . 158
Chương 13. Vấn đề tiếp tục và bài tập tự giải . . . . . . . . . . . . 162
13.1. Vai trò như nhau của các nhãn điểm . . . . . . . . . . . . . . . . 162
13.2. Những định lý nổi tiếng trong hình học phẳng . . . . . . 165
13.3. Lời cuối cùng . . . . . . . . . . . . . . . . . . . . . . . . . . . . . . . . . . . . . . . 173
13.4. Bài tập tự giải . . . . . . . . . . . . . . . . . . . . . . . . . . . . . . . . . . . . . . 174
Phương pháp số phức và hình học phẳng 183

Tài liệu tham khảo . . . . . . . . . . . . . . . . . . . . . . . . . . . . . . . . . . . . . . . . 179


Mục lục . . . . . . . . . . . . . . . . . . . . . . . . . . . . . . . . . . . . . . . . . . . . . . . . . . . . 180

You might also like